ROSH Emergency Medicine

Ace your homework & exams now with Quizwiz!

One Step Further Question: What is the antidote to benzodiazepine poisoning?

Answer: Flumazenil.

What is the dose of dexamethasone for croup in children?

0.6 mg/kg (up to 10 mg maximum dosage) typically given orally

One Step Further Question: What is the treatment for ethylene glycol poisoning?

Answer: Fomepizole.

Question: What is a normal PR interval?

120-200 ms.

Question: What percentage of patients with a calcaneal fracture will have an associated vertebral compression fracture?

10%.

One Step Further Question: Which of the toxic alcohols has the greatest intoxicating effect?

Answer: Isopropanol.

One Step Further Question: What is the best radiologic view for diagnosing a bilateral facet dislocation?

Answer: Lateral view.

One Step Further Question: What is the expected pulse oximetry finding in methemoglobinemia?

Answer: Low, typically around 85 percent. However, the Pa oxygen will be normal.

One Step Further Question: What other associated traumatic injury is seen most commonly in patients with an epidural hematoma?

Answer: Temporal bone fracture.

One Step Further Question: What is the most commonly injured nerve during a shoulder dislocation?

Answer: The axillary nerve.

AAA Overview

Abdominal Aortic Aneurysm (AAA) Advanced age, male, smoking hx, HTN Acute abdominal pain + hypotension + pulsatile abdominal mass US: 100% sensitive CT: 100% sensitive, detects rupture/leak AAA > 5 cm: ↑ risk of rupture Renal colic in elderly: r/o AAA

Mallet Finger Deformity Question: What are the indications for operative intervention of an acute mallet finger injury?

Absolute indications for surgery include an open mallet finger injury or volar subluxation of the distal phalanx.

Question: What classic eye findings are present in patients with a posterior communicating artery aneurysm?

Acute cranial nerve III palsy with ipsilateral pupillary dilation.

Question: What is the most common cause of small bowel obstruction?

Adhesions from prior surgery.

Question: What are the two most effective methods of increasing elimination of lithium?

Aggressive fluid resuscitation and hemodialysis.

One Step Further Question: What is the treatment for lead poisoning?

Answer: Edetate disodium calcium (EDTA) or succimer.

One Step Further Question: The return of what reflex demonstrates the end of spinal shock?

Answer: The bulbocavernosus reflex. This is performed by placing one finger in the rectum and squeezing the glans of the penis or clitoris leading to rectal sphincter contraction.

One Step Further Question: What is the most injured segment of the spine?

Answer: The cervical spine.

One Step Further Question: What is the most common long bone fracture?

Answer: Tibia.

Question: What symptoms are more likely to be seen in adults with parvovirus B19 infection than in children?

Arthralgias and arthritis.

Question: Supraventricular tachycardia that does not convert with adenosine can often be confused with what rhythm?

Atrial flutter with a 2:1 block.

What is the most common bacterial cause of acute chest syndrome in adults with sickle cell disease?

Atypical bacteria such as Chlamydia pneumoniae, Mycoplasma pneumoniae, Mycoplasma hominis.

: What is the name for an intra-articular fracture at the base of the thumb metacarpal with associated dislocation or subluxation at the carpometacarpal joint?

Bennett's fracture.

Question: What is the clinical presentation of anterior cord syndrome?

Bilateral loss of pain and temperature with preservation of vibration/proprioception and motor weakness below level of cord involvement

Question: What is the expected appearance of the chest radiograph in a patient with high output heart failure?

Bilateral pulmonary edema with a normally-sized heart.

Question: What vessels are the most commonly injured in patients with a subdural hematoma?

Bridging veins.

Rapid Review Carbon Monoxide (CO) Poisoning:

Carbon Monoxide (CO) Poisoning: Patient will be complaining of headache, nausea, and dizziness PE will show cherry-red skin Labs will show carboxyhemoglobin Treatment is 100% oxygen, hyperbaric oxygen

Question: What laboratory procedure must be performed to diagnose xanthochromia?

Centrifugation of the CSF sample.

Question: What is the most common cause of myocarditis worldwide?

Chagas disease caused by the protozoan Trypanosoma cruzi.

Chemical Burns

Chemicals that should not be immediately irrigated with water include dry lime, elemental metals (e.g. sodium, potassium, magnesium, and phosphorus), and phenols. Treatment for elemental metal burns involves removal of the substance and the affected area covered in mineral oil to prevent further exposure to air and moisture. Surgical debridement may be required if fragments are embedded in the wound. After decontamination, chemical burn patients are managed similar to thermal burn patients with fluid resuscitation and prevention of further complications like rhabdomyolysis, infection, and hypothermia. Medical toxicology and poison control consultations should be made in most cases of chemical burns.

Question: What is the treatment of choice for mastitis in beta-lactam sensitive patients?

Clindamycin.

Question: What is the mechanism of aspirin in acute coronary syndrome?

Decreases platelet aggregation by irreversibly inhibiting cyclooxygenase, which catalyzes the thromboxane enzymes.

A 35-year-old man presents to the Emergency Department four hours after being bitten on his left hand by a spider while cleaning out his shed. He currently complains of whole arm pain as well as headache, severe back spasms, and abdominal pain. What is the treatment of choice to control his symptoms? Intravenous calcium gluconate Intravenous diazepam Intravenous ketorolac Intravenous normal saline

Diazepam

Question: Why would a GI bleed increase the risk for hepatic encephalopathy?

Digestion of blood proteins leads to an increase in absorbed nitrogenous waste.

What is the treatment for heparin-induced thrombocytopenia (HIT)?

Discontinue heparin and start a direct thrombin inhibitor.

Question: What chemotherapeutic agent is associated with dilated cardiomyopathy?

Doxorubicin.

Question: In addition to antibiotics, what is the treatment of choice for cholangitis?

ERCP.

Question: What is the recommended treatment for benign paroxysmal positional vertigo?

Epley maneuver.

Question: What is the most common organism that causes acute pyelonephritis?

Escherichia coli.

True or False: The acellular pertussis vaccination (DTaP) is harmful to neonates and infants and should be avoided in pregnant women.

False: It is recommended that pregnant women receive a DTaP booster

Acute Pericarditis Question: What additional symptoms are often present in tuberculous pericarditis?

Fever, night sweats, and weight loss.

Question: What features comprise Reynold's pentad of cholangitis?

Fever, right upper quadrant pain, jaundice, hypotension and altered mental status.

Question: What test can confirm the diagnosis of macular degeneration and distinguish between atrophic or exudative disease?

Fluorescein angiography.

Question: Which two medication classes increase an individual's risk for Achilles tendon rupture?

Fluoroquinolones and corticosteroids.

How do you calculate the true sodium level in patients with DKA?

For every 100 mg/dL the glucose is above 100 mg/dL, the sodium is lowered by 1.6 mEq/L.

One Step Further Question: What two bones does the base of the fifth metatarsal articulate with?

Fourth metatarsal and cuboid.

Question: What is an alternative location for needle decompression?

Fourth or fifth intercostal space in the anterior axillary line.

Question: What is the prognosis for acute compression radial mononeuropathy?

Good, with most patients recovering fully by six months.

Question: What is the most common cause of hyperthyroidism?

Graves' disease, which involves autoantibodies that stimulate the TSH receptor.

Question: What is the name of the flank bruising due to severe hemorrhagic pancreatitis?

Grey-Turner's sign.

Question: What finding on peripheral blood smear is characteristic of G6PD deficiency?

Heinz bodies, which are clumps of denatured hemoglobin

Question: What syndrome closely resembles TTP but is more common in children?

Hemolytic uremic syndrome (HUS). TTP/HUS HUS: E. coli O157:H7 Autoimmune hemolysis, Renal failure, Thrombocytopenia (ART) Bloody diarrhea Avoid ABX TTP: ART + Fever + Neurologic abnormalities (FAT RN) Normal coagulation studies Plasmapheresis

Question: Besides the calf veins, what other areas of the venous system does proximal duplex ultrasonography have difficulty in visualizing when evaluating for DVT?

Iliac veins, or veins proximal to the common femoral vein, and the femoral vein within the adductor canal.

Question: What patient population at risk for developing fungal corneal ulcerations?

Immunosuppressed individuals.

Impetigo Review

Impetigo S. aureus > S. pyogenes Warm, humid conditions "Honey colored," weeping Face Bullous impetigo: neonates, bullae, S. aureus Complication: PSGN

Question: What is the most common cause of mesenteric venous thrombosis?

Inherited thrombotic disorder (75% of cases).

Question: What is the mechanism of action of colchicine in the management of acute gout?

Inhibits microtubule formation.

Question: What is McRoberts maneuver for shoulder dystocia?

It involves hyperflexion of the knees to place the thighs against the abdomen.

What is a Hampton's hump?

It is a wedge shaped infiltrate seen on chest X-ray indicative of a pulmonary infarct.

Rapid Review Jefferson/Burst Fracture

Jefferson/Burst Fracture Patient will be a football player or diver With a history of trauma via axial loading Most commonly caused by burst fracture of C1 Treatment is a halo Comments: Unstable fracture

Question: What spinous process level corresponds to the pelvic brim?

L4.

Question: What diseases are associated with Streptococcus bovis endocarditis?

Large bowel pathologies including colon cancer, polyps, and diverticula.

Rapid Review Lead Poisoning:

Lead Poisoning: Patient will be a child Complaining of headache, joint pain, and constipation X-ray will show hyperdense lines at metaphyses (lead lines) Labs will show microcytic, hypochromic anemia, and basophilic stippling on peripheral smear Treatment is oral succimer or IV EDTA (Calcium disodium edetate)

What medication is classically implicated in causing nephrogenic diabetes insipidus?

Lithium.

What are the most common side effects of topical antifungal therapies for the treatment of uncomplicated candidial vaginitis?

Local burning and tingling.

Macular Degeneration

Macular Degeneration Patient will be older Complaining of bilateral, gradual central field vision loss PE will show Dry macular degeneration (85% of cases): Atrophic changes and yellow retinal deposits (Drusen spots) Wet macular degeneration: Vascular changes Diagnosis is made by Amsler grid Most common cause of blindness in the elderly

What readily available condiment will kill adult lice within 10 minutes of application?

Mayonnaise

Question: Overmedication with nitroglycerin can cause what toxic effect?

Methemoglobinemia.

Question: What systemic neurologic disorder is associated with optic neuritis?

Multiple sclerosis.

Question: Which cause of peripheral vertigo is due to an increased amount of endolymph within the cochlea and labyrinth?

Ménière's syndrome.

Question: Is aggressive hypercalcemia treatment required in a well hydrated, well appearing patient with a mildly elevated serum calcium level?

No. Follow up should be arranged, but no other interventions are immediately necessary.

What months of the year contain the peak incidence of RSV in North America?

November to March.

One Step Further Question: What injury is likely to be present in a child with a raised elbow fat pad?

One Step Further Question: What injury is likely to be present in a child with a raised elbow fat pad?

Question: How long does PTSD last?

One third of patients have symptoms that resolve within a year, but another one third have symptoms lasting over ten years.

Question: What is the treatment of choice for patients with influenza?

Oseltamivir 75 mg by mouth twice daily for five days.

Question: What is a well-known complication of even well-drained paronychias?

Osteomyelitis of the distal phalanx.

What kind of incontinence do patients with spinal cord compression syndromes develop?

Overflow incontinence.

PDA

Patent Ductus Arteriosus Patient will be complaining of failure to thrive, poor feeding, tachycardia, and tachypnea PE will show continuous, rough, "machinery-like" murmur, heard best in the first interspaces of the LSB Diagnosis is made by echo Treatment is indomethacin

Question: Elevated pleural fluid amylase suggests what possible etiologies?

Pancreatitis, esophageal rupture or certain malignancies.

Question: What are the most common causes of obstructive urinary retention in women?

Pelvic masses and prolapse of the pelvic organs (e.g. bladder, rectum, or uterus).

Bullous Pemphigold

Pemphigus vulgaris (B) is an autoimmune disorder characterized by mucous membrane bullous lesions followed weeks to months later by cutaneous flaccid bullae formation. It affects patients aged 40-60 years and is also treated with wound care and oral steroids. Mortality rates approach 15% even with appropriate treatment. Staphylococcal scalded skin syndrome (C) is primarily a disease affecting young children and infants. It is characterized by tender erythroderma that progresses to extensive bullae formation and subsequent exfoliation. Toxic epidermal necrolysis (D) also has a positive Nikolsky sign on examination. Most cases are secondary to a hypersensitivity reaction to medication which causes separation of the epidermis and dermis resulting in widespread bullous formation and exfoliation. Mucous membrane involvement is prominent.

What is the emergent treatment of pericardial tamponade?

Pericardiocentesis.

Question: What is the definitive treatment for sinus bradycardia?

Permanent pacemaker

Question: What nerve is commonly injured in knee dislocations?

Peroneal nerve.

Question: What is the primary indication for glycoprotein inhibitors in acute coronary syndrome?

Planned percutaneous coronary intervention.

Question: When is an emergency department thoracotomy indicated in a child?

Post-traumatic arrest from penetrating trauma or blunt trauma with acute deterioration but initial signs of life in the emergency department.

Question: What artery is typically involved in strokes causing homonymous hemianopsia?

Posterior cerebral artery

Question: What condition is caused by prolonged or frequent kneeling?

Prepatellar bursitis.

Question: What is Todd's paralysis?

Postictal paralysis that follows a generalized or complex partial seizure and is a focal motor deficit that may persist up to 24 hours.

Question: What is the treatment for isoniazid-induced seizures?

Pyridoxine (vitamin B6).

Question: What black box warning is associated with droperidol?

QT prolongation and torsade de pointes.

Scabies

Sarcoptes scabiei Pruritic rash worse at night Linear burrows Interdigital spaces of hands/feet, penis, breasts Permethrin (first line) Ivermectin **head and back are sparred (head involved in children)

Question: What test should all patients with inferior myocardial infarction undergo to determine whether they have right ventricular involvement

Right-sided ECG.

Erysipelas

Rosacea (B) causes erythema in a malar distribution; however, this would not be associated with a prodrome of fever and chills. Scarlet fever (C) is associated with a "sandpaper" rash with circumoral sparing. Systemic lupus erythematosus (D) can be associated with a malar rash as well; however, it is not typically abrupt in onset, indurated, or associated with fever and chills.

Question: What blood smear abnormality is associated with Waldenstrom macroglobulinemia?

Rouleaux formation.

Hypercalcemia Question: What electrocardiogram finding is classically associated with hypercalcemia?

Short QT interval.

Name 3 risk factors associated with the development of osteomyelitis?

Sickle cell anemia, recent surgery, immunocompromise, diabetes, peripheral vascular disease, hemodialysis, IV drug abuse.

Question: Which two organisms are likely to cause septic arthritis in two or more joints simultaneously?

Staphylococcus aureus and Streptococcus pneumoniae.

Question: What is the most common bacteria associated with a spinal epidural abscess?

Staphylococcus aureus.

Question: What is the most common pathogen that causes erysipelas?

Strep. pyogenes.

Bell Clapper Deformity

Testicular Torsion Bimodal: < 1-year-old, puberty Increased risk: undescended testicle, Bell-Clapper deformity Sudden onset of unilateral pain during sleep or exercise Left > right Absent cremasteric reflex (one study showed normal reflex has 96% negative predictive value) Doppler ultrasound Immediate urologic consultation Manual detorsion (medial to lateral/opening a book) if delays expected

Question: What are two primary physical exam techniques used to diagnose shoulder impingement?

The Neer test (passively flexing the glenohumeral joint while simultaneously preventing shoulder shrugging) and the Hawkins test (internally rotating the shoulder while the elbow is flexed at 90 degrees).

Paraphimosis

The diagnosis of paraphimosis is based on clinical findings and should be intervened on immediately. Paraphimosis can often be reduced by compression of the glans for several minutes to reduce edema and allow for successful reduction of the now smaller glans through the foreskin.

Question: What is the discriminatory zone?

The discriminatory zone is the ßhCG level at which a normally developing pregnancy should be seen. In general transvaginal ultrasound level is 1,000 - 2,000mIU/ml and transabdominal ultrasound level is 6,500mIU/ml.

Question: What counseling should you offer parents of children with electrical burns of the lip?

The labial artery typically bleeds 2-5 days after the injury. You should teach them how to hold pressure on the area should this occur.

Question: Outside of the pelvis, what is the most frequent location of endometriosis?

The thorax.

Question: Which aspects of ventilator mechanics can be used to increase ventilation to help resolve a respiratory acidosis?

Tidal volume and respiratory rate.

Question: Name the condition with attacks of lancinating pain in the distribution of cranial nerve V.

Trigeminal neuralgia or tic douloureux.

Question: What is the treatment for H. pylori infection?

Triple therapy with a proton pump inhibitor, clarithromycin, and amoxicillin or metronidazole.

True or false: Incision and drainage should be avoided in cases of herpetic whitlow.

True, as this can lead to secondary bacterial infection.

Question: What method of suicide is most commonly successful?

Use of a firearm.

Question: What pulmonary function tests help determine need for intubation?

Vital capacity or negative inspiratory flow (NIF).

What are options for outpatient anticoagulation for atrial fibrillation?

Warfarin or new oral anticoagulant drugs (e.g., dabigatran, rivaroxaban, apixaban).

Question: Are patients with panic disorder at higher risk for suicide?

Yes and therefore suicidality screening should be performed.

Question: Is the presence of an IUD a risk factor for development of pelvic inflammatory disease?

Yes, but only in the first few weeks after placement.

Should family members of an infected individual also be treated for scabies?

Yes, family members and sexual contacts.

Question: Can compartment syndrome develop with an open fracture?

Yes.

Rapid Review ß-Blocker Toxicity

ß-Blocker Toxicity PE will show hypotension, bradycardia, and heart block Labs will show hypoglycemia Treatment is glucagon

Rapid Review ß-Blocker Toxicity:

ß-Blocker Toxicity: PE will show hypotension, bradycardia, and heart block Labs will show hypoglycemia Treatment is glucagon

One Step Further Question: What is the management for acid or base ingestions?

Answer: Upper endoscopy.

One Step Further Question: Elevation in which right sided lead indicates right ventricular infarction?

Answer: V4R.

One Step Further Question: What are the two most common conduction abnormalities seen on ECG with TCA overdose?

Answer: Widened QRS and prolonged QTc.

One Step Further Question: In what age group is atlanto-occipital dislocation most common?

Answer: Young children (due to large head size and relative laxity of ligamentous structures).

Rule of 9s Question: What is the Parkland Formula?

(4 x weight in kg) x (%TBSA burned) (excluding first-degree burns). Thermal Burns Fire: obtain CO levels Industrial fire: suspect CN toxicity Superficial: similar to sunburn Superficial partial: red, painful, blisters Deep partial: white, leathery, painless Full-thickness: charred, insensate Rule of 9s Parkland formula: 4 mL/kg × % total BSA burned (pediatrics give 3 mL/kg); 50% given in first 8 hours, remainder over 16 hours Target urine output: 0.5-1 mL/kg/h in adults; 1-2 mL/kg/h in children Consider escharotomy for circumferential and full-thickness burns

Question: What is the commonly accepted urine culture colony count to be considered a urinary tract infection?

100,000 colony forming units/mL.

Question: What management is indicated in patients with severe symptoms not controlled by opioids and benzodiazepines?

Answer: Latrodectus antivenom.

One Step Further Question: What type of Salter-Harris fracture is commonly missed on X-ray?

Answer: Type I.

Correct Answer ( D ) Explanation: Tricuspid valve vegetations are the most likely abnormalities seen on echocardiogram. This patient has endocarditis affecting his tricuspid valve. Risk factors include injection drug use, male sex, age over 65 years old, and underlying structural heart disease. Diagnostic workup includes blood cultures and a transesophageal echocardiogram. A transesophageal echocardiogram has a higher sensitivity to detect vegetations than a transthoracic echocardiogram. Complications include sepsis and embolic events. In this case, the vegetations are on the right side of the heart and have embolized to the lungs causing a multifocal pneumonia (as shown above). Right-sided endocarditis is more common in intravenous drug users. Treatment includes intravenous antibiotic therapy (directed specifically against Staph. aureus) and supportive care.

32-year-old man presents to the Emergency Department with complaints of fever and shortness of breath. On physical exam, he is noted to have coarse breath sounds bilaterally and a systolic murmur. A chest radiograph is obtained and shown above. Which of the following abnormalities is likely to be noted on echocardiogram? Diffuse myocardial hypokinesis Mitral valve vegetations Pericardial effusion Tricuspid valve vegetations

Question: What is the normal rate of a junctional escape pacemaker?

40-60 beats/minute.

Correct Answer ( D ) Explanation: This patient has a spontaneous simple pneumothorax. A pneumothorax is a collection of air in the pleural space. A simple pneumothorax has no communication with the atmosphere and does not show any signs of a tension. Simple pneumothoraces can be any size and may be spontaneous or the result of trauma. A communicating pneumothorax results from a defect in the chest wall (e.g. from a stabbing or gunshot wound). A tension pneumothorax occurs when progressive accumulation of air results in compression and shift of the mediastinal structures. This can lead to rapid onset of cardiovascular and respiratory distress. The incidence of spontaneous pneumothorax is greatest in young, healthy men and the majority are smokers. Patients present with pleuritic chest pain and dyspnea. Examination findings include decreased breath sounds and hyperresonance to percussion. Tracheal deviation, hypotension and JVD would be signs concerning for a tension pneumothorax. Diagnosis is made by chest X-ray. An end expiratory view increases the sensitivity of the chest X-ray in cases of a small pneumothorax. Ultrasound can also be used with absence of normal lung sliding being diagnostic of a pneumothorax. A small spontaneous pneumothorax in a healthy, minimally symptomatic patient can be treated with 100% oxygen, observation, and repeat radiography in 6 hours. If there is no progression on repeat chest X-ray, the patient can be discharged home with close follow up. Spontaneous pneumothoraces have a 20-50% chance of recurrence. A chest tube thoracostomy (C) is indicated in the following cases: large spontaneous pneumothorax, evidence of enlargement during observation, tension pneumothorax, trauma, positive pressure ventilation, and significant underlying lung disease.

A 22-year-old healthy man presents with acute onset pleuritic, left-sided chest pain and mild dyspnea. Vital signs include blood pressure 142/74 mm Hg, heart rate 82 beats/minute, and oxygen saturation 97% on room air. He is in no acute distress. His chest radiograph is shown above. Which of the following is the next best step in management? Discharge home with ibuprofen for pain relief Order a computed tomography scan of the chest Perform a chest tube thoracostomy Place patient on 100% oxygen and repeat chest radiograph in 6 hours

Correct Answer ( B ) Explanation: This patient sustained a Galeazzi fracture, which is a fracture of the middle to distal radial shaft and subluxation or dislocation of the distal radioulnar joint (DRUJ). The mechanism of injury is usually secondary to a fall that causes an axial load on a hyperpronated arm. Injury to the ulnar nerve and anterior interosseous branch of the median nerve can occur. In contrast to skeletally immature patients which are typically treated with closed reduction and casting, adults require immediate orthopedic consultation with open reduction and internal fixation (ORIF). A Colles' fracture (A) is a fracture of the distal radius with dorsal and radial displacement of the wrist and hand, and typically originates from a fall on an outstretched hand (FOOSH). A Hutchinson fracture (C), also known as a Chauffeur's fracture or backfire fracture, is an intra-articular fracture of the radial styloid process. The name originates from early chauffeurs that sustained a direct blow to the wrist from the backfiring of the starting crank of a vehicle. A Monteggia fracture (D) is a fracture of the proximal third of the ulna with a radial head dislocation. Monteggia fracture-dislocations are generally treated with open reduction and internal fixation of the ulna with closed reduction of the radial head dislocation. Monteggia fractures are less common than Galeazzi fractures.

A 34-year-old man presents to the Emergency Department with left arm pain after a fall on an outstretched hand while walking his dog. An X-ray is obtained as seen above. What eponym is associated with this fracture? Colles' fracture Galeazzi fracture Hutchinson fracture Monteggia fracture

Correct Answer ( C ) Explanation: The patient has a felon, which is a pyogenic infection of the digital subcutaneous tissue and pulp. Staphylococcus aureus is the most common causative organism. The finger pads contain septae that confine the infection leading to increased pressure at the distal finger. The infection can cause intense pain, swelling and redness. Minor trauma of the overlying skin is often the primary cause. Incision and drainage is the recommended initial treatment if the finger is swollen and tense with palpable fluctuance. If left untreated, the infection may spread to the flexor tendon sheath leading to flexor tenosynovitis, or to the bone leading to osteomyelitis. A unilateral longitudinal incision is made to spare the sensate portion of the finger pad. This approach usually achieves adequate drainage of infection. Dissection with a hemostat to break up septations may be required to sufficiently access the wound contents. Wounds should be thoroughly irrigated then dressed with a sterile, dry dressing. The wound should be evaluated 48 hours later. Oral antibiotics covering Staphylococcus aureus, Streptococcus pyogenes, and anaerobes (e.g., trimethoprim-sulfamethoxazole) are recommended as most felons are accompanied by cellulitis. Trephination (D) is a treatment for subungual hematomas associated with nailbed injuries but has no utility in managing infections of the hand or digits.

A 39-year-old man presents to the Emergency Department with left hand pain. He denies recent trauma, genital or oral lesions, or fever. His medical history is significant for diabetes mellitus and tobacco abuse. His examination is shown above. Which of the following is the most appropriate initial therapy for this patient?

Correct Answer ( B ) Explanation: This patient is exhibiting signs and symptoms consistent with thyroid storm. Thyroid storm is a rare, life-threatening hypermetabolic state caused by severe thyrotoxicosis. The most common precipitating factor is infection. A well-recognized and important complication of thyroid storm is high output heart failure. This patient is exhibiting signs of thyroid storm, as well as high output heart failure. Signs and symptoms of thyroid storm include anxiety, tremulousness, psychosis, obtundation, seizures, coma, fever, tachycardia (out of proportion to fever), circulatory collapse, diarrhea, and vomiting. Thyroid storm is a clinical diagnosis. Management of thyroid storm includes supportive therapy with airway protection, intravenous fluids, cardiac monitoring, and supplemental oxygen. Fever may be treated with acetaminophen as salicylates have the potential to increase T4 and T3 levels. Adrenergic blockade is the most important initial component of therapy. Propranolol, a nonspecific b-blocker, is used to decrease sympathetic hyperactivity and partially block peripheral conversion of T4 to T3. Antithyroid drugs should then be initiated. Propylthiouracil and methimazole are both options as both block the synthesis of thyroid hormone; however, propylthiouracil has the added benefit of decreasing conversion of T4 to T3 and works faster than methimazole. One hour after antithyroid drug administration, iodide should be given to inhibit the release of stored thyroid hormone. Steroids also serve as an adjunct to decrease peripheral conversion of T4 to T3. Cardiac tamponade (A) is more classically associated with rheumatologic diseases such as rheumatoid arthritis or systemic lupus erythematosus and presents with obstructive shock due to right ventricular collapse. Pulmonary embolism (C) also presents with acute obstructive right heart failure as opposed to high output heart failure. Tension pneumothorax (D) also presents with an obstructive shock picture as venous return to the right heart is severely impaired.

A 42-year-old man presents to the ED with racing heart rate and difficulty breathing. On physical exam, you note bilateral ocular proptosis and 3+ non-pitting edema to the bilateral lower extremities. His electrocardiogram is shown above. Which of the following complications of this disease is this patient most likely experiencing? Cardiac tamponade High output heart failure Pulmonary embolism Tension pneumothorax

Correct Answer ( D ) Explanation: Dyspepsia is a common complaint. There is no definitive association with age, sex, socioeconomic status, smoking, or alcohol abuse. Peptic ulcer disease is marked by ulcerations of the gastric mucosa, often due to nonsteroidal anti-inflammatory drug use or H. pylori infection. When the ulcer erodes through the entire bowel, the result is a perforated gastric ulcer. The patient often experiences abrupt onset of severe epigastric pain as gastrointestinal contents leak into the peritoneal cavity. Plain films of the chest or abdomen may demonstrate free air under the diaphragm, although this is not a universal finding. They initially develop a chemical and ultimately a bacterial peritonitis, and may become septic if left untreated. This is a surgical emergency. Emergent consultation with a general surgeon combined with broad spectrum antibiotic administration and aggressive resuscitation is merited

A 45-year-old woman, who works as a day laborer, presents with epigastric abdominal pain that began one hour prior to arrival. She has been having abdominal discomfort after meals for the past several weeks. Today, she developed sudden onset, severe abdominal pain far worse than what she has been experiencing. She denies back pain, nausea, and vomiting. On exam, she is tachycardic, but has otherwise normal vital signs. Her abdomen is significantly tender in the epigastric region with rebound and guarding. Chest X-ray is as above. What is the most likely diagnosis? Cholecystitis Gallstone pancreatitis Gastritis Perforated gastric ulcer

Correct Answer ( D ) Explanation: Sinus bradycardia is a rhythm that originates from the sinoatrial (SA) node with a rate of < 60 beats per minute. The SA node is the heart's normal pacemaker, and sinus bradycardia may be normal in some patients such as conditioned athletes. In these cases, patients will have a resting heart rate of < 60 per minute, but will not exhibit any other symptoms. Other times, sinus bradycardia may be due to organic heart disease such as coronary artery disease, cardiomyopathy, or myocarditis, resulting in symptomatic sinus bradycardia. Therefore, treatment largely depends on whether the patient is symptomatic (unstable) or not. An unstable patient is anyone who shows signs of poor perfusion, such as altered mental status, diaphoresis, dizziness, or syncope. A stable patient may be monitored closely. Initial pharmacologic treatment for an unstable patient is atropine. Since this patient presents with both a syncopal episode and diaphoresis, she is unstable and atropine is the appropriate therapy. After an initial dose of atropine, infusions of epinephrine or dopamine may also be considered, and for more emergent cases transcutaneous pacing may be necessary. Adenosine (A) is the treatment for supraventricular tachycardia, not bradycardia, and so is not appropriate for this patient. Amiodarone (B) is used to treat dysrhythmias, such as ventricular tachycardia, not a sinus rhythm. Amlodipine (C) is a calcium channel blocker that helps to control heart rate and blood pressure

A 50-year-old woman presents to the Emergency Department following a syncopal episode. On physical exam she is diaphoretic. She is alert and orientated. Her blood pressure is 95/50 mm Hg and respirations are 12 per minute. Her rhythm strip is shown above. What is the most appropriate pharmacologic treatment for this patient's condition? Adenosine Amiodarone Amlodipine Atropine

Correct Answer ( B ) Explanation: The patient has atrial fibrillation with rapid ventricular response. Atrial fibrillation results when multiple areas of the atrial myocardium simultaneously depolarize and contract. As a result, the atrial walls do not contract in a coordinated fashion, but rather "quiver" or "fibrillate." The electrocardiographic hallmarks of atrial fibrillation are a narrow, complex, irregularly irregular rhythm and no discernible P waves. The rate may be slow, normal, or fast. Atrial fibrillation is usually associated with ischemic or valvular heart disease, and can also be seen in cardiomyopathies, myocarditis, and thyrotoxicosis. Patients with atrial fibrillation may be asymptomatic, or may complain of palpitations, dizziness, lightheadedness, chest pain, or dyspnea. Patients in whom atrial fibrillation results in a rapid ventricular rate may develop clinical heart failure due to inadequate ventricular filling and decreased cardiac output, manifested by pitting edema and pulmonary edema. The management of atrial fibrillation includes ventricular rate control, consideration of rhythm conversion, and anticoagulation for stroke prevention. Patients who are stable can be treated with rate control with beta blockers or calcium channel blockers (e.g., diltiazem), and consideration of rhythm control, either with pharmacologic agents or via synchronized cardioversion. Patients with atrial fibrillation are at increased risk of having a mural thrombus, which could dislodge and travel to the cerebral circulation and cause a stroke. Therefore, patients who have been in atrial fibrillation for more than 48 hours should undergo a period of anticoagulation prior to rhythm conversion. Finally, patients with unstable vital signs or other signs of hypoperfusion (e.g., ischemic chest pain, hypotension, or confusion) should undergo emergent synchronized cardioversion Administer intravenous adenosine (A) is the treatment of choice for SVT, which is a narrow, complex, regular tachycardia

A 52-year-old man with a history of diabetes mellitus and hypertension presents to the ED with palpitations and dyspnea that have been present intermittently over the past week. His vital signs include HR 135 beats/minute, blood pressure 136/87 mm Hg, RR 15 breaths/minute, and oxygen saturation of 97% on room air. His electrocardiogram is shown above. Which of the following is the most appropriate initial management? Administer intravenous adenosine Administer intravenous diltiazem Sedate the patient and perform synchronized cardioversion with 100 joules Transport the patient to the cardiac catheterization suite

Correct Answer ( D ) Explanation: The patient has first-degree atrioventricular (AV) block evidenced by a prolonged PR interval but no other abnormalities. AV blocks refer to slowed or blocked conduction through the AV node. In a first-degree AV block, each atrial impulse is conducted to the ventricles via the AV node, just more slowly than normal. This results in a prolonged PR interval of > 200 milliseconds. There is a P wave preceding each QRS complex, and the PR interval is constant. The AV node is usually the site of the slowed conduction. In most cases, a first-degree AV block does not have any clinical significance. It can be a normal finding in a non-diseased heart or due to increased vagal tone, the effect of medications, or disease processes like myocarditis and cardiac ischemia. If a first-degree AV block is seen in association with myocardial infarction, it can indicate an increased risk of progression to complete heart block. These patients should be monitored closely on telemetry to make sure they do not develop a complete heart block. Since this patient is asymptomatic, it is appropriate to reassure the patient and the orthopedic surgeon that no treatment is necessary.

A 61-year-old woman presents with a trimalleolar ankle fracture after a fall. As part of her preoperative workup, an electrocardiogram is obtained. A portion of the rhythm strip is shown above. Which of the following is the most appropriate? Administer intravenous calcium chloride, insulin, glucose, and sodium bicarbonate Order serial cardiac biomarkers and admit to cardiology Place transcutaneous pacer pads and make arrangements for a transvenous pacemaker Reassure the patient and the orthopedic surgeon that no treatment is necessary

Correct Answer ( C ) Explanation: Acute coronary syndrome occurs when there is imbalance between myocardial oxygen demand and delivery. This most commonly occurs when an atherosclerotic plaque in a coronary artery is disrupted, resulting in platelet aggregation and thrombus formation, which occludes the affected vessel, blocking blood flow. Acute coronary syndrome can also occur from coronary vasospasm, or as a result of forces extrinsic to the coronary arteries, such as decreased blood flow (hypotension), increased myocardial oxygen demand (fever, tachycardia, thyrotoxicosis), or decreased oxygen delivery (anemia, hypoxemia). The electrocardiogram (ECG) is the most helpful initial test for acute coronary syndrome. ST-elevation in consecutive leads is used to diagnose ST elevation myocardial ischemia, and the pattern of elevation can predict which vessel is involved. In this case, the patient has ST elevation in leads II, III, and aVF and reciprocal ST depression in lead I and aVL. This is diagnostic of an inferior wall myocardial infarction. The coronary artery that supplies the inferior portion of the heart in most patients in the right coronary artery. This patient should be treated with aspirin and sent for an emergent cardiac catheterization. If cardiac catheterization is not available, thrombolysis with tissue plasminogen activator (tPA) is another appropriate treatment option.

A 62-year-old woman was walking her dog when she began to experience intense substernal chest pressure. She became very diaphoretic and felt nauseated. She called 911 and was brought to the ED. Her electrocardiogram is shown above. Which of the following best describes the location of the patient's myocardial infarction? Anterolateral Anteroseptal Inferior Posterior

Correct Answer ( C ) Explanation: This patient is exhibiting signs and symptoms consistent with myxedema coma. Myxedema coma is defined as severe hypothyroidism resulting in a decompensated metabolic state and mental status change. It is classically characterized by thickened, non-pitting edema of the skin diffusely associated with severe chronic hypothyroidism. Precipitating factors of myxedema coma include infection, cold exposure, stroke, or medications such as amiodarone or lithium. Mortality in myxedema coma is proportional to the severity of the patient's hypothermia. Other signs and symptoms of myxedema coma include hypoventilation, bradycardia, poor cardiac contractility, decreased cardiac output, pericardial effusion, and altered mental status. The diagnosis of myxedema coma is a clinical diagnosis based upon history and physical examination findings and exclusion of other causes of coma. As with other causes of hypothyroidism, the TSH is typically high while T4 and T3 are low. Other laboratory findings include hypoglycemia, hyponatremia, hypoxemia, and hypercapnia. ECG findings often include sinus bradycardia, prolonged QT interval, low voltage, and flattened T waves. If a pericardial effusion is present, the chest radiograph may show an enlarged cardiac silhouette. Definitive therapy for patients with myxedema coma is with the administration of intravenous thyroxine. Digoxin toxicity (A) typically presents with bradycardia. The classic ECG findings associated with digoxin toxicity are bidirectional ventricular tachycardia or slow atrial fibrillation

A 74-year-old woman with a history of hypothyroidism presents to the ED with confusion, rigors, and lightheadedness. Vital signs are BP 110/64 mm Hg, HR 48 beats per minute, RR 12 breaths per minute, and T 95.2°F. Her electrocardiogram is shown above. Which of the following is the most likely diagnosis? Digoxin toxicity Hyperosmolar nonketotic coma Myxedema coma Nephrogenic diabetes insipidus

Correct Answer ( A ) Explanation: This patient is exhibiting signs and symptoms of central retinal artery occlusion. Central retinal artery occlusion is defined by the occlusion of blood flow to the retina, typically by emboli from the carotid artery or from the heart in patients with atrial fibrillation. Risk factors for central retinal artery occlusion include hyperlipidemia, coronary artery disease, atherosclerosis, hypertension, diabetes, and glaucoma. Signs and symptoms include sudden, painless, complete loss of monocular vision. Physical exam will reveal an afferent pupillary defect or Marcus Gunn pupil which is a pupil that will have a normal consensual light response followed by a paradoxical dilatation with direct light exposure. Diagnosis is on funduscopic examination as you will see a boxcar appearance of the retinal vessels due to interruption of the blood flow and a pale retina with a cherry red spot on the fovea. Management of central retinal artery occlusion involves a multi-pronged approach. Ophthalmology should be consulted immediately upon suspicion of the diagnosis. Orbital massage may be used to attempt to dislodge the clot. Anterior chamber paracentesis may also be attempted. Pharmacologic agents may be used such as acetazolamide, mannitol, or timolol in an attempt to decrease intraocular pressure. Central retinal vein occlusion (B) will have the appearance of "blood and thunder" on funduscopic exam with retinal hemorrhages and cotton wool spots. Idiopathic intracranial hypertension (C) will have the appearance of papilledema on funduscopic exam and is often bilateral. These patients will typically have headaches and incomplete vision loss. Optic neuritis (D) may or may not have evidence of optic disc hyperemia and inflammation on funduscopic exam and is typically painful.

A 77-year-old woman presents to the ED with acute painless vision loss. On funduscopic exam, you note the above. Which of the following is the most likely diagnosis? Central retinal artery occlusion Central retinal vein occlusion Idiopathic intracranial hypertension Optic neuritis

Correct Answer ( A ) Explanation: The patient has multifocal atrial tachycardia (MAT). MAT occurs when at least three different atrial ectopic foci depolarize to pace the heart. These are seen on an ECG as an irregular rhythm with P waves with at least three distinct morphologies. The QRS complex is narrow, unless an underlying bundle branch block is present. Because of the irregular rhythm, MAT is easily confused with atrial fibrillation. However in MAT, distinct P waves should be visible. MAT is most commonly seen in elderly patients with chronic lung disease. MAT can also be seen in heart failure and sepsis. The treatment of MAT is directed at treating the underlying disorder. In a patient with symptomatic lung disease, oxygen and bronchodilators improve pulmonary function and oxygenation and therefore decrease atrial ectopy. Cardioversion is ineffective for MAT.

A 79-year-old man comes to the ED complaining of dyspnea. His rhythm strip is shown above. What is the most likely underlying process? Chronic obstructive pulmonary disease Crohn's disease Pericardial effusion Ventricular aneurysm

Question: What is the diagnostic criteria for chronic bronchitis?

A productive cough for at least 3 months in a year for 2 consecutive years.

Question: What additional test should be considered for African Americans with a hyphema?

A sicke cell prep.

One Step Further Question: Where is the most common fracture through the scaphoid bone?

Answer: Most scaphoid fractures occur at the waist or middle ⅓ of the bone.

Question: What environmental disorder is acetazolamide used to treat?

Acute mountain sickness.

Osteomyelitis Review

Adults: contiguous spread Children: hematogenous spread Most common: S. aureus Sickle cell: Salmonella Cat/dog bites: Pasteurella multocida Plain films: periosteal elevation or bony erosions Bone scan, MRI Long term ABX

One Step Further Question: If the patient presents early (within 1 hour of ingestion) after a toxic ingestion, what can be administered to decrease the amount of drug absorbed?

Answer: Activated charcoal.

One Step Further Question: What is the treatment of choice for organophosphate poisoning?

Answer: Atropine.

One Step Further Question: What medications do you use to treat hypertension caused by cocaine toxicity?

Answer: Benzodiazepines and phentolamine.

One Step Further Question: What is the normal pressure of a tissue compartment?

Answer: Between 0-10 mm Hg.

One Step Further Question: What type of spider bite causes a hemorrhagic blister, necrotic ulcer, or target lesion?

Answer: Brown recluse spider.

One Step Further Question: Which incomplete spinal cord syndrome has the best prognosis of full recovery?

Answer: Brown-Séquard syndrome.

One Step Further Question: What syndrome is most commonly seen in Asian men and associated with sudden cardiac death secondary to ventricular fibrillation?

Answer: Brugada syndrome.

One Step Further Question: What is the skin finding in methemoglobinemia?

Answer: Cyanotic skin.

One Step Further Question: What type of venom does a brown recluse spider have?

Answer: Cytotoxic.

One Step Further Question: What is the treatment for iron poisoning?

Answer: Deferoxamine.

One Step Further Question: What antibiotics can be used for prophylaxis in penicillin allergic patients?

Answer: Doxycycline, moxifloxacin, or clindamycin plus ciprofloxacin.

One Step Further Question: What intra-abdominal injuries are most commonly seen with Chance fractures?

Answer: Duodenal hematoma or perforation and pancreatic injury.

One Step Further Question: What, in addition to high-dose insulin, is the classic antidote to beta blocker toxicity?

Answer: Glucagon.

One Step Further Question: At what lead level do you start chelation therapy?

Answer: Greater than or equal to 45.

One Step Further Question: What are the most common side effects of acetazolamide?

Answer: Paresthesias and polyuria.

One Step Further Question: What medications are used for treatment of mild intermittent asthma?

Answer: Short-acting bronchodilators such as albuterol and levalbuterol.

One Step Further Question: Where is the most common location of linear skull fractures?

Answer: Temporoparietal skull.

One Step Further Question: Which major neuronal tracts are damaged in anterior cord syndrome?

Answer: The corticospinal and spinothalamic tracts.

One Step Further Question: The terminal branches of which nerve traverse the mental foramen?

Answer: The inferior alveolar nerve.

One Step Further Question: What is Shenton's line?

Answer: The normal smooth, curved line defined by the obturator foramen and the femoral epiphysis.

Rapid Review Anterior Cord Syndrome

Anterior Cord Syndrome PE will show loss of motor, pain, and temperature below injury Most commonly caused by flexion injury Comments: proprioception and vibration intact

What age is testicular torsion most likely to occur?

Bimodal distribution with peak incidence in the neonate within first few days of life and in preadolescence.

Cauda Equina

Cauda Equina Syndrome Causes: herniated disk (most common), tumor, abscess Urinary retention + back pain Saddle anesthesia Flaccid anal sphincter MRI Rx: emergent spine consultation

Question: Which antipsychotic is known to cause agranulocytosis?

Clozapine.

Compartment Syndrome Overview

Compartment syndrome infrequently involves the compartments of the thigh, but it may occur with major trauma. Symptoms of compartment syndrome of the anterior compartment of the thigh (B) includes pain with passive flexion of the knee and paresthesias of the anterior, medial, and lateral thigh. The lateral compartment of the leg (C) contains the peroneus longus, the peroneus brevis, and the superficial peroneal nerve. The volar compartments of the forearm (D) are the most likely compartment of the forearm to develop compartment syndrome. Compartment syndrome of the forearm is most frequently associated with supracondylar fractures in children and radial head fractures in adults.

A 28-year-old woman is brought to the clinic by her sister, who is concerned about her recent reckless behavior. The patient has been spending money excessively, acting sexually promiscuous, speaking delusionally, and has not slept for three days. The patient is visibly irritated by her sister's comments. The patient reports periods of depression in the past, which were untreated, but now feels "absolutely great". She is extremely talkative, easily distracted in her thought process, and reports several different grand new ideas, which she explains within the space of a ten minute discussion. What is the most likely diagnosis? Acute stress disorder Bipolar disorderCorrect Borderline personality disorder Dysthymic disorder

Correct Answer ( B ) Explanation: Bipolar I disorder, also commonly known as manic depression, involves the patient having one or more manic episodes. Often depressive episodes cycle with the mania, although this is not required for diagnosis. Manic episodes are characterised by acute mood escalation; the patient's mood may be persistently euphoric or irritable. Typical manic episodes may involve lack of sleep, including the patient perceiving no need for sleep. The patient may become disinhibited, flamboyant, and excessively talkative. Racing thoughts and flight of ideas are common. Judgment is often impaired, especially with financial decision-making and hypersexuality. Psychotic symptoms may occur, including paranoia, delusions or hallucinations. Bipolar disorder has a 1% lifetime prevalence, with average diagnosis at age 30 years. Misdiagnosis is common, often mistaken for depression during depressive episodes. Mood stabilizers are the mainstay of treatment, with lithium or valproic acid commonly prescribed. Both of these drugs need regular monitoring due to a narrow therapeutic window and risk of toxicity. Antipsychotic medications are often used as adjuncts. Haloperidol or benzodiazepines are used for acute treatment of mania with agitation. Antidepressant medication may precipitate the onset of mania. A diagnosis of bipolar II disorder is given when the patient experiences one or more depressive episodes and at least one hypomanic episode. Hypomania differs from mania with less severe symptoms, including lack of any psychotic symptoms or agitation. Borderline personality disorder (C) is characterized by unstable mood, with frequent mood swings, often several within the space of a few hours. Usually associated with low self-esteem, the patient often appears in a state of crisis. Dysthymic disorder (D) is a chronic disorder with persistent mild depression and no manic or hypomanic symptoms.

What is the most common site of compartment syndrome? Anterior compartment of the leg Anterior compartment of the thigh Lateral compartment of the leg Volar compartment of the forearm

Correct Answer ( A ) Explanation: Compartment syndrome is an acute, limb-threatening emergency. Most frequently associated with fractures, compartment syndrome occurs when increased pressure within a non-expandable compartment results in compromised venous blood flow, worsening congestion, and subsequent arterial compromise. This leads to ischemic necrosis of the muscles and nerves within the compartment. The most frequently involved compartment is the anterior compartment of the leg occurring in association with a tibia fracture. Clinically, patients present with pain out of proportion to the injury, pain with passive stretching of the muscles in the compartment, and paresthesias. Involvement of the anterior compartment of the leg can present with decreased sensation between the first and second toes and weakness with foot dorsiflexion. Pallor and loss of pulses are late and ominous findings. While compartment syndrome is diagnosed clinically, measurement of the pressure within the compartment can provide confirmation of the diagnosis. A delta pressure (diastolic blood pressure - compartment pressure) of < 30 indicates the need for a fasciotomy.

A 41-year-old man with a long history of alcohol abuse presents to the ED with several weeks of worsening shortness of breath and lower extremity edema. An echocardiogram shows biventricular chamber enlargement, increased systolic and diastolic volumes, and an ejection fraction of 35 percent. What is the most likely diagnosis? ADilated cardiomyopathy BHypertrophic cardiomyopathy CPericardial effusion DPulmonary embolism

Correct Answer ( A ) Explanation: Dilated cardiomyopathy is the most common cardiomyopathy. Most cases of dilated cardiomyopathy are idiopathic, but they can also occur in a familial pattern and can result from alcohol abuse or due to medications, such as chemotherapeutic agents. Infections that attack the cardiac tissue (coxsackie virus, Lyme disease) can also result in dilated cardiomyopathy. Postpartum cardiomyopathy usually manifests as a dilated cardiomyopathy. In dilated cardiomyopathy, there is systolic and diastolic dysfunction, resulting in decreased left ventricular contractile force, low cardiac output, and increased end-systolic and end-diastolic ventricular volumes. This leads to dilation of the left ventricle, and often the right ventricle. Patients with dilated cardiomyopathy present with typical findings of heart failure, including dyspnea on exertion, orthopnea, paroxysmal nocturnal dyspnea, pulmonary edema, and dependent edema. EG often shows LV hypertrophy and left atrial enlargement, as well as poor R wave progression. Atrial fibrillation and ventricular ectopy are also common. Chest radiographs may show an enlarged cardiac silhouette, but an echocardiogram is needed to confirm the diagnosis, showing decreased ejection fraction, increased systolic and diastolic volumes, and chamber enlargement. Patients are treated with standard heart failure therapy, and those with a low ejection fractions may be implanted with a cardioverter-defibrillator. In severe cases, a left ventricular assist device may be used as a bridge to heart transplantation.

A 32-year-old G0P0 woman presents with dull, crampy pelvic pain that has been intermittent over the last six months. She is currently menstruating and notes the pain seems to worsen with menses. She also reports pain with intercourse, but denies any vaginal discharge. On examination, she is no acute distress and is afebrile. She has scant blood in the vaginal vault and no significant focal tenderness. No masses are appreciated. Her pregnancy test was negative. What is the most likely diagnosis? AEndometriosisCorrect Answer BMittelschmerz CPelvic inflammatory disease DRuptured ovarian cyst

Correct Answer ( A ) Explanation: Endometriosis is a benign, estrogen-dependent condition that results in endometrial tissue developing in extrauterine sites. The most common site for endometrial implantation is the pelvis, with the ovaries, posterior cul-de-sac, and anterior cul-de-sac affected most frequently. Endometriosis is a disease of women of reproductive age and is rare in postmenopausal women unless they are on estrogen replacement therapy. Dysmenorrhea, pelvic pain, and dyspareunia are the most frequent presenting complaints, but patients may have bowel or bladder symptoms as well. Infertility is found in a quarter of patients. Physical examination is often normal, but depending on the location and size of the implants patients may have localized tenderness on pelvic examination. Pelvic ultrasound is routinely ordered as a first-line test in women with pelvic pain. Although it is unlikely to provide a diagnosis of endometriosis, it can rule out other causes of the patient's symptoms. Definitive diagnosis is made via laparoscopy. Treatment of endometriosis consists of pain management with nonsteroidal anti-inflammatory medications, interruption or cessation of the menstrual cycle via oral contraceptives, and gynecology referral. Surgical management with laparoscopic removal of the implants is another option, especially in those patients wishing to get pregnant.

A 59-year-old woman presents with pain and swelling of the face that began yesterday. She states that the symptoms began abruptly with a fever and chills. Physical exam reveals bright red, indurated skin in a malar distribution. The skin has a peau d'orange appearance. What is the most likely diagnosis? Erysipelas Rosacea Scarlet fever Systemic lupus erythematosus

Correct Answer ( A ) Explanation: Erysipelas is an infection of the skin that is characterized by an abrupt onset of fever, chills, and malaise followed by the development of a bright red, well-demarcated, indurated area of the skin. The skin can develop a peau d'orange appearance and classically involves the face in a malar distribution. The diagnosis is clinical. Treatment is elevation and antibiotics. In patients that develop bullae, crepitus, or pain out of proportion to the exam, a necrotizing infection must be considered and a surgical consult may be warranted.

32-year-old Nigerian man presents to the Emergency Department for blurry vision. He immigrated to the United States six months ago. He was diagnosed with a "lung disease" and was prescribed several medications in Nigeria. Which of the following agents is most likely responsible for his symptoms? Ethambutol Isoniazid Pyrazinamide Rifampin

Correct Answer ( A ) Explanation: Ethambutol is an antitubercular medication whose side effect profile includes optic neuritis and red-green color blindness. It is a component of the first-line "RIPE" therapy: rifampin, isoniazid, pyrazinamide and ethambutol. The four-drug therapy is for active tuberculosis, and initial treatment lasts for 4 weeks. This is followed by a two-drug regimen (typically isoniazid and rifampin) for an additional 18-31 weeks. Isoniazid (B) classically causes hepatotoxicity or hepatitis. Patients with HIV, alcohol abuse, underlying liver disease or pregnancy may be at increased risk. Pyrazinamide (C) may cause hepatitis, arthralgias, or hyperuricemia. Rifampin (D) can cause thrombocytopenia, hepatitis, GI disturbances, or body fluid discoloration (classically orange).

A 61-year-old previously healthy man presents with 3 days of gross hematuria. He denies any dysuria, flank pain, fever, or trauma. On examination, his heart rate is 78 beats per minute, blood pressure 142/72 mm Hg, and temperature 37.4oC. He has no abdominal tenderness or fullness on palpation. Genitourinary exam is unremarkable. Urinalysis is negative for leukocyte esterase and nitrites. There are > 100 red blood cells/hpf and 6 white blood cells/hpf. Creatinine is 1.1. What is the next step in management of this patient? Computed tomography of the abdomen and pelvis with intravenous contrast Continuous bladder irrigation Discharge home with ciprofloxacin for 5 days Discharge home with urology referral

Correct Answer ( A ) Explanation: Hematuria is a frequently encountered condition in the emergency department, whether a patient presenting with gross hematuria or microscopic hematuria found incidentally on urinalysis. Bleeding anywhere along the upper or lower genitourinary tract can produce hematuria, with the most common causes being urinary tract infection, trauma, and renal calculi. Unlike microscopic hematuria, which is often transient and not indicative of serious underlying pathology, gross hematuria (especially in older patients) is often a presenting symptom of malignancy. Risk factors that should prompt a thorough evaluation include age > 40 years, history of cigarette smoking, persistent hematuria, abdominal pain, and trauma. Workup should include a full history and physical, urinalysis, and assessment of renal function. If findings do not point to a specific cause of hematuria, a CT scan with IV contrast or renal ultrasound should be ordered to rule out underlying structural disease or mass

What is the most common symptom of hemorrhoids? Bleeding with defecation Mucoid discharge Painful defecation Pruritis

Correct Answer ( A ) Explanation: Hemorrhoids occur with engorgement of the anal vascular cushions and deterioration of the supportive tissue around these cushions. Bleeding with defecation is the most common symptom associated with hemorrhoids. Most hemorrhoids are painless unless they become thrombosed. Symptoms get exacerbated by frequent bowel movements, constipation, prolonged sitting, lifting, and straining.

A 38-year-old man presents to the emergency department with low back pain that began this morning when he rolled over in bed. Throughout the day, he tried ibuprofen and acetaminophen without relief, which prompted him to come to the Emergency Department for further evaluation. Which of the following historical findings is an indication for further investigation? Fever Intentional weight loss with diet and exercise Palpable paraspinal muscle spasm Prolonged use of oxycodone

Correct Answer ( A ) Explanation: In the United States, low back pain is a common reason for seeking out emergency department care. While there are many potentially serious causes of low back pain, the majority of cases are due to benign, such as musculoskeletal spasm, and can be treated with acetaminophen and nonsteroidal anti-inflammatory drugs (NSAIDs). The "red flags" of back pain are historical features and physical exam findings that should prompt further investigation. Among the red flags is a fever, which could indicate an infection. Investigation for infection should include spinal imaging, urinalysis, complete blood count, C-reactive protein, and erythrocyte sedimentation rate.

Which of the following conditions has a negative Nikolsky sign? Bullous pemphigoid Pemphigus vulgaris Staphylococcal scalded skin syndrome Toxic epidermal necrolysis

Correct Answer ( A ) Explanation: Nikolsky sign is a clinical finding defined as the slipping of the epidermis away from the underlying dermis when gentle lateral pressure is applied. It results in extension of an existing bullae or new bullae formation on previously intact skin. It is found in certain blistering skin conditions such as staphylococcal scalded skin syndrome, Stevens-Johnson syndrome, toxic epidermal necrolysis, and pemphigus vulgaris. Nikolsky sign is absent in bullous pemphigoid. Affecting patients between 60-80 years of age, bullous pemphigoid is a chronic, relapsing autoimmune disorder that involves autoantibody-mediated damage to the epithelial basement membrane zone. Patients present with a prodrome of a pruritic erythematous urticarial or papular rash followed weeks later by the development of tense bullae atop normal or erythematous skin. The bullae can rupture leaving moist, erythematous erosions and crusting. The lesions can be numerous and widespread, but do not typically affect the mucous membranes. Treatment includes topical steroid ointment for localized disease. Oral prednisone is indicated in widespread disease.

A 22-year-old woman presents with lower abdominal pain that started two days ago. Today, she had three episodes of vomiting, but denies any diarrhea or dysuria. On examination, her temperature is 38.8oC, heart rate 105 beats per minute, and blood pressure 128/72 mm Hg. She is ill-appearing with bilateral lower quadrant tenderness. Pelvic examination reveals moderate yellow discharge and uterine tenderness. There are no adnexal masses appreciated. Pregnancy test is negative. What is the next step in management of this patient? Admission for intravenous cefotetan and doxycycline Admission for intravenous ciprofloxacin and metronidazole Discharge home following intramuscular ceftriaxone and with a 14 day course of oral doxycycline Order a computed tomography scan of the abdomen and pelvis with intravenous and oral contrast

Correct Answer ( A ) Explanation: Pelvic inflammatory disease comprises a spectrum of disorders involving the female upper genital tract including endometritis, salpingitis, and tubo-ovarian abscess. While N. gonorrhoeae and C. trachomatis are implicated in many cases, it is most frequently polymicrobial in etiology involving other organisms such as G. vaginalis, anaerobes, E. Coli and aerobic streptococci as well. The CDC recommends empiric treatment of PID in young, sexually active women with pelvic or lower abdominal pain who experience cervical motion tenderness, adnexal tenderness, or uterine tenderness that cannot be ascribed to another cause. Other criteria that support the diagnosis of PID include fever, abnormal purulent vaginal discharge, presence of white blood cells on wet mount of vaginal discharge, elevated erythrocyte sedimentation rate or C-reactive protein, or documented infection with N. gonorrhoeae or C. trachomatis. Patients with mild-to-moderate disease can be treated as an outpatient with ceftriaxone 250 mg IM and doxycycline 100 mg BID for 14 days. This patient is ill-appearing with vomiting and a high fever making her a candidate for inpatient management with intravenous cefotetan and doxycycline

A 61-year-old man with a history of chronic obstructive pulmonary disease presents with altered mental status. His oxygen saturation is 90% on 4 liters of oxygen via nasal cannula. A venous blood gas is obtained with the following results: pH: 7.15 pO2: 45 mm Hg pCO2: 100 mm Hg HCO3: 34 mEq/L The patient has which of the following acid-base disturbances? Acute on chronic respiratory acidosis Acute respiratory acidosis Metabolic acidosis Metabolic alkalosis with respiratory compensation

Correct Answer ( A ) Explanation: Serum pH is dependant primarily on the ventilatory function of the lungs for volatile acids and the kidneys for fixed acids. The pulmonary system can deal with volatile acids fairly rapidly, while the renal system requires hours to fully equilibrate. As a result, the respiratory system can compensate well for metabolic acid-base changes, but the renal system cannot rapidly adapt to fluctuating levels of volatile acids. A patient who chronically retains CO2, such as a patient with COPD, will often have chronic renal compensation, but a patient who has an acute respiratory decompensation will be unable to rapidly renally compensate. Additionally, there can never be full compensation of the primary derangement. In this case, the pH is acidemic, so this is an acidosis. The pCO2 is elevated indicating a respiratory acidosis, and the HCO3 is elevated indicating a metabolic alkalosis. Since metabolic compensation can only occur chronically and the patient has a derangement beyond the expected compensation, this is an acute on chronic respiratory acidosis.

45-year-old woman presents to the emergency department with palpitations. She is placed on the monitor which shows a rapid narrow-complex tachycardia. Her heart rate is 160 bpm and blood pressure is 120/70 mm Hg. Vagal maneuvers are attempted and fail. Next, adenosine 6 mg IV is given intravenously without change in her rhythm. Repeat vitals show a heart rate 165 bpm and blood pressure 120/70 mm Hg. What is the most appropriate next step in management? Adenosine 12 mg intravenous Adenosine 6 mg intravenous Synchronized cardioversion Vagal maneuvers

Correct Answer ( A ) Explanation: Supraventricular tachycardia (SVT) or paroxysmal supraventricular tachycardia (PSVT) is most commonly caused by a re-entrant pathway in the atrioventricular node. Clinically, patients experience an abrupt onset of tachycardia with a ventricular rate of 120 to 200 beats per minute. Rates greater than 225 beats per minute suggest an accessory pathway. Regardless, all regular, narrow-complex tachycardias are treated in the same way with vagal maneuvers, drug therapy and cardioversion. In a stable patient, after vagal maneuvers and a single dose of adenosine 6 mg have been attempted and fail, it is most appropriate to give adenosine 12 mg intravenously

An 18-year-old boy presents to the emergency department with severe pain in his penis. He indicates he has had some mild irritation to the glans and foreskin area recently. After intercourse tonight he was unable to reduce the foreskin and now has severe pain in the tip of his penis. On examination, the penis is flaccid and the foreskin is retracted proximally. There is swelling to the tip of the penis with mild erythema. What is the most likely diagnosis? Paraphimosis Penile fracture Phimosis Priapism

Correct Answer ( A ) Explanation: The patient in this question has a story and exam consistent with paraphimosis. Paraphimosis is a true urologic emergency of the uncircumcised penis. It is the inability to reduce the proximal edematous foreskin distally over the glans penis into its natural position. The resulting glans edema and venous engorgement can progress to arterial compromise and gangrene. Paraphimosis can be caused by infection, masturbation, trauma or hair or clothing tourniquets. Patients typically appear very anxious and the history reveals that the patient retracted the foreskin and could not replace the foreskin over the glans. Physical examination reveals a flaccid proximal penis with erythema and engorgement distal to the obstruction. The foreskin is retracted and cellulitis may be present. The diagnosis of paraphimosis is based on clinical findings and should be intervened on immediately. Paraphimosis can often be reduced by compression of the glans for several minutes to reduce edema and allow for successful reduction of the now smaller glans through the foreskin. Phimosis (C) is the inability to retract the foreskin proximally. Infection, poor hygiene and previous preputial injuries with scarring are common causes of pathologic phimosis.

A 32-year-old woman presents to the emergency department with dysuria, back pain and fever for two days. She indicates she has had urinary tract infections in the past but never this severe. She denies other past medical history and does not take any medications. Vital signs show HR 114, BP 132/80, R 16, T 102.2°F. Examination shows a moderately ill appearing woman with right upper quadrant and right costovertebral angle tenderness. What test or tests are required in making an appropriate diagnosis in this patient? Urinalysis Urinalysis and contrast enhanced computed tomography of abdomen/pelvis Urinalysis and non-contrast enhanced computed tomography of abdomen/pelvis Urinalysis and ultrasonography of renal parenchyma

Correct Answer ( A ) Explanation: The patient in this question is presenting with signs and symptoms of acute pyelonephritis. Acute pyelonephritis is a urinary tract infection of the renal parenchyma and collecting system manifesting with the clinical syndrome of fever, chills, flank pain and nausea or vomiting in conjunction with a urinalysis that is consistent with infection. The majority of patients with acute pyelonephritis do not need emergency imaging to secure the diagnosis of pyelonephritis. Patients with either unusually severe signs and symptoms or an atypical presentation are candidates for imaging. Additionally, patients with a known current history of urinary tract infection who are receiving antibiotics but persistently have fevers and general toxicity require further imaging to look for complications from pyelonephritis such as abscess formation.

A 76-year-old man presents to the emergency department with shortness of breath and lightheadedness. Vital signs include blood pressure 70/56 mm Hg, heart rate 124 beats/minute, respiratory rate 22 breaths/minute, and temperature 37.6°C. He has distended neck veins and occasional dropped radial beats. His lungs are clear to auscultation, but his heart sounds are distant. He has some fullness to palpation of the right upper quadrant of his abdomen. Which of the following is the most appropriate diagnostic test? Bedside echocardiography Chest radiograph Computed tomography angiogram of the chest Electrocardiogram

Correct Answer ( A ) Explanation: This patient presents with a clinical picture consistent with obstructive shock. His distended neck veins, full right upper quadrant, muffled heart sounds, and hypotension are all consistent with pericardial tamponade. A pericardial sac slowly accumulating fluid can stretch without obstructing cardiac function. Tamponade occurs when rapid fluid accumulation results in elevated pressures that inhibit venous return. This is a dynamic process over the course of the cardiac cycle, therefore bedside echocardiography is the diagnostic test of choice. Fluid around the heart with evidence of right atrial compression and right ventricular diastolic collapse are diagnostic of pericardial tamponade.

What is the most likely cause of acute anemia in an African-American patient with HIV infection who recently began pneumocystis pneumonia prophylaxis? Glucose-6-phospahate-dehydrogenase (G6PD) Deficiency Hereditary spherocytosis Paroxysmal nocturnal hemoglobinuria Pyruvate kinase deficiency

Correct Answer ( A ) Explanation: The patient mostly likely has hemolytic anemia caused by oxidative stress in the setting of G6PD deficiency. Pneumocystis pneumonia prophylaxis is most commonly initiated with trimethoprim/sulfamethoxazole, an agent known to be associated with hemolysis in patients with G6PD deficiency. Up to 85% of the energy generated by RBCs to sustain membrane stabilization occurs via anaerobic glycolysis. At least 8 known enzymatic deficiencies are found in the glycolytic pathway, including pyruvate kinase deficiency, a cause of hemolytic jaundice, usually diagnosed in infants. The other 15% of energy comes from the production of nicotinamide adenine dinucleotide phosphate (NADPH) via the hexose monophosphate shunt pathway. NADPH is necessary for the reduction of oxidized glutathione, an antioxidant that contributes significantly to membrane stabilization. The first enzyme in the hexose monophosphate pathway is G6PD. Without G6PD, the RBC membrane is subject to hemolysis when exposed to oxidative stressing agents, including aspirin, antimalarials, nitrofurantoin, sulfa drugs, fava beans, and methylene blue. G6PD deficiency is diagnosed by enzymatic screening. Hereditary spherocytosis (B) is a congenital intrinsic membrane abnormality in which RBCs are shaped like spheres. Clinical sequelae can range from compensated hemolysis to aplastic crisis. Diagnosis is made via blood smear and osmotic fragility testing. Splenectomy is the definitive treatment. Paroxysmal nocturnal hemoglobinuria (PNH) (C) is a stem cell defect in which there is an abnormal sensitivity to complement mediated destruction of RBCs. This destructive process is a result of a defect in the formation of surface proteins on the red blood cell, which normally function to inhibit immune reactions. For this reason, blood transfusion can be fatal, and only washed blood cells should be given to patients with PNH. Pyruvate kinase deficiency (D) is a cause of hemolytic jaundice, usually diagnosed in infants.

A 75-year-old man with a history of renal colic presents with right flank pain radiating to the right groin. During triage, the patient has a syncopal event. Urinalysis shows no red cells. His blood pressure is 110/60 mm Hg. What is the best management indicated for this patient? CT abdomen and pelvis with contrast CT abdomen and pelvis without contrast Discharge home with urology follow up Intravenous antibiotics

Correct Answer ( A ) Explanation: The patient presents with signs and symptoms concerning for an abdominal aortic aneurysm (AAA) and should have a CT scan of the abdomen and pelvis with contrast to confirm the diagnosis. AAA occurs when there is weakening of all three layers (intima, media and adventitia) of the arterial wall creating a true aneurysm. It is rare before the age of 50 but is seen in 2-5% of men older than 50. The normal diameter of the infrarenal abdominal aorta is about 2 cm and any measurement over 3 cm defines a AAA. These aneurysms progressively enlarge over time and the risk of rupture markedly increases at diameters greater than 5 cm. Rupture of aneurysms under 4 cm is rare. In any older patient with abdominal pain, AAA should be considered. Patients often present with symptoms consistent with renal colic and this diagnosis often leads to missing early diagnosis of a leaking or large AAA. Syncope is common particularly in leaking or ruptured AAA. The combination of back and abdominal pain should prompt the consideration of AAA in all patients. Physical examination may reveal a pulsatile abdominal mass as well. Management of a large, leaking or ruptured AAA is directed at supportive care, blood product transfusion and emergent operative intervention.

65-year-old man presents to the ED with diffuse bone pain, abdominal cramping, and severe constipation. He is currently undergoing dialysis for end-stage renal disease, but has not been to dialysis this week. Physical exam reveals a lethargic man with diffuse abdominal tenderness without rebound or guarding. Which of the following is the most likely etiology for his symptoms? Hypercalcemia Hyperkalemia Hypomagnesemia Hypophosphatemia

Correct Answer ( A ) Explanation: This patient is exhibiting signs and symptoms consistent with hypercalcemia, which in a patient with chronic renal failure, is classically caused by tertiary hyperparathyroidism. The parathyroid gland secretes parathyroid hormone, which typically acts on the kidney to decrease calcium clearance (thus increasing serum calcium) and to decrease phosphate reabsorption (thus lowering serum phosphate). Parathyroid hormone levels are normally inversely proportional to serum calcium levels. In a patient with chronic renal failure, phosphate excretion decreases as the GFR decreases. As this occurs, the parathyroid gland secretes more parathyroid hormone in an attempt to decrease renal phosphate reabsorption. Over a long period of time, the hyperparathyroidism causes increased serum calcium levels by leaching calcium from the bones and preventing it from being cleared from the serum by the kidneys. This also leads to what is known as renal osteodystrophy. The serum calcium level will be high in these patients, and the parathyroid hormone levels will also be inappropriately high. Signs and symptoms of hypercalcemia include bone pain and remodeling, renal stones, abdominal pain, nausea, constipation, and psychiatric disturbances such as depression, lethargy, psychosis, and even coma. This can be remembered by the classic mnemonic of bones, stones, groans, and psychiatric overtones.

A 34-year-old woman presents to the ED with severe right eye pain. The pain began after she entered a dark movie theatre. On physical exam, you note a mid-dilated pupil (4 mm) and corneal edema. Which of the following is the correct series of treatment for this process? Acetazolamide IV, topical timolol, topical prednisolone, topical pilocarpine Mannitol IV, topical pilocarpine, topical timolol, topical prednisolone Topical prednisolone, mannitol IV, topical timolol, topical pilocarpine Topical timolol, topical pilocarpine, acetazolamide IV, topical prednisolone

Correct Answer ( A ) Explanation: This patient is exhibiting signs and symptoms of acute angle closure glaucoma. Acute angle closure glaucoma is defined by increased intraocular pressure caused by obstruction of aqueous humor drainage via the canal of Schlemm. The anterior chamber "angle" refers to the angle formed by the cornea and the iris. The ciliary body produces aqueous humor and is located posterior to the iris. The canal of Schlemm is located in the angle of the anterior chamber and serves to drain the aqueous humor in the anterior chamber. Contact between the iris and cornea due to anterior movement of the iris obstructs the canal of Schlemm. This happens when either there is increased pressure in the posterior chamber or pupillary dilation occurs causing narrowing of the anterior chamber and thus obstruction of the canal. Signs and symptoms of acute angle closure glaucoma include ocular pain, nausea and vomiting, unilateral blurring of vision, photopsia or colored halos around lights, elevated intraocular pressure (often 30 mm Hg or higher), conjunctival injection, a cloudy or steamy cornea due to corneal edema, and a mid-dilated nonreactive pupil. Management of acute angle closure glaucoma includes several essential steps. First, you need to reduce the production of aqueous humor using intravenous acetazolamide and a topical beta blocker such as timolol. Second, topical steroids such as prednisolone are needed to decrease inflammation. Finally, agents are needed to reverse the angle closure. Topical miotic agents such as pilocarpine are given one hour after the initiation of treatment and require two doses 15 minutes apart. Adjuncts, including hyperosmotic agents like intravenous mannitol, may be needed to further reduce the intraocular pressure two hours after the initiation of treatment if not adequately controlled. Ophthalmology should be consulted as soon as this diagnosis is suspected for definitive laser iridotomy.

A three-year-old boy is brought to the emergency department due to acetaminophen ingestion. About four hours prior, he was found by his mother with an open bottle of acetaminophen and several tablets scattered on the floor. The mother was not sure how many tablets were missing. The boy had one episode of vomiting at home. He is asymptomatic upon arrival at the emergency room. On examination, the boy is active with normal vital signs and physical examination. Which of the following is the most appropriate next step in management? Get acetaminophen level four hours after ingestion Obtain transaminases, synthetic and renal function Perform immediate gastric lavage Treat with syrup of ipecac urgently

Correct Answer ( A ) Explanation: Acetaminophen is commonly present in the home, which can be unintentionally ingested by young children. The initial signs of acetaminophen toxicity are nonspecific such as nausea and vomiting, which is usually followed by an asymptomatic period. Any child with a history of acute ingestion of greater than 200 mg/kg should be referred to a healthcare facility. If a toxic ingestion is suspected, a serum acetaminophen level should be calculated four hours after the reported time of ingestion. For patients who present to medical care more than four hours after ingestion, a stat acetaminophen level should be obtained. Acetaminophen levels obtained fewer than four hours after ingestion are difficult to interpret and cannot be used to estimate the potential for toxicity. The serum acetaminophen level is then plotted on the Rumack-Matthew nomogram and any level that is in the possible or probable hepatotoxicity range should then be treated with N-acetylcysteine.

Which of the following is the treatment of choice in preventing acute mountain sickness? Acetazolamide Dexamethasone Ginkgo biloba Nifedipine

Correct Answer ( A ) Explanation: Acute mountain sickness (AMS) is characterized by symptoms similar to those with a mild viral illness or "hangover" including headache, nausea, vomiting, fatigue, dizziness, and difficulty sleeping. Symptoms typically occur within hours of reaching a high altitude (generally > 8,000 feet) and peak in 24-48 hours. Slow ascent to allow time for acclimatization is the best method of prevention. When that is not possible or when there is a previous history of acute mountain sickness with ascent, use of acetazolamide (125-250 mg twice daily from one day prior to ascent and continuing for 48 hours after reaching altitude) has been shown to prevent the majority of symptoms. Acetazolamide is a carbonic anhydrase inhibitor that induces a renal bicarbonate diuresis resulting in a metabolic acidosis which thereby increases ventilation and arterial oxygenation. The bottom graphic demonstrates the decline in partial pressure of inspired oxygen compared to its value at sea level.

A 13-year-old boy is brought to the emergency room by his mother after a suicide attempt. She reports that he swallowed a bottle of pills but is unsure of what he ingested. His vital signs are BP 90/60, HR 135, RR 16, and T 100.3°F. On exam, the patient is somnolent, his pupils are dilated, mucous membranes are dry, and skin is warm and flushed. Which of the following substance did this patient most likely ingest? Amitriptyline Methylphenidate Oxycodone Sertraline

Correct Answer ( A ) Explanation: Amitriptyline is a tricyclic antidepressant (TCA) and the most frequent cause of poisoning in this class of drugs. Overdose can lead to a variety of anti-cholinergic symptoms, including tachycardia, dry mouth, mydriasis, hyperreflexia, warm flushed dry skin, gastrointestinal complaints, urinary retention, and confusion or agitation. Severe overdose can cause life-threatening symptoms such as coma, seizure, dysrhythmia, and cardiac arrest. Initial management may include gastric decontamination with activated charcoal but is otherwise supportive, depending on the presenting symptoms (intravenous fluids, cardiac agents, respiratory support). Methylphenidate (B) is an amphetamine, and overdose leads to sympathomimetic symptoms such as tachycardia, hypertension, hyperthermia, psychosis, seizures, and mydriasis.

A five-year-old boy is playing with his two-year-old sister when he calls to tell his mother that the girl put a dime in her mouth. The mother could not find anything in the girl's mouth so she brings her to the emergency department. Your physical examination is normal. Which of the following is the next best step? Obtain X-ray of the neck, chest, and abdomen Order barium contrast studies Provide reassurance to the mother Request for urgent endoscopy

Correct Answer ( A ) Explanation: Any history of foreign body ingestion should be taken seriously and investigated. The majority (80 percent) of foreign body ingestions occur in children between six months and three years of age. Coins and small toy items are the most commonly ingested foreign bodies. There may be symptoms of choking, gagging, and coughing that may be followed by excessive salivation, dysphagia, food refusal, emesis, or pain in the neck, throat, or sternal notch regions. There may be respiratory symptoms like stridor, wheezing, cyanosis, or dyspnea that may be encountered if the esophageal foreign body impinges on the larynx or the membranous posterior tracheal wall. Symptoms of cervical swelling, erythema, or subcutaneous crepitations suggest perforation of the oropharynx or proximal esophagus. The evaluation of a child with a history of foreign body ingestion should begin with plain anteroposterior (AP) radiographs of the neck, chest, and abdomen, along with lateral views of the neck and chest.

A 36-year-old man presents after being bitten by "something". He felt a pinprick sensation on his forearm while cleaning out his garage followed by worsening pain over the next sixty minutes that spread to involve his entire arm. Shortly after, he began having severe cramping of his chest and abdomen. What was he likely bitten by? Black widow spider Brown recluse spider Hornet Ixodes tick

Correct Answer ( A ) Explanation: Black widow spiders, Latrodectus mactans, are found throughout the United States. The female is twice as large as the male and is the only one able to envenomate humans. The spider is glossy black and has a bright red marking, either two spots or an hourglass figure, on the abdomen. It is not typically aggressive unless protecting its eggs. The venom is highly potent and results in release of acetylcholine resulting in neuromuscular symptoms and norepinephrine resulting in cardiovascular symptoms. Patients will typically recall a pinprick sensation at the site of the bite followed by mild erythema and swelling. Within minutes to an hour, the pain increases and spreads to the entire body. Severe cramping pain occurs in the chest, abdomen, and back and can mimic an acute abdomen. Systemic symptoms include nausea, vomiting, headache, and dizziness. Hypertension and tachycardia are common. Symptoms typically last several hours to days, with resolution in 2-3 days. Young children and patients with preexisting hypertension and cardiovascular or cerebrovascular disease are at highest risk for complications.

Which of the following describes a patient with Brown-Séquard syndrome? Ipsilateral loss of motor strength, vibratory sensation, and proprioception with contralateral loss of pain and temperature sensation below the level of injury Ipsilateral loss of pain and temperature and contralateral loss of motor strength, vibratory sensation, and proprioception Pain, loss of temperature below the level of injury, and complete loss of motor function, but retains proprioception and the ability to sense vibration and deep pressure Preservation of motor function with loss of proprioception and vibration below the level of injury

Correct Answer ( A ) Explanation: Brown-Séquard syndrome results from hemitransection of the spinal cord with unilateral damage to the corticospinal and spinothalamic tracts. This injury is usually the result of penetrating injuries or a lateral mass fracture of the cervical spine. It is also caused by spinal cord tumors, infections, and ischemia. It is rarely seen in its pure form, typically occurring with incomplete involvement of related tracts. Prognosis is excellent, with most patients recoverin

A seven-year-old boy is involved in a rollover motor vehicle collision. He arrives intubated by the paramedic service. On exam, you note him to be flexing both upper extremities. Which of the following is the most sensitive indicator of herniation? Bradycardia Hypertension Hypotension Irregular respiration

Correct Answer ( A ) Explanation: Cushing's triad is usually a pre-terminal event seen in patients with increased intracranial pressure and cerebral herniation through the foramen magnum. It is associated with decreased level of alertness, hypertension, bradycardia, and irregular respirations. Bradycardia is the first sign and is therefore the most sensitive indicator. Although hypotension (C) can be seen in neurological trauma, it is more commonly seen in neurogenic shock and is not part of Cushing's triad. Hypertension (B) and irregular respirations (D) are part of Cushing's triad, but they are less sensitive and manifest later in the course than bradycardia.

You are called to the emergency department to see a five-year-old boy for ingestion. He was playing in the garage when his mother found him with an opened bottle of antifreeze. The liquid was all over his mouth and clothes. He had two episodes of vomiting and fell asleep on the way to the hospital. On physical examination, you note tachycardia. Which of the following laboratory abnormalities would you expect in this type of ingestion? Calcium oxalate crystals in urine Decreased lactate levels Hypercalcemia Non-anion gap metabolic acidosis

Correct Answer ( A ) Explanation: Ethylene glycol is commonly found in antifreeze. Unintentional ingestion is the most common exposure in children, and even small-volume ingestions of concentrated products have the potential for severe toxicity. Early symptoms begin with nausea, vomiting, CNS depression. Late manifestations include anion gap metabolic acidosis, hypocalcemia, and kidney failure (secondary to deposition of calcium oxalate crystals in the renal tubules). It can be diagnosed by ethylene glycol blood concentrations. Osmolar gap may also be used as a surrogate marker. Examination of the urine with a Wood lamp is neither sensitive nor specific for ethylene glycol ingestion. Calcium oxalate crystals can be seen on urine microscopy. The evaluation of patients should include electrolytes (including calcium), acid-base status, kidney function, and ECG. Gastric decontamination is generally not of value because ethylene glycol is rapidly absorbed. The classic antidote for ethylene glycol poisoning was ethanol. However ethanol has been replaced by fomepizole, a potent competitive inhibitor of alcohol dehydrogenase. This is because of the ease of administration, lack of CNS and metabolic effects, and overall excellent patient tolerability profile of fomepizole.

A 24-year-old woman presents to the Emergency Department after a high-speed motor vehicle collision. Her GCS is 14. On neurological examination, she has intact fine touch, but decreased strength and loss of pain and temperature sensation below the T10 level. What is the most likely diagnosis? Anterior cord syndrome Brown-Séquard syndrome Cauda equina syndrome Central cord syndrome

Correct Answer ( A ) Explanation: Injury to the anterior two-thirds of the spinal column results in anterior cord syndrome which is characterized by bilateral loss of motor function and pain and temperature sensation below the level of injury. The dorsal column is usually spared so vibration sense and proprioception are preserved. Anterior cord syndrome occurs in cases of disruption or injury to the anterior spinal artery which can result from protrusion of bone fragments from a traumatic injury to the vertebrae, disc herniation, or following aortic surgery. It can also result from a flexion/compression injury to the cord. Most improvement in function occurs in the first 24 hours. Anterior cord syndrome carries the worst prognosis of all the incomplete spinal cord syndromes, with less than 20% regaining any degree of muscle function. Brown-Séquard syndrome (A) results from hemisection of the spinal cord, often due to penetrating injuries. Patients present with ipsilateral motor, vibration and proprioception loss, and contralateral pain and temperature loss. Cauda equina syndrome (C) is typically seen in cases of midline disk herniation at the L4-5 level that results in dysfunction of nerve roots, not the spinal cord itself. Patients present with urinary retention with overflow incontinence, fecal incontinence, distal motor weakness, and sensory loss in a saddle distribution. Central cord syndrome (D) is characterized by bilateral motor paresis with the upper extremities affected to a greater degree than the lower extremities. Sensory loss is variable

A 1-year-old boy presents unconscious to the emergency department with multisystem trauma after he was a passenger in a motor vehicle accident. He is placed a cervical collar. Which of the following, can definitively clear the cervical spine? Cervical spine computed tomography scan Cervical spine magnetic resonance imaging Cervical spine radiographs Clinical clearance

Correct Answer ( A ) Explanation: The Glasgow coma scale (GCS) is a grading of patient responsiveness based on three components: eye opening (1 to 4 points), verbal response (1 to 5 points), and motor response (1 to 6 points). The severity of traumatic brain injury (TBI) is defined using the GCS with mild TBI being defined as a GCS of 14-15, moderate TBI a GCS of 9-13, and severe TBI a GCS 3-8. While other grading tools for mental status exist, the GCS is commonly used due to its interobserver reliability and prognostic value. The Glasgow coma scale has been shown to have prognostic value in traumatic brain injury, spontaneous subarachnoid hemorrhage, and bacterial meningitis. Limitations of the scale include its decreased utility in the setting of drugs, alcohol, sedation, and intubation and its reliance on behavioral response without regard for the underlying pathophysiology. The patient above would receive a score of 2 for eye opening, 3 for verbal response, and 5 for motor response for a total of 10.

72-year-old man presents to the Emergency Department with left eye pain. Earlier this evening he was involved in a fistfight. He reports getting punched in the eye with no loss of consciousness. He now has eye pain and blurry vision. On physical exam his visual acuity is 20/50 in the left eye and 20/20 in the right eye. His intraocular pressure is 14 on the left. His left eye is shown above. What is the most likely diagnosis? AHordeolum BHyphema CSubconjunctival hemorrhage DVitreous hemorrhage

Correct Answer ( B ) Explanation: He has evidence on exam of a traumatic hyphema. This can occur from either penetrating or blunt trauma. Most commonly a high energy object impacts the globe causing a rupture of the blood vessels in the iris or ciliary body. Then the hemorrhage layers in the anterior chamber and can partially obstruct the visual field. If no evidence of globe rupture is present, then intraocular pressure measurement should be obtained. Approximately one third of patients will develop intraocular hypertension within 1 week of the injury. Initial management includes an eye shield, activity restrictions, steroid eyedrops, and close ophthalmology follow-up.

A 32-year-old woman presents with dysuria and lower abdominal discomfort. She is sexually active with one male partner and uses condoms inconsistently. She denies vaginal bleeding or discharge, and her pelvic examination is unremarkable. Urinalysis shows 10 WBC/hpf with positive nitrites. Which of the following is the most likely causative organism? AAcinetobacter BE. coli CEnterobacter DPseudomonas

Correct Answer ( B ) Explanation: Acute cystitis is isolated bladder infection. Women are more likely than men to have lower urinary tract infections due to shorter urethral length. Urinary tract infections (UTIs) typically result from ascending bacteria from the urethra. Symptomatic patients with a urine culture that grows 102 or 103 colony forming units (CFU)/mL is considered to be infected. The concentration of bacteria in the bladder can increase exponentially in women after sexual intercourse. E. coli is the most common causative organism and is responsible for >80% of UTIs. Escherichia coli (E. coli) converts nitrates to nitrite, which is a very useful test to confirm the diagnosis of a UTI. The urine nitrate reaction has >90% specificity. Other organisms causing UTIs include Pseudomonas, Acinetobacter, Enterobacter, Klebsiella, and Proteus species.

A 70-year-old man presents to the ED after a fall down twelve stairs at home. He complains of a headache. There is a large parietal scalp hematoma. His non-contrast computed tomography scan of the head is shown above. Which of the following is the most likely source of bleeding? Bridging veins Meningeal veins Middle meningeal artery Superior sagittal sinus

Correct Answer ( A ) Explanation: This patient's imaging and physical examination are consistent with a traumatic acute subdural hematoma. A subdural hematoma is a collection of blood between the dura and the arachnoid mater. Subdural hematomas occur when there is rupture of the bridging veins from movement of the brain relative to the skull, such as that seen with acceleration-deceleration injuries. Subdural hematomas occur most commonly in individuals with brain atrophy, such as alcoholics and elderly patients. This results in blood filling the potential space between the dura and arachnoid. Subdural hematomas may be acute, subacute, or chronic. Acute subdural hematomas are characteristically seen after a traumatic head injury. Patients typically present with headache, mental status changes, seizures, or focal deficits. Subdural hematoma is diagnosed on non-contrast CT of the head and appears as a crescent-shaped hematoma that may cross suture lines. Management of subdural hematomas includes neurosurgical consultation for possible surgical evacuation; however, small subdural hematomas may be closely observed with serial non-contrast CTs of the head. Indications for emergent surgery in a patient with a subdural hematoma include neurologic deterioration or > 5 mm midline shift on CT.

A 56-year-old woman with a history of hypertension, diabetes mellitus, and hypercholesterolemia presents with acute, substernal chest pain that is worse with exertion and associated with nausea and shortness of breath. Her sister and her mother both had "heart attacks" before age 55 years. Which of the following offers a proven mortality benefit in acute coronary syndrome? Amlodipine Aspirin Morphine Nitroglycerin

Correct Answer ( B ) Explanation: Aspirin is the prototypical antiplatelet agent for acute coronary syndrome and irreversibly inhibits platelet activity for the entire platelet life cycle (approximately eight to ten days). It also inhibits thromboxane A2 synthesis and has indirect antithrombotic activity. Aspirin has been proven to reduce mortality in patients with acute myocardial infarction by 25-50% both independently and with the use of fibrinolytic therapy. The standard dose is 324 mg of non-enteric coated aspirin that is chewed and swallowed

An 8-year-old African-American girl with a history of sickle cell anemia presents with diffuse pain consistent with an acute sickle cell pain crisis. While in the emergency department, she develops acute onset headache, right sided facial droop and right arm weakness. A CT scan confirms the diagnosis. Which of the following is the next best step in management? Alteplase Exchange transfusion MRI brain Tranexamic acid

Correct Answer ( B ) Explanation: Cerebrovascular events are a potential complication of sickle cell disease. This patient developed symptoms concerning for acute ischemic stroke. For pediatric patients with acute ischemic stroke in the setting of sickle cell disease, exchange transfusion is the treatment of choice. Transfusion goals include decreasing hemoglobin S levels to less than 30% and obtaining a total hemoglobin level of 10 g/dL. For adults with acute ischemic stroke in the setting of sickle cell disease, consider tissue plasminogen activator (tPA). tPA is not indicated or approved for use in pediatric patients Tranexamic acid (D) is an antifibrinolytic agent used in the management of hemorrhage. Indications include acute traumatic hemorrhage, intraoperative blood loss, and obstetric hemorrhage. Tranexamic acid is not used for ischemic events

A 58-year old man presents with 2 days of fever and lower back pain. Digital rectal exam reveals a swollen and tender prostate. His urinalysis reveals 100 WBC/hpf, leukocyte esterase and nitrites. What is the most likely pathogen causing this condition? Chlamydia trachomatis Escherichia coli Klebsiella Pseudomonas

Correct Answer ( B ) Explanation: Eighty percent of the cases of acute bacterial prostatitis are secondary to infection with Escherichia coli. Acute bacterial prostatitis is characterized by fever, low back and perineal pain. Constitutional symptoms include malaise, fatigue, myalgias and arthralgias. Patients may also experience increased urinary frequency, urgency and dysuria, as well as urinary retention. Prostate exam reveals a tender, swollen and boggy prostate. Acute cystitis often accompanies acute prostatitis and therefore urine culture generally gives the causative pathogen. Chlamydia trachomatis (A) and Neisseria gonorrhea should be considered in patients less than 35 years of age and those with multiple sexual partners. Other less common causes of bacterial prostatitis include Klebsiella (C), Pseudomonas (D), Enterobacter and Proteus.

A 65-year-old man with a past medical history of hypertension presents to the Emergency Department with "tearing" chest and abdominal pain radiating towards his back. His blood pressure is 185/98 mm Hg. Which of the following medications should be administered first? Aspirin Esmolol Heparin Nitroprusside

Correct Answer ( B ) Explanation: Esmolol is the first line treatment for a hypertensive patient with an aortic dissection. Rate-controlling medication (e.g. esmolol) should be given before vasodilators to prevent reflex tachycardia and subsequent increase in aortic shearing forces. An aortic dissection occurs when the intima tears allowing blood to cause a false channel and intramural hematoma. If this extends to the heart it can precipitate myocardial infarction, severe aortic regurgitation, or pericardial tamponade leading to death. Risk factors include a history of hypertension, collagen vascular disease or aortic aneurysm, as well as current pregnancy and cocaine use. Based on the Stanford classification, a dissection is classified as type A if it involves the ascending aorta while a type B dissection is limited to the descending aorta. Treatment of a type A dissection requires surgical management while type B dissections are typically managed medically Nitroprusside (D) reduces blood pressure through vasodilation and can be given after a rate controlling agent, such as esmolol, has been initiated

A 25-year-old woman with a history of hypertension presents to the Emergency Department after a syncopal event. She is complaining of lower abdominal pain. Her heart rate is 132 beats/minute and her blood pressure is 85/41 mm Hg. Her urine pregnancy test is positive. Which is the following is the most appropriate next step in management? Administer rho(D) immune globulin Establish intravenous access Obtain a 12-lead electrocardiogram Order an abdominal ultrasound

Correct Answer ( B ) Explanation: Establishing intravenous access is the most appropriate next step in this hemodynamically unstable patient. This patient likely has significant hemorrhage from a ruptured ectopic pregnancy and requires fluid resuscitation and possible blood transfusion. An ectopic pregnancy occurs when the developing blastocyst implants somewhere outside of the endometrium of the uterus. Risk factors include smoking tobacco, multiple sexual partners, fertility medications (e.g. clomiphene citrate), and a history of pelvic inflammatory disease. Symptoms include abdominal pain, vaginal bleeding, and in severe cases signs of hypovolemic shock (e.g. hypotension, tachycardia, syncope). Medical therapy with methotrexate can be considered to treat a select group of stable patient with no signs of rupture. However, unstable patients require emergency surgery.

A 22-year-old woman presents with symmetric lower extremity weakness. She recently had a diarrheal illness. Which of the following might you expect on further work-up? Creatine kinase elevation Elevated protein of the cerebrospinal fluid Hyperreflexia Meningeal enhancement on MRI of the brain

Correct Answer ( B ) Explanation: Guillain-Barré syndrome (GBS) is the most commonly encountered peripheral neuropathy in developed countries despite an overall rare annual incidence of 1-2 cases per 100,000 population. Patients often have an antecedent respiratory or gastrointestinal infection although the duration of time between infection and development of neuropathy is quite variable. The most common organisms associated with GBS are Campylobacter jejuni, Cytomegalovirus (CMV), Epstein Barr virus (EBV), and Mycoplasma pneumoniae. Macrophages attack the myelin finding antigens similar to those expressed on the recent infectious agent. Patients develop symmetric weakness usually worse distally. It is often described as an ascending paralysis with progressive weakness and loss of reflexes. The tone of the rectal sphincter is preserved. There may be sensory involvement as well although the degree to which this occurs is variable. More than 90% of cases are the classic acute inflammatory demyelinating polyneuropathy. Approximately half of patients have autonomic dysfunction and up to 1/3 will require ventilatory support. Electrophysiologic testing helps identify the peripheral neuropathy. Additionally, lumbar puncture is abnormal showing a markedly elevated CSF protein with a mild pleocytosis. The CSF may be normal early in the course of the disease and is not sensitive enough to rule out the process. However, in the context of a patient with symptoms consistent with the syndrome and these CSF findings the diagnosis is made. Patients are treated with plasma exchange or intravenous immunoglobulin (IVIG).

Which of the following is the most common pulmonary complication of influenza? Empyema Pneumonia secondary to superimposed bacterial infection Primary viral pneumonia Spontaneous pneumothorax

Correct Answer ( B ) Explanation: Influenza is generally a self-limited illness of the upper respiratory tract that presents with fever, myalgias, headache, cough and malaise. Transmission of infection is via aerosolized respiratory secretions from an infected individual, with symptoms typically starting one to four days after exposure. Although most patients have an uneventful course with symptoms lasting one to two weeks, select patients are at higher risk of complications including those over 65 years of age or less than 2 years of age, patients with known pulmonary or cardiovascular disease, chronic renal insufficiency, diabetes, immunosuppression and morbid obesity, and pregnant women. The most frequent complication of influenza is pneumonia secondary to a superimposed bacterial infection. Streptococcus pneumoniae is the most frequently implicated pathogen, with Staphylococcus aureus and Haemophilus influenzae being cited as well. Patients with a superimposed bacterial pneumonia often initially show improvement in acute symptoms before developing worsening productive cough, recurrent fever and radiographic evidence of a pulmonary infiltrate.

An 80-year-old woman, with a history of type 2 diabetes mellitus, presents with slowly worsening vision. Her vision is worse in low intensity light and she has blurring in the center of her vision. Physical examination shows intact peripheral visual fields with a positive Amsler grid distortion. Fundoscopic exam reveals drusen located in the macula of the retina. What is the most likely diagnosis? Diabetic retinopathy Macular degeneration Open-angle glaucoma Presbyopia

Correct Answer ( B ) Explanation: Macular degeneration is characterized by central vision loss caused by atrophy of the cells in the central macular region of the retinal pigment epithelium. This leads to loss of central vision with the peripheral visual fields remaining intact. Risk factors are multifactorial and include advanced age, tobacco abuse, genetic predisposition, race, obesity, and existing cardiovascular disease. Symptoms include wavy or distorted central vision and central blind spots. An Amsler grid will be perceived as distorted by the patient on exam. Fundoscopic examination in early disease shows drusen. In late disease, areas of depigmentation of the macula can be seen. In the severe exudative form, subretinal neovascularization may be visualized. Treatments include laser photocoagulation and intravitreal antiangiogenic therapy. Diabetic retinopathy (A) can also be largely asymptomatic or can manifest as slow decrease in visual acuity. Fundoscopic exam may reveal intraretinal hemorrhages, cotton-wool spots, neovascularization, and macular edema. Open-angle glaucoma (C) is characterized by optic neuropathy and can be, but is not always, associated with increased intraocular pressure. Risk factors include family history, advanced age, and diabetes mellitus. Symptoms include a gradual loss of peripheral visual fields, but often patients are asymptomatic. Presbyopia (D) is a normal decrease in near-vision as a part of aging. The fundoscopic exam should be normal.

Which of the following is the most common injury seen in a child with severe blunt chest trauma such as being struck by a motor vehicle? Pneumothorax Pulmonary contusion Rib fracture Tracheobronchial laceration

Correct Answer ( B ) Explanation: Major life-threatening thoracic injuries require a significant amount of force and are uncommon in low mechanism injuries, such as a fall from a bicycle or swing. Children with severe thoracic trauma are usually victims of a motor vehicle crash and rarely present with penetrating trauma. Patterns of chest injuries in children are different than in adults because the chest wall is more elastic and mediastinal structures are more mobile. As such, many children with significant thoracic injuries have no external signs or even rib fractures. The most common injury in children with severe blunt chest trauma is a pulmonary contusion, occurring in nearly half of such victims. Patients may complain of chest pain or difficulty breathing but often have no other symptoms. Patients with a pulmonary contusion should be monitored closely for clinical deterioration.

A 28-year-old woman presents to the ED following a finger injury while playing flag football. The patient states that she sustained a direct blow to the pointer finger when the ball struck her fingertip. On physical exam, the patient has swelling and tenderness at the distal interphalangeal joint. The fingertip rests in 45 degrees of flexion. The patient is unable to demonstrate active extension of the distal interphalangeal joint but she has full passive extension. The skin is intact. X-ray of the finger is obtained which shows soft tissue swelling but no fracture or dislocation. Which of the following is the most appropriate management for this patient? Buddy taping Extension splinting of the distal interphalangeal joint Extensor tendon reconstruction Physical therapy for range of motion and strengthening

Correct Answer ( B ) Explanation: Mallet finger occurs when the extensor tendon of the distal interphalangeal (DIP) joint is ruptured. This injury commonly occurs with ball sports when a patient sustains a direct blow to the fingertip resulting in forced flexion of the DIP joint. Mallet finger injuries may occur with or without a dorsal avulsion fracture of the distal phalanx. Treatment consists of extension splinting of the DIP joint as long as the injury is acute (< 12 weeks) and there is no displaced bony injury. An aluminum splint or stack splint applied to either the dorsal or palmar surface of the middle and distal phalanges is appropriate. The proximal interphalangeal joint should be allowed free motion and should not be included in the splint. Total length of splinting is six to eight weeks.

A 28-year-old woman presents with pain and swelling of her left breast, fever, and myalgias. She had a full-term vaginal delivery one week ago and is breastfeeding her infant. On examination, she has a temperature of 38.1oC. The superior lateral quadrant of the left breast is erythematous, warm, and tender to palpation. There is no fluctuance appreciated. What is the next step in management of this patient? AInstruct her to continue breastfeeding and start intravenous vancomycin BInstruct her to continue breastfeeding and start oral cephalexin CInstruct her to discontinue breastfeeding and start intravenous vancomycin DInstruct her to discontinue breastfeeding and start oral cephalexin

Correct Answer ( B ) Explanation: Mastitis is most frequently a complication of breastfeeding, usually in the first six weeks postpartum, although it may occur in non-breastfeeding women as well. Risk factors include poor drainage, prolonged engorgement, and cracked nipples. Staphylococcus aureus is the usual cause with transmission occurring via inoculation from the infant's nasopharyngeal secretions. Patients present with signs of cellulitis (erythema, warmth, and tenderness) localized to one area of the breast, as well as fever. Fluctuance, or expression of pus from the nipple, is unlikely unless an abscess has developed. Treatment involves warm compresses and analgesics. In cases of simple mastitis, breastfeeding should continue to ensure complete drainage of the breast. Oral antibiotics (dicloxacillin or cephalexin) should be prescribed. Patients who fail initial treatment should have an ultrasound to rule out abscess formation and antibiotic coverage should be expanded.

A 25-year-old man present to the Emergency Department after a gunshot wound to the chest. He is awake and following commands. His vital signs are T 97.5ºF, BP 89/50 mm Hg, HR 120 beats/minute, RR 35 breaths/minute, and oxygen saturation 87% on a nonrebreather facemask. Jugular venous distension is present and he has diminished breath sounds on the left. Which of the following is the next best step in management? Focused assessment with sonography for trauma examination Needle decompression of the chest Pericardiocentesis Upright chest X-ray

Correct Answer ( B ) Explanation: Needle decompression of the chest in the second intercostal space in the midclavicular line is recommended for emergent management of a tension pneumothorax. A tension pneumothorax is caused by a pleural defect that creates a one-way valve so that extrathoracic air enters the thorax, but is not able to escape. This leads to a mass effect within the chest and creates tension physiology by shifting mediastinal structures away from the side of the defect. This can lead to hypotension and shock as venous structures (e.g. superior vena cava) are compressed, which leads to reduced preload and eventually obstructive shock. A tension pneumothorax is a clinical diagnosis, and definitive management should not be delayed to obtain imaging. Clinical findings include diminished or absent breath sounds on the side of the injury, tracheal deviation away from the side of the injury, and evidence of mediastinal mass effect (e.g. hypotension, jugular venous distension). As part of the primary survey of any trauma patient, breathing should be assessed. When a tension pneumothorax is clinically suspected, appropriate management is immediately indicated before completing the survey. Management involves needle decompression with a large bore (i.e. 14 gauge or greater) angiocatheter followed by chest tube insertion.

A 6-week-old infant presents to your office for a well child exam. The child is underweight for her age and her mother says that she's not eating well. Physical exam reveals a continuous heart murmur in the left infraclavicular space and a thrill in the suprasternal notch. Which of the following findings would be expected on echocardiogram? Overriding aorta Patent ductus arteriosus Right ventricular outflow obstruction Ventricular septal defec

Correct Answer ( B ) Explanation: Patent ductus arteriosus (PDA) is a congenital heart defect that occurs when the fetal ductus arteriosus fails to close after birth. In vitro, the ductus arteriosus is a normal structure that connects the pulmonary artery to the descending aorta. This structure keeps the blood away from the pulmonary bed while in utero and is kept patent by low arterial oxygen and circulating prostaglandin E2, which is produced by the placenta. Changes to circulation occur at birth with the need for blood to enter the lungs, resulting in constriction of the ductus arteriosus beginning at birth and reaching completion by age two to three weeks. Clinical manifestations of PDA depend on the size of the PDA as well as the degree of left-to-right shunting. Patients with small PDAs may be asymptomatic, while those with larger ones can present with failure to thrive, respiratory distress, poor feeding and other signs of heart failure. Physical exam findings include a continuous "machine-like" heart murmur in the left infraclavicular space. Diagnosis is based on clinical manifestations and confirmed by echocardiogram, which allows for visualization of the patent ductus arteriosus. Treatment options include intravenous indomethacin if detected within the first 10-14 days of life or surgical interventions for more severe cases. One of the most common congenital heart disorders is tetralogy of Fallot, which is made up of four anatomical findings. These patients are found to have an overriding aorta (A), right ventricular outflow obstruction (C), a large ventricular septal defect (D) and right ventricular hypertrophy. They have difficulty feeding, episodes of turning blue while crying or feeding ("Tet spells") and failure to thrive. Treatment is surgical.

A 49 year-old woman presents with chest pain and shortness of breath. Her pain is constant, non-exertional and does not radiate into the back. She denies hemoptysis, history of previous venous thromboembolism, trauma or surgery within the last 4 weeks or unilateral leg swelling. She takes no medications. She smokes a half pack of cigarettes per day. Her vital signs at triage are temperature 98.6°F, blood pressure 120/80 mm Hg, heart rate 108 bpm. Her heart rate improves to 97 bpm upon reevaluation. Her physician decides to evaluate her for a pulmonary embolism. In addition to an ECG and a chest X-ray, which of the following is the most appropriate next step in diagnosis of pulmonary embolism? CT pulmonary angiogram D-dimer followed by a CT pulmonary angiogram if positive Use the pulmonary embolism rule out criteria to exclude pulmonary embolism V/Q scan

Correct Answer ( B ) Explanation: Patients who present with signs and symptoms concerning for a pulmonary embolism should be evaluated by a stepwise approach. The first step is to determine the likelihood of pulmonary embolism. Physician gestalt has been shown to have similar sensitivity and specificity as the Wells' criteria. If the patient is deemed to be low risk, either by the Wells' criteria or clinician gestalt, the Pulmonary Embolism Rule-out Criteria (PERC) rule should be evaluated. If the patient has any one of the following, D-dimer testing should be performed: age ≥ 50 years, HR ≥ 100 bpm at any time in the emergency department, room air oxygen saturation < 95%, prior history of venous thromboembolism, trauma or surgery within 4 weeks, hemoptysis, exogenous estrogen use, or unilateral leg swelling. This patient is low risk for pulmonary embolism but had a heart rate greater than 100 bpm at triage, and therefore, should be evaluated by a D-dimer followed by a CT pulmonary angiogram if positive.

A 6-year-old boy with a past medical history of glucose 6-phosphate dehydrogenase deficiency presents to the Emergency Department for a cough associated with vomiting. His parents state that he has been coughing every day for the last week. He is unimmunized but otherwise healthy. He appears well on exam with a dry cough and occasional wheezes. His laboratory workup reveals a leukocytosis with lymphocytic predominance and a chest X-ray with peribronchial thickening. He was seen by his pediatrician three days ago and tested positive for pertussis. Which of the following is the most appropriate therapy at this time? Ampicillin-sulbactam Azithromycin Ertapenem Trimethoprim-sulfamethoxazole

Correct Answer ( B ) Explanation: Pertussis is a highly contagious acute respiratory infection caused by the gram-negative rod Bordetella pertussis. Infection is spread by respiratory droplets. Childhood vaccination and natural immunity do not confer lifelong immunity against pertussis. School-aged children are usually affected, but adults may be carriers. Three clinical phases exist: catarrhal, paroxysmal and convalescent. The catarrhal phase, during which infectivity is highest, lasts one to two weeks and is characterized by cough, low-grade fever, rhinitis and anorexia. The paroxysmal phase lasts two to six weeks and is characterized by defervescence, increased coughing, and paroxysms of coughing with inspiratory "whooping" and posttussive emesis. The convalescent phase may include a cough lasting several months. The diagnosis is often clinical but is commonly missed due to atypical presentations and misdiagnosis as bronchitis. A polymerase chain reaction of nasopharyngeal secretions, nasopharyngeal culture or serological antibodies may be used for definitive diagnosis. Diagnostic workup may reveal lymphocytosis and radiographic evidence of peribronchial thickening, atelectasis or pulmonary consolidation. Treatment is with azithromycin, although trimethoprim-sulfamethoxazole may be used in macrolide-intolerant patients. Treatment is best if started during the first week of the illness and may have limited utility after the catarrhal phase.

A 22-year-old woman who is sexually active with multiple partners presents with dysuria for two days. Urine pregnancy test is negative. Her urinalysis reveals 36 WBC/hpf and is nitrite and leukocyte esterase positive. She is diagnosed with cystitis. What is the most likely bacterial pathogen? Chlamydia trachomatis Escherichia coli Klebsiella species Proteus species

Correct Answer ( B ) Explanation: Urinary tract infections are a common diagnosis among women. Escherichia coli is by far the most common bacterial cause of a urinary tract infection. Infection typically occurs due to ascending infection from the urethra to the bladder. Women are more frequently affected than men due to the short length of the female urethra. Presentation with isolated dysuria without flank pain, fever, or nausea in a normal woman is acute cystitis. Acute pyelonephritis is an infection of the kidney and often manifests with flank pain, fever, nausea, and vomiting.

A 67-year-old man with a history of bronchogenic carcinoma present with fever and shortness of breath. His vital signs are T 101.5ºF, HR 112 beats/minute, RR 22 breaths/minute, BP 100/60 mm Hg, and oxygen saturation 92% on room air. He has right basilar crackles on auscultation and dullness to percussion. A chest X-ray demonstrates a right lower lobar consolidation with costophrenic angle blunting. Which of the following values suggests the presence of an exudative pleural effusion? Partial pressure of arterial oxygen (PaO2) < 70% Pleural fluid LDH twice that of serum LDH Pleural fluid pH 7.6 Pleural fluid:serum protein ratio < 0.5

Correct Answer ( B ) Explanation: Pleural effusions are fluid collections within the potential space between the parietal and visceral pleura. In developed countries, the most common causes are pneumonia, malignancy and congestive heart failure. Effusions have traditionally been classified as either transudative or exudative. Exudative effusions are due to pleural diseases that produce pleural fluid secretion or leakage with high protein content, whereas transudative effusions have a low protein content. The Light criteria for the diagnosis of a pleural exudative effusion includes the presence of at least one of the following: pleural fluid lactate dehydrogenase (LDH) to serum LDH ratio > 0.6, pleural fluid to serum protein ratio > 0.5, or pleural fluid LDH greater than two-thirds the upper limit of normal for serum LDH. Therefore, a pleural fluid LDH twice that of serum LDH would suggest the presence of an exudative effusion. Effusions may be radiographically apparent on chest X-rays with as little as 150 to 200 mL of fluid in the hemithorax. Lateral decubitus films may help further evaluate for dependent fluid layering. Pleural fluid is accessed by performing a thoracentesis under sterile technique. A diagnostic thoracentesis aids in differentiating the etiology of the effusion. A therapeutic thoracentesis, in which 1 to 1.5 L of fluid is drained, may be performed for patients experiencing dyspnea at rest.

Which of the following is the most common cause of status epilepticus? Hypoglycemia Medication noncompliance Meningitis Trauma

Correct Answer ( B ) Explanation: Status epilepticus has been traditionally defined as continuous seizure activity lasting longer than 30 minutes. This is the estimated duration necessary for neuronal injury. Recently, the definition of status epilepticus has changed. It is now defined as a seizure lasting more than 5 minutes or more than 2 discrete seizures without intervening recovery of consciousness. The most common cause of status epilepticus is discontinuation of antiepileptic therapy and medication noncompliance. Status Epilepticus Seizure lasting > 5 minutes or > 2 discrete seizures without recovery of consciousness Causes: antiepileptic therapy discontinuation, medication noncompliance BZDs, phenytoin (second line), phenobarbital (third line)

Which of the following tests is used to diagnose benign paroxysmal positional vertigo? Caloric testing Dix-Hallpike maneuver Head thrust test Rinne test

Correct Answer ( B ) Explanation: The Dix-Hallpike maneuver is used to reproduce vertigo in patients with positional dizziness. With the patient in a seated position, the neck is extended and turned 45 degrees to one side. The patient is then rapidly placed into a supine position with the head hanging over the edge of the bed. The patient is kept in this position for 30 seconds while evaluating for the presence of nystagmus. The test can then be performed on the other side. This maneuver will usually provoke symptoms in patients with benign paroxysmal positional vertigo. A positive test is reproduction of vertigo and the presence of nystagmus. The head thrust test (C) is used in the diagnosis of vestibular neuronitis. The head thrust test is positive when the examiner quickly turns the patient's head and they are unable to maintain visual fixation. The Rinne test (D) is performed using a tuning fork and can help to distinguish between conductive and sensorineural hearing loss.

A 25-year-old man is brought in the the Emergency Department by police for bizarre behavior. His family arrives and states that his behavior changed about eight months ago. He does not abuse drugs or alcohol. On examination, he has a flat affect and appears to be responding to internal stimuli. His speech is disorganized and he says, "I've been running from the Russians who are trying to steal my thoughts." His laboratory and urine studies are unremarkable. What's the most likely diagnosis? Schizoaffective disorder Schizophrenia Schizophreniform disorder Schizotypal personality disorder

Correct Answer ( B ) Explanation: The most common psychotic disorder is schizophrenia (meaning "split mind"), with a prevalence of up to 1% of the population. Symptom onset is usually in the late teens to mid-30s. Males are slightly more affected than females, and there appears to be a higher incidence among migrants, urban populations and ethnic minorities. The diagnostic criteria for schizophrenia include symptoms lasting at least six months and include delusions or disorganized speech and one of the following: hallucinations, negative symptoms, or grossly negative or catatonic behavior. Negative symptoms include a flat affect, decreased motivation, anhedonia, decreased emotional expression, decreased social interaction, and decreased speech. Additionally, the symptoms may not be attributed to an underlying medical or psychiatric condition or a substance. The lifetime risk of suicide is 5%. Many other psychotic disorders mimic schizophrenia, including schizoaffective disorder, schizotypal personality disorder, schizophreniform disorder, brief psychotic disorder, and delusional disorder. However, these conditions are variable in their diagnostic specificity and duration of symptoms, as defined by the fifth edition of the Diagnostic and Statistical Manual of Mental Disorders (DSM-V). Emergency department management may include keeping the patient free from harming themselves or others, addressing any aggressive or agitated behavior, psychiatric consultation, and administration of antipsychotic medications (e.g., haloperidol) for rapid tranquilization.

A 32-year-old man presents with severe, acute left knee pain that occurred while wrestling with his brother. He is unable to bear weight. His knee is held in flexion and he is unable to actively extend it. His leg is shown above. Which of the following is the next best step in management? Angiogram Closed reduction Computed tomography scan Orthopedic consultion

Correct Answer ( B ) Explanation: The patient has a lateral patella dislocation. Closed reduction is the management of a patellar dislocation. The hip is flexed and the knee is passively extended while simultaneous medial stress is applied to reduce the laterally displaced patella to its anatomic location. Pain relief is usually immediate, but the patient may continue to be sore. Patellar dislocations must not be confused with knee dislocations, which are more severe. Patellar dislocations are usually caused by tibial rotation against a flexed knee during quadriceps contraction. The patella is most commonly displaced laterally. The hip and knee are classically flexed at 90 degrees. Once closed reduction is successful, an X-ray should be performed to rule out fractures or other abnormalities and an knee immobilizer placed. Management is with rest and orthopedic follow up. An angiogram (A) should be obtained in patients with suspected knee dislocations (not patellar dislocations). The most feared complication from a knee dislocation is a popliteal artery injury. These injuries often present after relocation and may appear clinically normal. However, if vascular repair does not occur within 6 to 8 hours of injury, permanent ischemia and limb loss may ensue.

27-year-old previously healthy man comes to the ED with chest pain. He had a viral syndrome with low-grade fever a few days earlier. He describes the chest pain as sharp, retrosternal in location, with radiation to the left trapezius ridge. It improves with sitting forward and worsens when he lays down. Vital signs are normal. His electrocardiogram is shown above. Which of the following treatments is most appropriate? Administration of aspirin and emergent cardiac catheterization Administration of ibuprofen and discharge home Hospital admission for cardiac monitoring and serial cardiac enzymes Prescription for azithromycin and discharge home

Correct Answer ( B ) Explanation: The patient has acute pericarditis. Pericarditis refers to inflammation of the pericardial sac. Classic symptoms include precordial or retrosternal sharp and stabbing chest pain, sometime with radiation to the trapezius ridge or left arm. Patients may report that pain is worsened in the supine position and improved by sitting up and leaning forward. Associated symptoms may include low grade fever, dyspnea, and cough. A pericardial friction rub is specific for pericarditis, but often absent. Characteristic ECG changes of pericarditis include diffuse ST segment elevation and PR segment depression. Pericarditis can be caused by infections (viral, bacterial, and fungal), systemic inflammatory disorders (rheumatoid arthritis, systemic lupus erythematosus, scleroderma, etc.), uremia, malignancies (particularly leukemia, lymphoma, and metastatic breast and lung cancer), or iatrogenic or idiopathic. The treatment of pericarditis depends on the underlying cause. When viral infection is suspected, treatment is supportive with nonsteroidal antiinflammatory medications. Therefore, administration of ibuprofen and discharge home is

A 51-year-old man with a history of diabetes mellitus and hypertension presents with approximately 60 minutes of left-sided jaw pain that began while he was shoveling snow. Which of the following tests should be ordered? Computed tomography scan of the maxillofacial region Electrocardiogram Electrolyte sedimentary rate Panorex X-rays

Correct Answer ( B ) Explanation: The patient has jaw pain that began during exertion. This presentation is concerning for acute coronary syndrome (ACS) and the patient should have an immediate electrocardiogram (ECG) performed. Acute coronary syndrome is a constellation of signs and symptoms that occurs when there is an imbalance between myocardial oxygen demand and supply. The most common cause of acute coronary syndrome is disruption of an atherosclerotic plaque, which results in platelet aggregation or thrombus formation at the site of the plaque, blocking or reducing coronary blood flow through the affected vessel. Coronary arterial spasm and factors extrinsic to the coronary arteries (e.g. hypotension, increased myocardial demand from tachycardia or thyrotoxicosis) can also cause acute coronary syndrome. The most common symptoms of acute coronary syndrome is chest pain, often described as discomfort, pressure, tightness or heaviness. Associated symptoms are common, including diaphoresis, nausea, vomiting, dyspnea, lightheadedness, and syncope. Symptoms triggered by exercise or exertion, stress, and a cold environment are also concerning for acute coronary syndrome. Additional features, which are predictive of acute coronary syndrome include radiation to the jaw, right arm, left arm, or both arms. It is important to be aware that some patients, particularly the elderly, women, and patients with diabetes, may not present with classic ACS symptoms and may instead complain of generalized weakness, fatigue, epigastric discomfort or "indigestion," and nausea. Providers should have a low threshold to evaluate for ACS in these patients with atypical symptoms.

46-year-old man with a history of diabetes mellitus and chronic alcoholism presents to the Emergency Department with lower back pain that has progressively worsened over the last five days. He denies history of trauma. On examination, he has a temperature 37.9oC, heart rate 87 beats/minute, and blood pressure 146/75 mm Hg. He has tenderness with percussion over the midline lower lumbar spine. Achilles and patellar reflexes are intact. He has mild decreased sensation of both feet, but his strength is intact. What is the next best step in management? Complete blood count Magnetic resonance imaging of the lumbosacral spine Pain control with ibuprofen and discharge home Plain radiographs of the lumbosacral spine

Correct Answer ( B ) Explanation: This patient has signs and symptoms concerning for a spinal epidural abscess. High risk patients include intravenous drug users, alcoholics, and immunocompromised patients (e.g. history of diabetes mellitus). Recent instrumentation and trauma are also risk factors. Patients present with lower back pain and often have tenderness with percussion over the spinal processes. Fever is present in only 50% of patients. Focal neurologic deficits can also be present but, again, are lacking in nearly half of patients. Given the potentially subtle presentations and the fact that twenty percent of cases occur in patients who are not considered high risk, a high index of suspicion is needed to avoid missing this critical diagnosis. Emergent MRI of the lumbosacral spine is the modality of choice for diagnosing a spinal abscess. A CT myelogram can be done in patients who have contraindications to MRI. Treatment includes emergent neurosurgical consultation for drainage and decompression in addition to broad spectrum antibiotics.

A 23-year-old man with a history of insulin-dependent diabetes presents to the ED with vomiting and shortness of breath. He recently became homeless and has not been using his insulin. On physical exam, he has a fruity odor to his breath. After two hours of treatment in the ED, his serum labs include: glucose 220 mg/dL, sodium 133 mEq/L, chloride 101 mEq/L, and bicarbonate 12 mmol/L. Which of the following is the most appropriate next step in management? Allow the patient to eat a turkey sandwich Change to dextrose-containing intravenous fluid Discharge home with endocrinology follow-up Transition to subcutaneous insulin therapy

Correct Answer ( B ) Explanation: This patient is exhibiting signs and symptoms consistent with diabetic ketoacidosis (DKA). DKA is more common in insulin-dependent diabetics. Management of DKA includes cardiac monitoring, aggressive intravenous fluid hydration, and insulin administration most often in the form of an insulin drip. Therapy should be continued until the anion gap is closed; however, this may take longer to achieve than the normalization of the serum glucose level. Once the glucose level is < 250 mg/dL, glucose-containing fluids should be initiated to prevent hypoglycemia. Subcutaneous insulin will need to be given prior to discontinuation of intravenous insulin therapy. Potassium should be supplemented as well if the level is < 5 mEq/L. Electrolytes and blood gases should be checked every 2 hours to monitor progress during treatment. DKA is most commonly the result of noncompliance with insulin therapy. Signs and symptoms of DKA include tachycardia, altered mental status, tachypnea with deep respirations (Kussmaul breathing) in response to acidosis, dry mucous membranes, abdominal pain, blurry vision, and some may smell a fruity odor to the breath from acidosis. The diagnosis of DKA involves several laboratory abnormalities including glucose levels usually > 300 mg/dL, an anion gap metabolic acidosis, glucosuria and ketonuria on urinalysis, and elevated serum β-hydroxybutyrate. The serum potassium level may be falsely elevated as patients are total body potassium depleted; however, it may also be normal or low

A 46-year-old woman without significant past medical history presents with drooping of the left side of her face which she noticed this morning when she looked into the mirror. She is unable to completely close her left eye and cannot raise her eyebrows on the left side. Her vision is intact. When asked to puff out her cheeks, air leaks out the left side. The rest of her neurologic exam is normal. Which of the following is the most appropriate initial management? CT head followed by administration of tPA Discharge home with prednisone and taping of the eye shut at night Magnetic resonance imaging of the brain Stat neurology consult for neurointerventional procedure

Correct Answer ( B ) Explanation: Weakness of the facial muscles is a sign of Bell's palsy, which refers to unilateral cranial nerve VII weakness. The most important priority in evaluating a patient with facial weakness is ruling out a central cause such as stroke or hemorrhage. Symptoms of Bell's palsy may include ipsilateral facial weakness, drooling, loss of taste sensation, ipsilateral tongue numbness, and ear or retroauricular pain. Preserved ability to raise the eyebrows suggests a central process. This is because the forehead receives bilateral upper motor neuron innervation, so a central stroke will spare the forehead and allow the patient to raise the eyebrows. The cause of Bell's palsy is idiopathic in most cases, but some studies point to a correlation between viral infection, particularly HSV, and the development of Bell's palsy. When Bell's palsy occurs in the context of a facial varicella zoster infection, it is termed Ramsay-Hunt syndrome. The treatment of Bell's palsy includes prednisone and eye care, such as liberal use of a lubricating ointment during the day and taping of the eye shut at night to prevent corneal drying. When the clinical presentation is severe, valacyclovir is can be added to the treatment regimen, although benefit is controversial.

64-year-old woman is brought to the emergency room by ambulance after being the restrained driver in a motor vehicle collision. She fell asleep at the wheel, crossed the highway median, and hit another car head-on at 60 miles per hour. There was a fatality in the other vehicle. On examination, the patient is alert and screaming in pain. Her exam is notable for absent shoulder shrug, 0/5 strength of the upper extremities and 5/5 strength of the lower extremities. Her X-ray reveals anterior displacement of the occiput relative to the atlas. Which of the following is the most likely diagnosis? AAtlanto-axial dislocation BAtlanto-occipital dissociation CTranslational fracture-dislocation DTraumatic spondylolisthesis

Correct Answer ( B ) Explanation: Atlanto-occipital dislocation, commonly referred to as internal decapitation, is a highly unstable and severe ligamentous injury at the craniocervical junction. It is often immediately fatal, but, more recently, more patients have been surviving due to advances in prehospital medicine. Atlanto-occipital dislocation can be caused by a variety of different traumatic mechanisms that transmit excessive force to the craniocervical junction leading to ligamentous disruption. The most common mechanism of injury is high-speed motor vehicle collision. The classic presentation is cruciate paralysis, with paralysis of the upper extremities and sparing of the lower extremities. However, atlanto-occipital dislocation can present with a wide spectrum of symptoms, ranging from isolated neck pain to lower cranial nerve deficits, unilateral or bilateral weakness, and quadriplegia. Spinal cord injury is relatively rare but when present is often immediately fatal due to brainstem injury. Diagnosis can be made by computed tomography or X-ray and is based on abnormalities in the measurements of the basion-dental interval and the basion-atlantal interval. Management of atlanto-occipital dislocation in the field and the emergency department includes confirmation of a stable airway, hemodynamic stabilization, and application of rigid cervical collar. Definitive management is through halo placement.

An 18-year-old man presents after being stabbed in the back. His vital signs are stable and his FAST examination is negative. Physical examination demonstrates a stab wound to the lower back located just to the left of the midline. The patient is unable to move his left leg and reports that his right leg is numb. Which of the following describes his syndrome? Anterior cord syndrome Brown-Séquard syndrome Central cord syndrome Spinal shock

Correct Answer ( B ) Explanation: Brown-Séquard syndrome is a hemisection of the spinal cord most commonly occurring after a penetrating injury. In this syndrome, patients have proprioception and vibratory sensory loss on the ipsilateral side of the injury as well as loss of motor function on the same side. The contralateral side loses pain and temperature sensation since these fibers cross the spinal cord at the level of their nerve roots. The anterior cord syndrome (A) most commonly occurs from a contusion of the spinal cord after a hyperflexion injury. It may also be called by injury or occlusion of the anterior spinal artery. In this syndrome, patients lose motor function and have decreased pain and temperature distal to the lesion. The components of the posterior cord (position, touch and vibratory sensation) are maintained. Central cord syndrome (C) is the most common incomplete spinal cord syndrome seen after hyperextension injuries. The central area of the cord contains components of the pyramidal and spinothalamic tracts. The upper extremities are affected more than the lower extremities because distal parts of the body are located more peripherally in the cord. Spinal shock (D) involves total loss of function distal to the point of injury due to a contusion or concussive injury to the entire spinal cord.

A 45-year-old man is brought to the emergency department after being the unrestrained passenger in a motor vehicle collision. His vital signs on arrival are T 37.3°C, HR 90, BP 140/94, RR 23. His physical exam is significant for a GCS of E3V4M5 and a large left temporal scalp laceration. CT imaging of his head reveals a left temporal linear skull fracture under the area of the laceration with no signs of intracranial hematoma formation. Which of the following therapies is indicated in this patient? Hypertonic saline Intravenous antibiotics Mannitol Nicardipine

Correct Answer ( B ) Explanation: Linear skull fractures are single fractures that extend through the thickness of the calvarium. Linear skull fractures may or may not be associated with underlying trauma to the brain. Fractures that cross the path of the middle meningeal artery or a major venous sinus may cause significant bleeding. A linear fracture of the skull with an overlying scalp laceration is considered open. Patients with open skull fractures should receive antibiotic prophylaxis with vancomycin and ceftriaxone. This is also true of patients with skull fractures that are depressed, cross a sinus, or are associated with pneumocephalus.

A 34-year-old woman presents with elbow pain and diminished arm movement following a fall on her outstretched hand. Initial history and physical exam are notable for pain, swelling, and tenderness over the lateral elbow, and inability to fully extend the elbow. Which of the following is indicative of a radial head fracture? ADisplacement of the radiocapitellar line BPosterior fat pad sign CPronator sign DWrist drop

Correct Answer ( B ) Explanation: Patients with radial head fractures typically present with localized swelling, tenderness, and decreased motion of the elbow following a fall on an outstretched hand. These fractures account for approximately 50% of elbow fractures in adults. They may be subtle and are easily missed on plain films. Presence of an elbow joint effusion, as indicated by elevation of the anterior or posterior fat pads or both, points to an occult intra-articular fracture. The posterior fat pad is not seen on a normal lateral X-ray of the elbow and thus is a more reliable indicator of joint effusion. The anterior fat pad can be visible as a thin translucent line on a normal elbow radiograph, but may bulge away from the joint in the setting of an effusion creating the anterior 'sail sign'. Tenderness of the lateral elbow combined with a positive fat pad sign indicate the injury should be treated as a radial head fracture.

Which of the following is feature of the injury shown above? AHematoma formation will not cross cranial suture lines BIt is often associated with concurrent brain injury and parenchymal damage CIt most commonly results from an arterial bleed DRisk for this injury decreases with decreasing age

Correct Answer ( B ) Explanation: Subdural hematomas are caused by tearing of bridging dural veins due to sudden acceleration-deceleration. This results in accumulation of blood in the space between the dura mater and arachnoid mater. Extensive atrophy of the brain, as seen in the elderly and alcoholics, is a major risk factor, as it allows for generation of more force from sudden acceleration-deceleration. Other risk factors include age < 2 years, low cerebrospinal fluid volume, prior head trauma, and anticoagulation. While subdural hematomas accumulate more slowly due to the venous nature of the bleed, subdural hematomas are more likely to be associated with concurrent brain injury and parenchymal damage. There is no specific clinical syndrome associated with SDH and presentations can range from asymptomatic to altered mental status and death. Because hematoma formation occurs between the dura and arachnoid, subdural bleeds readily cross cranial suture lines. Subdural hematomas can be acute, subacute or chronic. Radiographic findings include a convex, "crescent shaped" lesion that crosses suture lines on non-contrast CT. CT may also be able to help identify the chronicity of a SDH, with more acute hematomas appearing hyperdense (white) and more chronic hematomas appearing isodense.

Which of the following is the most sensitive finding in cauda equina syndrome? Back pain Saddle anesthesia Urinary incontinence Urinary retention

Correct Answer ( D ) Explanation: Cauda equina syndrome refers to a characteristic pattern of neuromuscular and urogenital symptoms resulting from the simultaneous compression of multiple lumbosacral nerve roots below the level of the conus medullaris. Midline rupture of the L4/L5 disk is the most common cause. Other causes include tumors, trauma, or spinal epidural abscesses. The presence of urinary retention is the most sensitive finding (90% sensitivity). A postvoid residual of greater than 100-200 ml should alert the emergency physician to the likelihood of this syndrome in the right clinical setting. Lower motor neuron findings may be present, in contrast to upper motor neuron findings, which suggest conus medullaris syndrome (a true cord syndrome). MRI is the diagnostic modality of choice. Cauda equina syndrome is a neurosurgical emergency that requires operative decompression

A five-year-old boy is brought to the emergency department after being found unresponsive at home. He was found lying on the floor in his mother's room with prescription medications scattered all over. His mother called 911, and he was immediately rushed to the hospital by ambulance. On examination, the boy is sedated with a heart rate of 69 beats per minute, respiratory rate of 15 per minute, blood pressure 70/50 mm Hg, pulse oximetry of 99 percent, pupils 1-2 mm reactive to light, and 1+ reflexes on all extremities. Blood sugar is 200 and ECG shows QTc interval prolongation. Which of the following is the most likely medication ingested? Clonidine Methadone Propranolol Salicylate

Correct Answer ( B ) Explanation: The boy has signs and symptoms consistent with opioid ingestion. Methadone ingestion can manifest with the classic opioid toxidrome of respiratory depression, sedation, and miosis. Signs of more severe toxicity can include bradycardia, hypotension, and hypothermia. Methadone has been associated with a prolonged QTc interval and risk of torsades de pointes. Patients with significant respiratory or CNS depression should be treated with naloxone, which is a mu receptor antagonist. Because the half-life of methadone is longer than naloxone, patients can require multiple doses of naloxone. Also, serial ECGs are needed to monitor for the development of a prolonged QTc interval. If a patient does develop a prolonged QTc, management includes close cardiac monitoring, repletion of electrolytes, and having magnesium readily available should the patient develop torsades de pointes. Clonidine (A) toxicity manifests as lethargy, miosis, and bradycardia. Although, findings may be similar to opioid overdose, QTc interval prolongation and torsades de pointes are typically only seen with opioid overdose. Propanolol (C) toxicity causes bradycardia and hypotension that typically develops within six hours of ingestion. Heart block and hypoglycemia may also be seen. Clinical manifestations of salicylate (D) toxicity include nausea, vomiting, diaphoresis, and tinnitus. Moderate cases can manifest as tachypnea, tachycardia, and altered mental status.

You are called to the emergency department to examine a two-year-old girl for possible ingestion. She was accompanied by her grandparents who found her with an opened bottle of pills. There were two pills found in her mouth although the caretakers are not certain as to how many pills are missing. In transit to the ED, the girl was asymptomatic. Your examination at the ED reveals an alert girl with bradycardia, hypotension, normal pupillary size and reaction. You perform an ECG that shows sinus bradycardia. Blood glucose is normal. Which of the following medications could the girl most likely have ingested? Clonidine Metoprolol Nortriptyline Phenobarbital

Correct Answer ( B ) Explanation: The findings on the girl are most consistent with beta-blocker toxicity like metoprolol. Beta-blockers competitively inhibit the action of catecholamines at the β receptor. Toxicity results in decreased chronotropy and inotropy in addition to slowing conduction through AV nodal tissue. These effects are manifested as bradycardia, hypotension, and heart block. Patients with reactive airways disease can experience bronchospasm due to blockade of β2-mediated bronchodilation. Beta-blockers interfere with glycogenolysis and gluconeogenesis, which can lead to hypoglycemia. Evaluation after an overdose should include an ECG and frequent reassessments of hemodynamic status. Blood glucose should be measured in all patients as hypoglycemia may be seen. Treatment involves supportive care and gastrointestinal decontamination as indicated. Glucagon and high-dose insulin are the the antidotes of choice for beta-blocker toxicity.

A 7-year-old male presents to the Emergency Department with an ankle injury. He fell while on equipment at a nearby playground. There is pain, swelling, and ecchymosis over the medial malleolus. An ankle X-ray is obtained and is shown above. Which of the following is the most likely diagnosis? Greenstick fracture Salter-Harris type II fracture Salter-Harris type III fracture Torus fracture

Correct Answer ( C ) Explanation: The Salter-Harris classification is used to describe pediatric injuries involving the physis, the cartilaginous epiphyseal growth plate at the end of long bones. There are five types of fractures in this classification, and management depends on the type and severity of the fracture. Salter-Harris type III fractures are intra-articular injuries that involve the physis and the epiphysis. These are often unstable fractures and are managed based on size and degree of displacement. Consultation with an orthopedic surgeon is recommended.

A 17-year-old high school football player is seen in your clinic with neck pain after practice. He reports the pain started after he tackled a teammate and was associated with tingling in both arms. What is next best step in management? AAnalgesia and observation BCervical spine immobilization CNeurosurgical referral DX-ray of cervical spine

Correct Answer ( B ) Explanation: This patient is reporting neck pain and neurological symptoms after football practice, which is concerning for a cervical spinal cord injury. Initially, the patient's spine should be immobilized while simultaneously assessing the airway, breathing, and circulation (ABCs). If the injury occurs on the football field, the helmet and shoulder pads should not be removed to ensure spine stabilization. Initial immobilization should be done by stabilizing the head and neck with two hands. If the player is not supine, he should be log-rolled into supine position for the initial assessment. If there is airway compromise, the face mask should be removed. In a clinically stable patient immobilization is done by placing the patient in supine position and placing the neck in a rigid cervical collar. The most common mechanism of injury in adolescents is via axial loading, which can cause a cervical fracture of C1 (Jefferson fracture). This is most likely to occur in football, when a player runs headfirst into another player, and diving, when a diver's head hits the bottom of the pool.

A 17-year-old baseball player presents to the emergency department complaining of sudden left shoulder pain. The patient states he heard his shoulder "pop" and felt pain as he was extending his arm back to throw a fastball. On examination, the patient is holding his left shoulder in abduction and external rotation with loss of the deltoid contour. There is limited and painful range of motion. Which of the following is the most likely diagnosis? Acromioclavicular joint injury Anterior glenohumeral dislocation of left shoulder Midshaft fracture of the left humerus Posterior glenohumeral dislocation of the left shoulder

Correct Answer ( B ) Explanation: The patient suffered an anterior glenohumeral dislocation of his left shoulder. The shoulder is the most commonly dislocated joint in the body. Dislocations can occur from a trauma (95%) or from laxity of the ligaments (5%). The glenohumeral ligaments, the rotator cuff muscles, and the joint capsule or labrum stabilizes the shoulder. The shoulder joint is often injured during a fall on an outstretched arm, engaging in sports activities, a direct blow to the joint, or grand mal seizures. Patients often report hearing or feeling a pop during the incident and present with significant pain, limited range of motion, swelling, or deformity. Anterior shoulder dislocations are the most common type; however, they can occur posteriorly and inferiorly. Anterior dislocations result from a force applied to the shoulder while the arm is abducted and externally rotated such as while throwing a ball. On examination, the arm is held in abduction and external rotation (down adjacent to the body) with a loss of the deltoid contour when compared to the other shoulder, and the humeral head can often be palpated anteriorly beneath the clavicle. The patient will have very limited range of motion and be unable to touch the opposite shoulder. Obtaining pre-reduction and post-reduction radiographs and neurovascular examinations are imperative to successful management. Complications of shoulder dislocations include fractures (e.g. Hill-Sachs lesion), soft tissue injury (e.g. Bankart lesion), nerve injury (e.g. axillary nerve), or vascular injury (e.g. axillary artery).

A 16-year old boy presents with a superficial bite to his right arm. He states he was camping in the woods and a bat flew into his tent. Which of the following treatments should this patient receive? Human rabies immunoglobulin and 3 doses of inactivated rabies vaccine over 7 days Human rabies immunoglobulin and 4 doses of inactivated rabies vaccine over 14 days Human rabies immunoglobulin only Inactivated rabies vaccine only

Correct Answer ( B ) Explanation: The rabies virus enters the CNS and causes a wide range of symptoms from headaches, anorexia, hallucinations, agitation to seizures, and hydrophobia. Bats are the most common animals to infect humans. Raccoons are the most common animal infected with rabies. The patient's history and presentation suggest an infection with the rabies virus and post-exposure prophylaxis with the inactivated rabies vaccine and the human rabies immunoglobulin should be administered. The human rabies immunoglobulin is a one time dose where as the inactivated rabies vaccine is given 4 times over 14 days. In addition the bite wound should be examined and cleaned and antibiotics may be of benefit.

Which of the following laboratory findings suggests ethylene glycol ingestion? Anion gap 16 and pH 7.32 associated with vision changes Lactate 0.9 mmol/L, pH 7.12, and creatinine 2.5 mg/dL Osmol gap 20, anion gap 8, and ketonemia pH 7.10, lactic acid 5.5 mmol/L, and anion gap 21

Correct Answer ( B ) Explanation: The toxic alcohols include ethanol, ethylene glycol, methanol, and isopropanol. Each alcoholic overdose has its own clinical characteristic. Markers of intoxication with ethylene glycol include high anion gap metabolic acidosis with an absence of significant lactate or ketone concentrations, calcium oxalate formation leading to acute renal failure. Therefore, lactate 0.9 mmol/L, pH 7.12, and creatinine 2.5 mg/dL would be characteristic of ethylene glycol poisoning; although, lactate can be slightly elevated (both falsely or truly), but not at levels that would explain the the significant acidosis. Sodium bicarbonate may be used for urine alkalinization to improve acidemia. Ethanol or fomepizole may be utilized for acute management in conjunction with a nephrology consultation. Hemodialysis is indicated for ethylene glycol levels > 25 mg/dL. Adjunctive therapy includes thiamine and pyridoxine every 4 to 6 hours.

Which of the following is a known complication of chronic lithium toxicity? Diabetes mellitus Hypoparathyroidism Nephrogenic diabetes insipidus Syndrome of inappropriate antidiuretic hormone

Correct Answer ( C ) Explanation: Lithium toxicity can present in the setting of acute, acute-on-chronic and chronic ingestions. Chronic lithium ingestion may cause nephrogenic diabetes insipidus, a concentrating defect in the kidneys due to resistance of antidiuretic hormone. This results in dilute urine output and development of hypernatremia. Acute onset of nocturia is often the first sign of nephrogenic diabetes insipidus. Patients develop polyuria and polydipsia to compensate for the excess fluid loss. However, if patients are unable to keep up with the free water losses, either due to illness or mental disability, dehydration and elevated lithium levels may result. In addition to dehydration, certain medications such as nonsteroidal anti-inflammatories and angiotensin-converting enzyme inhibitors cause a decrease in lithium elimination. In contrast to patients with acute lithium ingestions, patients with chronic ingestions often initially present with neurologic signs such as ataxia, lethargy, neuromuscular excitability (e.g. tremors, clonus) and seizures

A 20-year-old man presents to an urgent care clinic complaining of right foot pain after landing wrong while playing basketball. On physical exam, he has maximal tenderness over the lateral foot. A radiograph is obtained and shown above. Which of the following is the most likely diagnosis? Bennett fracture Jones fracture Rolando fracture Smith fracture

Correct Answer ( B ) Explanation: This man has a Jones fracture. A Jones fracture is a transverse fracture at the fifth metatarsal base, 1.5-3 cm distal to the proximal tuberosity. The most common mechanism of injury is significant adduction while the foot is plantar flexed. This mechanism may occur when an athlete lands awkwardly after jumping or when the ankle is inverted while running. Different portions of the proximal aspect of the fifth metatarsal have blood supply variations that contribute to differences in healing. The tuberosity receives blood supply from multiple metaphyseal arteries and from branches of the nutrient artery. The proximal diaphysis relies solely on the nutrient artery for blood supply, thereby increasing the likelihood for blood supply disruption and poor healing. Because most Jones fractures occur with ankle inversion, most patient present complaining of a sprained ankle. Applying the Ottawa ankle rules has been shown to detect the majority of Jones fractures, while reducing unnecessary radiographs. Three-view radiographs should be obtained in all patients meet the Ottawa rules. Initial treatment involves immobilization with a posterior splint, nonweightbearing status, and orthopedic referral in 3-5 days. Adjunctive therapies should include icing the area, elevation, and analgesics. Diaphyseal fractures often require surgical fixation.

A 19-year-old man presents to the ED with facial and mouth pain after being assaulted with a club. On physical exam, you note significant swelling over the left inferior and lateral face and misalignment of the inferior premolars. Which of the following is the most common anatomic location of this traumatic injury? AAngle of mandible BBody of mandible CMandibular condyle DMental foramen

Correct Answer ( B ) Explanation: This patient has signs and symptoms consistent with a mandibular fracture. The mandible is the second most commonly fractured facial bone behind the nasal bone. The most common fracture location is the body of the mandible, which is more likely to show signs of obvious tooth misalignment on exam. Other signs and symptoms of a mandibular fracture include trismus, malocclusion of the bite, intraoral lacerations, and missing teeth. The tongue blade test is a way of detecting even subtle mandibular fractures. The patient bites the blade while the examiner attempts to break it. Inability to stabilize the blade is an indication that the patient may have a mandibular fracture and should undergo imaging. Plain mandibular X-rays and panoramic radiographs are screening tests for mandibular fractures; however, CT of the facial bones is the gold diagnostic standard. Management is variable depending upon location and whether the fracture is open or closed. Open fractures require IV antibiotics, such as penicillin or clindamycin, and hospital admission. Closed fractures may be discharged with dental follow-up. The patient should be given adequate oral analgesics and instructed to observe a soft diet and chew on the non-fractured side. Many fractures will require open reduction and internal fixation, but do not require admission initially if not open fractures.

A 19-year-old migrant worker presents to the ED with vomiting, diarrhea, diaphoresis, wheezing, and excessive tearing. Vital signs are BP 150/100 mm Hg, HR 36 beats per minute, RR 28 breaths per minute, and T 98.6°F. Which of the following is the most appropriate initial step in management? Atropine Decontamination Endotracheal intubation High-dose insulin

Correct Answer ( B ) Explanation: This patient is exhibiting signs and symptoms consistent with cholinergic toxicity secondary to organophosphate poisoning. Management of organophosphate poisoning should begin with decontamination. Anyone coming into contact with the patient should wear a gown and gloves to prevent transmission of the toxin. Clothing should be removed and discarded in a well-ventilated area and the patient should be thoroughly washed including ocular irrigation. The patient should then be resuscitated in a similar manner to any other ED patient. These patients often require early intubation and aggressive IV fluid resuscitation. There are two keys to definitive treatment in patients with organophosphate poisoning. The first is to temporize the life-threatening signs and symptoms of cholinergic toxicity. Atropine is a competitive inhibitor of acetylcholine at muscarinic receptors. The goal of treatment with atropine is to titrate to the drying of bronchial secretions. Pralidoxime (2-PAM) is the definitive antidote to organophosphate poisoning. 2-PAM forms a complex with the bound acetylcholinesterase enzyme to cause the release of the organophosphate from the enzyme. This results in regeneration of its ability to metabolize acetylcholine. Organophosphates bind tightly to acetylcholinesterase preventing the breakdown of acetylcholine. Signs and symptoms of organophosphate poisoning relate to excess acetylcholine at the nicotinic and muscarinic receptors. This produces the cholinergic toxidrome due to high postsynaptic parasympathetic activity, resulting in the classic SLUDGE mnemonic: Salivation, Lacrimation, Urination, Diarrhea, GI cramps and Emesis. Patients will also demonstrate diaphoresis, pupillary miosis, bradycardia, muscular fasciculations, paralysis, agitation, seizures, or even coma.

What is the most common pathogen found in infected cat bites? Bartonella henselae Pasteurella multocida Staphylococcus aureus Streptococcus pyogenes

Correct Answer ( B ) Explanation: While only approximately 5% of dog bites become infected, a significant number of cat bites will become infected. Some studies have found up to 70% of cat bites become infected although this may be an overestimation given that many patients only seek treatment after an infection has developed. Cats have narrower, smaller teeth, resulting in most bites being deep puncture wounds. Cat bites are also more likely to be found on the hands which increases the risk of infection as well. Pathogens responsible for infection include those found in the oral flora of the cat as well as skin flora of the patient. The most common pathogen found in infected cat bites is Pasteurella multocida, a virulent facultatively anaerobic gram negative rod. Infection from Pasteurella typically occurs within 24 hours (earlier than cellulitis due to other pathogens) with erythema, warmth, swelling, and pain on examination. Purulent drainage is often seen as well. Prophylaxis with a 3-5 day course of amoxicillin-clavulanate should be prescribed to all patients who present with a cat bite. For those who present with wounds already infected, treatment options include ampicillin-sulbactam, a third-generation cephalosporin, or a fluoroquinolone.

55-year-old woman presents with right finger pain. She admits to habitually biting her nails. On examination, the lateral nail fold and cuticle of the right ring finger is erythematous, fluctuant and tender to palpation. Which of the following is the most likely diagnosis? Felon Flexor tenosynovitis Paronychia Tinea unguium

Correct Answer ( C ) Explanation: A paronychia is an infection of the lateral nail fold or paronychium and is the most common hand infection. Most cases are caused by minor trauma such as nail biting, manicures, or hangnails. Most are polymicrobial with Staphylococcus aureus and Streptococcal species being the most common aerobic organisms. Chronic paronychias may occur in immunocompromised people and may contain fungal organisms such as Candida albicans. Paronychias usually begin as cellulitis but often progress to a frank abscess. Management may or may not require incision and drainage depending on the presence or absence of fluctuance or identifiable pus. If no pus is present, warm soaks, elevation and oral antibiotics are recommended. If visible pus or fluctuance is present, the hand should be soaked in warm water and the lateral nail fold should be elevated to release the pus. Incision and drainage with a No. 11 blade scalpel may be necessary for severe cases. If more extensive debridement is required, a digital block may be needed to facilitate proper analgesia. Felon (A) is a localized suppurative infection of the pulp space of the distal finger or thumb. It does not usually involve the nail or cuticle, but may be a complication of a paronychia. Infectious flexor tenosynovitis (B) is an infection of the flexor tendon sheath characterized by the four Kanavel signs: fusiform swelling of the digit, pain with passive extension, tenderness along the tendon sheath and a finger held in a flexed position at rest. Tinea unguium (D) is a fungal nail infection, or onychomycosis, that affects the toenails more often than the fingernails.

A 23-year old professional soccer player collapses during a match. He is pulseless, and cardiopulmonary resuscitation is initiated. Which of the following is the most common cause of sudden death in competitive athletes? Arrhythmogenic right ventricular dysplasia Dilated cardiomyopathy Hypertrophic cardiomyopathy Long QT syndrome

Correct Answer ( C ) Explanation: Cardiomyopathies are a heterogeneous group of disorders in which there is abnormal cardiac structure, impaired myocardial function, or altered myocardial electrical activity. Hypertrophic cardiomyopathy is most common cause of sudden cardiac death in competitive athletes. The prevalence in the general population is estimated to be 1 in 500. In hypertrophic cardiomyopathy, patients have asymmetric hypertrophy that primarily involves the intraventricular septum. This results in reduced compliance of the hypertrophied ventricle, impaired diastolic relaxation and restricted left ventricular filling. Although many patients are asymptomatic, the most frequent complaint is dyspnea on exertion. Additional symptoms include chest pain, decreased exercise tolerance, and syncope. There may be a family history of sudden cardiac death. On physical examination, patients may have a systolic ejection murmur, heard at the lower left sternal border or at the apex, and a fourth heart sound is often present. Characteristic ECG findings include findings of left ventricular hypertrophy and left atrial enlargement. Chest radiographs are usually nonspecific. Diagnosis is confirmed by echocardiography.

Which of the following bone fractures is most commonly involved in acute compartment syndrome? Fibula Radius Tibia Ulna

Correct Answer ( C ) Explanation: Muscles in the human body are divided into compartments lined with fascial membranes. Acute compartment syndrome (ACS) occurs when increased tissue pressure within a compartment results in nerve and muscle ischemia. ACS generally develops after a significant trauma such as a fracture, but may also occur with minor trauma or due to a nontraumatic etiology. The most common sites for ACS to occur are the leg and arm. Long bone fractures are the cause of 75% of cases of ACS, with the tibia being the most common bone involved. ACS is considered to be a surgical emergency. Measurement of compartment pressures assists in determination of the need for fasciotomy, which is the definitive treatment for ACS. Frequent, serial examinations are critical in the management of patients at risk for ACS. Prognosis is good in patients with early diagnosis and appropriate intervention.

A 44-year-old woman presents to the Emergency Department with one week of crampy abdominal pain that is worse after she eats. She denies alcohol consumption or tobacco abuse. On physical examination, she is overweight and has epigastric and right upper quadrant abdominal tenderness to palpation. Which of the following examination findings is most consistent with cholecystitis? Epigastric rebound tenderness Gray-Turner sign Inspiratory arrest with right upper quadrant palpation Right scapular tenderness

Correct Answer ( C ) Explanation: Cholecystitis is an inflammatory process of the gallbladder. The most common cause is an obstructing gallstone. It affects women more than men, and prevalence increases with age and body mass index. Symptoms include epigastric and right upper quadrant abdominal pain that is worse after eating. This is due to gallbladder activation and contraction by cholecystokinin so that bile may be expelled for food digestion. On physical examination, a Murphy's sign may be elicited and is considered positive if the patient demonstrates inspiratory arrest with palpation of his or her right upper quadrant while having him or her take a deep breath. Upon deep inspiration, the diaphragm is contracted and displaces the liver and gallbladder caudally (toward the patient's feet). With deep palpation of the right upper quadrant during this maneuver, the inflamed gallbladder makes contact with the examiner's hand and causes pain, forcing the patient to pause deep inspiration.

A 45-year-old man with a history of alcoholic cirrhosis presents to the Emergency Department for confusion. His family brought him. They deny recent trauma, infection, bloody emesis or stool. He is unable to give a coherent history, but seems pleasantly confused. He denies pain. Family report that he no longer drinks alcohol and has not used any medications or drugs recently. On exam, he is drowsy and delirious. His hands flap when held up, and his tongue gyrates back and forth when protruded. Guaiac testing is negative. His abdomen is distended, but soft and nontender. A noncontrast computed tomography scan of the head shows no evidence of bleeding. His renal function is normal, sodium levels are within normal ranges, liver function tests are within his normal ranges, and blood counts are normal. His ammonia level is elevated at 114 µmol/L. What therapy should be initiated? Albumin Cefotaxime Lactulose Paracentesis

Correct Answer ( C ) Explanation: Hepatic encephalopathy is a poorly understood complication of hepatic disease. Patients present with symptoms ranging from fatigue to coma. The liver is usually active in processing nitrogenous waste, but when that fails and waste products build up encephalopathy ensues. Hepatic encephalopathy is a diagnosis of exclusion, and other etiologies must be sought and eliminated prior to making the diagnosis. Treatment revolves around removing nitrogenous waste from the system, by means of lactulose. Lactulose is a disaccharide that degrades into lactic acid in the colon. The acidified colonic environment traps ammonia and other nitrogenous waste in the colon, allowing for removal. Albumin (A) is an important adjunct when treating ascites with large volume paracentesis. It is recommended that albumin be given for paracentesis > 4 liters. Cefotaxime (B) is used to treat spontaneous bacterial peritonitis (SBP). Although it must always be considered in a liver failure patient, this patient has no abdominal pain, tenderness or fever, and is unlikely suffering from SBP. A therapeutic paracentesis (D) is useful when a patient has respiratory distress due to the ascites in their abdomen.

Which of the following patients presenting to the emergency department with suicidal ideation would be considered highest risk for successful suicide if released from the department? A 25-year-old African American man who reports depression in the setting of a recent break-up with his girlfriend A 35-year-old white woman with a history of schizophrenia who has not been compliant with her medications A 50-year-old white man who works as a physician and has been diagnosed with severe depressive disorder A 70-year-old married, African American woman with cancer who is tired of her chemotherapy and is ready for "it all to end"

Correct Answer ( C ) Explanation: Identification of individuals at short-term risk of suicide is difficult because mortality cuts across all ages, income levels, educational attainment, geographic regions, and state of health. However, particular groups of people have been identified as higher risk for successful suicide attempts and include men, especially those who are above age 45 and white, adolescents, veterans, American Indian teenagers, persons with particular occupations (e.g., agricultural workers, physicians, attorneys, and veterinarians), youths and adults who identify as gay, lesbian or bisexual, and persons with mental illness, disabilities, or chronic pain. Of the patients presented, the 50-year-old white man who is a physician with major depressive disorder is at highest risk for successful suicide. His risk factors include being a white man with age greater than 45 years, a high risk occupation, and severe depression. He carries four risk factors that make him at highest risk for successful suicide.

Which of the following correctly describes electrocardiographic features of second-degree type I atrioventricular block? Absence of discernible P waves Constant PR interval followed by a dropped beat Grouped beats Sawtooth P waves

Correct Answer ( C ) Explanation: In a second-degree AV block, atrial impulses are intermittently conducted through the AV node. There are two types of second-degree block, Mobitz type I and type II. In Mobitz type I block, also known as Wenckebach, there is a progressive delay in each conduction through the AV node until conduction is finally blocked. This is because each atrial depolarization prolongs the refractory period of the AV node until an impulse eventually arrives during the absolute refractory period and cannot be conducted. On the ECG, this is seen as progressive lengthening of each PR interval until a QRS is eventually dropped. The conduction ratio describes the ratio of atrial to ventricular depolarizations. For example, a 4:3 ratio indicates that for every four atrially generated beats, only three are conducted to the ventricles. This results in clustering of QRS complexes separated by a pause from the dropped beat. This is known as grouped beating and is characteristic of second-degree Mobitz type I AV block. Mobitz type I block is often transient, seen in association with inferior wall myocardial ischemia, medication toxicity, and cardiac surgery. It can also be a normal variant. Specific treatment is not usually necessary, unless a very slow ventricular rate products signs of hypoperfusion, in which case atropine can be helpful. The other type of second-degree AV block, Mobitz type II, is characterized by a fixed PR interval with periodic non-conducted impulses. Because Mobitz type II blocks are associated with progression to third-degree, or complete heart block, patients should have transcutaneous pacer pads placed in anticipation of possible need. Absence of discernible P waves (A) is seen in supraventricular tachycardias (SVT). Constant PR interval followed by a dropped beat (B) is characteristic of second-degree Mobitz type II AV block. Saw-tooth P waves (D) are seen in atrial flutter.

A 67-year-old man with a history of hypertension and diabetes mellitus presents to the emergency department with sudden onset left leg weakness. His examination is notable for 3/5 strength in his left lower extremity as well as diminished sensation to temperature and light touch. Which of the following vascular territories is most likely affected in this patient? Left lenticulostriate arteries Left posterior cerebral artery Right anterior cerebral artery Right middle cerebral artery

Correct Answer ( C ) Explanation: Ischemic strokes are the most common type of cerebrovascular accident and can be caused by thrombosis, embolism, and hypoperfusion. Thrombosis is the most common cause of ischemic stroke. Risk factors for ischemic stroke include smoking, increasing age, male gender, diabetes, hypertension, and hyperlipidemia. The middle cerebral artery is the artery most commonly affected by ischemic strokes. Anterior cerebral artery (ACA) infarcts are more rare due to collateral blood flow supplied by the anterior communicating artery. The ACA supplies the frontal, prefrontal, and supplementary motor cortex. Symptoms of an ACA infarction include contralateral lower extremity weakness and sensory loss with sparing of the upper extremity and face. Management of an ACA stroke depends on the timing of presentation, with administration of tPA improving outcomes in eligible candidates. Management of all stroke syndromes should include reduction of risk factors, including optimization of blood sugar, smoking cessation, and control of blood pressure and cholesterol.

Bells Palsy

Patient with a history of viral prodrome Complaining of waking up with unilateral facial nerve paralysis, hyperacusis and taste disturbance PE will show CN VII nerve palsy that does not spare the forehead Most commonly caused by HSV Treatment is prednisone, artificial tears, tape eyelid shut Comments: Bilateral: Lyme disease, infectious mononucleosis

Which of the following causes acute painful loss of vision? Central retinal artery occlusion Central retinal vein occlusion Optic neuritis Retinal detachment

Correct Answer ( C ) Explanation: Optic neuritis, an inflammatory, demyelinating disease of the optic nerve, presents as an acute monocular loss of vision. It is more frequently seen in young women. Although often idiopathic, approximately 30% of patients will develop multiple sclerosis (MS) within five years. Vision loss, most commonly a loss of central vision, and loss of color perception, develops over a period of hours and peaks in about one week. Eye pain, typically worse with eye movements, occurs in about 90% of patient. On examination, an afferent pupillary defect is usually present as is a swollen disk. Diagnosis is made based on history and physical exam findings. Magnetic resonance imaging of the brain and orbits with gadolinium will confirm the diagnosis and is helpful in determining those patients at risk for developing multiple sclerosis. After peak vision loss at one to two weeks, symptoms will gradually improve. Intravenous corticosteroids have been shown to increase the rate of recovery, as well as delay the onset of multiple sclerosis, although long-term visual outcome and rate of developing MS at five years are not affected by this treatment. Central retinal artery occlusion (A) presents with acute, painless loss of vision. On examination, the retina is edematous and the fovea is seen as a cherry-red spot. Central retinal vein occlusion (B) also presents with acute, painless loss of vision. Funduscopic examination reveals hemorrhage described as "blood and thunder". A retinal detachment (D) is typically painless. Patients describe flashing lights and floaters, with vision loss curtain-like in appearance.

A 5-year-old boy presents to the emergency department complaining of right hip pain. He is febrile and refuses to bear weight on the right foot. There is no history of trauma. X-rays are obtained and are unremarkable. Which of the following diagnostic tests would best confirm or exclude a diagnosis of osteomyelitis? Blood culture Complete blood count with differential Erythrocyte sedimentation rate Serum glucose

Correct Answer ( C ) Explanation: Pediatric patients with underlying osteomyelitis almost universally have an elevated erythrocyte sedimentation rate (ESR). An elevated ESR and C-reactive protein have a 98% sensitivity for osteomyelitis in children. The two most common symptoms of osteomyelitis in children are fever and joint pain. Other symptoms include malaise, fatigue, sudden refusal to bear weight on the affected joint and limp. The affected site may be warm, swollen, and painful to the touch. The gold standard in diagnosis is a bone biopsy and culture. Otherwise there is no one single laboratory test that confirms the diagnosis of osteomyelitis. A complete blood cell count may (B) be helpful if the white blood cell count and/or platelets are abnormal. However an elevated white blood cell count is not specific for osteomyelitis and may also be normal despite an underlying infection. Platelets are acute phase reactants and a thrombocytosis may indicate an underlying inflammatory process, however this is also non-specific. An elevated serum glucose (D) may be seen in individuals with a history of diabetes as osteomyelitis can precipitate diabetic ketoacidosis. However, in the general population serum glucose would not be helpful in diagnosing osteomyelitis. Blood culture (A) may be positive for the causative agent, however the result will not be available in a time frame that is useful to the EM physician. X-rays are often unremarkable early in the disease course. CT scan, bone scan, and MRI are often used in the acute phase to reliably diagnose osteomyelitis.

A 24-year-old woman presents to the emergency department with a headache and receives a lumbar puncture. She is eventually discharged home in improved condition, but returns 24 hours later with a worsened headache, now throbbing in nature, which is worse in the standing position and relieved in the supine position. Which of the following reduces the risk of post lumbar puncture headache? Inserting the needle bevel perpendicular to the spine Lying supine for one hour after the lumbar puncture is completed Using a higher gauge needle Using a Quincke needle

Correct Answer ( C ) Explanation: Post-lumbar puncture headache is the most common complication of a lumbar puncture (LP). The headache usually presents within the first 48 hours after LP and tends to be worsened by standing up and improved by lying down. The exact etiology is unclear, but may be related to persistent leak of cerebrospinal fluid from the dural puncture site. Post-LP headache is generally benign and self-limiting within 24 to 48 hours, but can in some cases be debilitating. Bed rest and intravenous caffeine have been shown to improve symptoms, as has epidural blood patching, though the invasiveness of this procedure leads most providers to reserve it only for the most intractable and debilitating cases. Prevention of post-LP headache has been the subject of much research, with mixed and sometimes inconsistent results. Modern evidence-based strategies for prevention include use of a higher gauge spinal needle, orientation of the needle's bevel parallel to the spine, use of a noncutting needle like the pencil-point Whitacre or Sprotte models, and reinsertion of the stylet prior to needle removal.

An 18-month-old boy presents with his father for a cough and difficulty breathing. The father states that the child has had nasal congestion and coryza for the last two days. On exam, he has a barking cough, inspiratory stridor, and a prolonged inspiratory phase. Which of the following would you expect to see on radiographic evaluation? Edema of the epiglottis Intraluminal membranes of the tracheal wall Subglottic narrowing Thickening of the retropharyngeal space

Correct Answer ( C ) Explanation: Subglottic narrowing and distention of the hypopharynx may be seen on chest radiograph or a soft tissue neck radiograph, respectively, in a child with croup. Subglottic narrowing is often called the "steeple sign" since on the posterior to anterior view of the chest, the trachea appears to narrow toward the top and come to a point like a steeple. Croup is typically caused by parainfluenza virus although a number of other viruses as well as mycobacterial pneumonia can also cause croup. Croup most often occurs in children and is less likely after six years of age. The virus causes swelling just below the vocal folds in the subglottic area. This results in the characteristic "barking" cough and inspiratory stridor. The onset of illness usually begins with several days of nasal congestion and coryza. A fever may also be present. Care should be taken to distinguish cases of croup from epiglottitis or other causes of acute upper airway obstruction. This can be accomplished through careful history taking, physical exam, and radiographic findings. Treatment should initially begin with dexamethasone and possibly nebulized epinephrine depending on severity. Children who are observed for a period of time and do not have recurrent or persistent symptoms, have good air exchange, and normal vital signs may be discharged home. For children with persistent, severe symptoms, hospitalization is indicated. Edema of the epiglottis (A) is seen in epiglottitis and can be noted on a lateral soft tissue radiograph of the neck as the "thumb print" sign. Epiglottitis can quickly cause airway compromise and if this finding is present, steps should be taken to protect the airway. Visualization of intraluminal membranes of the tracheal wall (B) is a finding with bacterial tracheitis. Patients with bacterial tracheitis may initially present with fever, cough and stridor but rapidly progress to upper airway obstruction. They are often toxic-appearing. Thickening of the retropharyngeal space (D) can be seen with a retropharyngeal abscess. Symptoms include pain in the throat, fever, difficulty swallowing or breathing, and neck stiffness.

Which of the following is one of the most helpful signs to rule out testicular torsion? Presence of a bell-clapper deformity Presence of Prehn's sign (relief of scrotal pain upon elevation of scrotum) Presence of the cremasteric reflex Vertical lie of testicle

Correct Answer ( C ) Explanation: The cremasteric reflex is a superficial reflex elicited by lightly stroking the superior and medial (inner) thigh in a male. The normal response is an immediate contraction of the cremaster muscle that pulls up the testis (>0.5 cm) on the side stroked. The presence of a cremasteric reflex appears to be one of the most helpful signs in ruling out testicular torsion. The cremasteric reflex may be absent in conditions such as testicular torsion, upper and lower motor neuron disorders, spinal injury of L1-L2, and iatrogenic transection of the ilioinguinal nerve during surgery for hernia repair. Despite being one of the most helpful signs, it is important to note that the presence of a cremasteric reflex can not rule out testicular torsion with 100% certainty. The bell-clapper deformity (A) is an entity in which the tunica vaginalis completely encircles the epididymis, distal spermatic cord, and the testis rather than attaching to the posterolateral aspect of the testis. This causes the testicle to hang free within the tunica vaginalis, in a horizontal fashion. This anatomic abnormality predisposes to spermatic cord torsion. Relief of pain with elevation of a painful testicle represents a positive Prehn's sign (B) and points toward a diagnosis of epididymitis rather than testicular torsion. It is thought that by elevating the painful testicle, the pain of epididymitis improves because the elevation takes the weight of the testis off the epididymal suspension but does not affect the testicle in cases of testicular torsion. However, Prehn's sign is unreliable and should not be used to rule out testicular torsion. Although a vertical lie (D) of the testicle is normal, it does not rule out testicular torsion

A 38-year-old man with a past medical history of hypertension and chronic kidney disease presents with severe left foot pain that began in the middle of the night. He appears comfortable, but has exquisite tenderness to palpation of the left, first metatarsophalangeal joint. The joint is erythematous and warm. His vital signs include a temperature of 99.1ºF, BP 132/85 mm Hg, RR 16 breaths/minute, and oxygen saturation 99% on room air. Arthrocentesis of the involved joint is performed and synovial fluid analysis reveals negatively birefringent crystals. Which of the following is the most appropriate therapy at this time? Allopurinol Naproxen Prednisone Probenecid

Correct Answer ( C ) Explanation: The patient has acute gouty arthritis. Gout is caused by precipitation of sodium urate crystals in the synovial fluid. Pain is most commonly in the knee and foot, classically the first metatarsophalangeal joint (podagra). The diagnosis is made by joint aspiration and analysis of synovial fluid for the presence of polarizing crystals. Negatively birefringent crystals are diagnostic of gout. Negative birefringence describes the blue color and perpendicular orientation of the uric acid crystals relative to the microscopic light source. First line treatment for acute gout is with colchicine and nonsteroidal anti-inflammatory drugs (NSAIDs), such as indomethacin or naproxen unless contraindications exist. Renal insufficiency is a contraindication to NSAID use and should be avoided in this patient. Patient intolerant to NSAIDs may be treated with steroids such as prednisone.

A 42-year-old man with a history of alcohol abuse presents with severe abdominal pain and vomiting following an alcohol binge. Lab values are as follows: WBC 14,000, Hgb 10, platelets 98, lipase 9000, AST 200, ALT 107, T. bill 1.6, Na 134, K 5.1, Cr. 3.1, and glucose 313. Which of the following lab values is associated with increased mortality? AST of 200 Creatinine of 3.1 Glucose of 313 Lipase of 9000

Correct Answer ( C ) Explanation: The patient has acute pancreatitis. Pancreatitis can be caused by gallstones, alcohol, infection, hypertriglyceridemia, hypercalcemia, tumors, pancreatic divisum, medications, and trauma. Together, gallstones and alcohol are responsible for 80% of cases. Pancreatitis spans a gamut from mild to severe, with some experiencing a mild, self-limiting illness while others develop SIRS and a shock-like state. Presenting symptoms include epigastric abdominal pain that is constant and often radiates to the back, with associated nausea and vomiting. Physical examination is notable for marked epigastric tenderness. Jaundice may be present if an obstructing stone is the cause. Ranson's criteria use clinical and laboratory parameters at two different time points to predict mortality in acute pancreatitis. Ranson's criteria on admission include age > 55, glucose > 200, WBC > 16,000, AST > 250, LDH > 350. At 48 hours, findings associated with increased mortality include a decrease in hematocrit > 10%, an increase in BUN > 5 mg/dL, Ca < 8 mg/dL, PaO2 < 60 mm Hg, base deficit > 4 mEq/L, rapid fluid sequestration > 6 L. Patients with 0-3 positive findings have a mortality rate of 1%, while those with 3-4, 5-6, > 7 findings have mortality rates of 15%, 40%, 100%, respectively. In this case, glucose of 313 is a predictor of increased mortality. AST of 200 (A), elevated creatinine (B), and elevated lipase (D) are not predictors of mortality in acute pancreatitis according to Ranson's criteria.

A 44-year-old man with systemic lupus erythematosus comes to the ED with chest pain and dyspnea. Vital signs are notable for tachycardia and hypotension with a narrow pulse pressure. Bedside ultrasound confirms a large pericardial effusion with collapse of the right ventricle during diastole. Which classic electrocardiogram findings are associated with this disease process? Delta wave and shortened PR interval Diffuse ST elevation Low voltage and electrical alternans S wave in lead I and a Q wave and inverted T wave in lead III

Correct Answer ( C ) Explanation: The patient has cardiac tamponade. In cardiac tamponade, fluid distends the pericardial sac and compresses the heart, impairing ventricular filling and cardiac output. Many disorders can result in a pericardial effusion, including inflammatory disorders like SLE, infections (tuberculosis, viruses), and malignancies. The pericardium can stretch to accommodate excess fluid, but when fluid accumulates to the point where intrapericardial pressure exceeds the normal filling pressures of the right heart, ventricular filling is impaired, and cardiac tamponade results. Cardiac tamponade is diagnosed by a combination of clinical signs and ultrasound findings. Tachycardia and hypotension with a narrow pulse pressure are characteristic, and echocardiogram will reveal a large pericardial effusion causing right atrial compression and right ventricular diastolic collapse. Classic ECG findings of cardiac tamponade include low voltage QRS complexes, diffuse ST elevation, and PR-segment depression (due to pericardial inflammation), and electrical alternans, where there is beat-to-beat variation in the amplitude of the R waves. The treatment for cardiac tamponade causing hemodynamic instability is emergent pericardiocentesis. Delta wave and shortened PR interval (A) are ECG findings of Wolff-Parkinson-White syndrome. Diffuse ST elevation (B) is seen in pericarditis. S wave in lead I and a Q wave and inverted T wave in lead III (D) is the S1Q3T3 pattern of right heart strain.

A 32-year-old woman, who is at 20 weeks gestational age, presents to the ED after a seizure. Her vital signs are BP 115/70, HR 105, RR 16, T 37.5°C, and pulse oximetry 98% on room air. On exam, you note some confusion, but otherwise there are no focal deficits. Lab results reveal a hemoglobin of 7 g/dL and platelets of 12,000/µL. A peripheral blood smear reveals schistocytes. Which of the following is the most appropriate treatment for her condition? Delivery of fetus Magnesium sulfate Plasmapharesis Platelet transfusion

Correct Answer ( C ) Explanation: The patient has thrombotic thrombocytopenic purpura (TTP). The classic pentad of TTP includes CNS abnormalities, renal pathology, fever, microangiopathic hemolytic anemia, and thrombocytopenia. However, diagnostic criteria have recently been simplified to include all adults with microangiopathic or microvascular hemolytic anemia and thrombocytopenia with no other explanation for these findings. TTP shares many clinical and laboratory features of HELLP syndrome. HELLP syndrome is less common before 24 weeks gestation. The derangements in hemoglobin and platelet levels are more severe in TTP. The mainstay of treatment for TTP is plasmapheresis (plasma exchange), which can achieve remission of disease in 80% of patients. If plasmapheresis cannot be immediately performed, fresh frozen plasma (FFP) should be administered until pheresis can be performed.

A 39-year-old woman presents to the emergency department with right leg pain and mild swelling. She denies any previous medical history or trauma. She does not take any medication. Examination is remarkable for tenderness along the right calf venous distribution and minimal swelling of the right lower leg. What is the most appropriate diagnostic workup of this patient? Duplex ultrasonography to evaluate for deep venous thrombosis. If negative, send a D-dimer. If D-dimer is positive, treat for deep venous thrombosis. Duplex ultrasonography to evaluate for deep venous thrombosis. If negative, the patient may be discharged. Send a D-dimer. If negative the patient may be discharged. Send a D-dimer. If positive, treat for deep venous thrombosis.

Correct Answer ( C ) Explanation: The patient in this question is presenting to the emergency department with signs and symptoms characteristic of deep venous thrombosis (DVT). When determining the appropriate workup for a patient presenting with possible DVT, pretest probability for DVT should be assessed and the patient should be risk stratified accordingly. The use of Well's criteria can help the clinician risk stratify the patient into the three-tiered or two-tiered scoring system. The patient in this question has localized tenderness along the distribution of the deep venous system and has minimal swelling to her leg. These findings give this patient a Well's score of one which puts her at moderate risk for DVT. Patients thought to have low to moderate pretest probability can be effectively ruled out for DVT with a high-sensitivity D-dimer. D-dimer can be used to rule out venous thrombotic disease in the correct patient population as the test is sensitive but not specific. Use of the D-dimer should not be instituted in patients at high risk for DVT. In a patient with low-to-moderate risk for DVT, the appropriate work up includes at least a negative high-sensitivity D-dimer test in order to complete the work up without imaging. If duplex ultrasonography is performed first in this moderate risk patient and is negative, either a high-sensitivity D-dimer test should be sent and should be negative or the patient should seek serial duplex ultrasonography.

A seven-year-old girl presents to the ED with noisy breathing, fever, and drooling. On physical exam, she is in obvious respiratory distress with audible stridor at rest. Vital signs are BP 90/60 mm Hg, HR 150 beats/minute, RR 40 breaths/minute, and T 103.8 °F. Which of the following are you likely to see on a lateral radiograph of the soft tissues of the neck? Steeple sign Subcutaneous emphysema Thumbprint sign Widening of the prevertebral soft tissues

Correct Answer ( C ) Explanation: This child is exhibiting signs and symptoms of epiglottitis. Diagnosis of epiglottitis is clinical. If the diagnosis is not clear, a lateral soft tissue X-ray of the neck could be obtained and would reveal a "thumbprint" sign. This should be done at the bedside given the risk of losing the patient's airway in radiology. Epiglottitis is historically most commonly caused by Haemophilus influenzae type B; however, the incidence of this has declined substantially with childhood vaccinations. Signs and symptoms of epiglottitis include rapid onset of fever, dysphagia, dysphonia, and stridor. Children will often present in the tripod position for comfort and ease of breathing. Management of epiglottitis includes a primary focus on the airway which involves keeping the patient in a position of comfort and consultation of otolaryngology and anesthesia for appropriate airway management preferably in the operating room. A third-generation cephalosporin should also be administered intravenously to treat the most likely infectious etiology.

A 25-year-old G1P0 woman at 38 weeks gestation presents to the ED after feeling a gush of fluid. On tocometry you note late decelerations of the fetal heart rate. You perform a sterile vaginal exam and feel a pulsating mass protruding from the cervical os. Which of the following appropriately matches the diagnosis with the next step in management? Breech positioning, attempt external version Shoulder dystocia, knee-to-chest positioning Umbilical cord prolapse, elevate presenting part Uterine inversion, administer tocolytics

Correct Answer ( C ) Explanation: This patient has evidence of umbilical cord prolapse, which is defined by delivery of the umbilical cord prior to the delivery of the fetus. Pressure of the presenting part, typically the occiput, onto the prolapsed cord leads to anoxic injury to the fetus. Umbilical cord prolapse is significantly associated with malpresentations and premature rupture of membranes (PROM). The diagnosis of umbilical cord prolapse is made on sterile vaginal exam by either visualization of the cord in the vault or palpation of a pulsatile mass at the os, or doppler ultrasound. Management of umbilical cord prolapse is by immediate elevation of the presenting part and emergent cesarean section. Other options include trendelenburg or knee-to-chest positioning, telling the mother to avoid pushing, and instilling 500 cc of normal saline into the bladder.

32-year-old woman presents with progressive fatigue, shortness of breath and chest discomfort that has worsened over the last two days. She had a mild upper respiratory infection last week but otherwise denies any significant past medical history. She does not take any medications and denies any illicit drug use. On examination, she has a temperature 37.9ºC, heart rate 132 beats per minute, respiratory rate 16 breaths per minute, and oxygen saturation 98% on room air. Lungs are clear to auscultation. Chest radiograph demonstrates mild cardiomegaly without infiltrates. Electrocardiogram shows sinus tachycardia with nonspecific ST segment and T wave changes. Troponin is 0.25 ng/mL. What is the likely cause of her symptoms? Acute coronary artery thrombosis Community acquired pneumonia Myocarditis Pulmonary embolism

Correct Answer ( C ) Explanation: This patient has findings concerning for myocarditis, an inflammatory disease of the myocardium. In developed countries, viruses are most frequently implicated (e.g., enteroviruses, parvovirus B19, herpes virus 6) but it can also be caused by bacteria, fungi, protozoa and helminths. Presentation is variable depending on the etiology and stage of disease. Fatigue, fever, chest discomfort, dyspnea and palpitations are common presenting complaints in adults. Patients with advanced disease may present with dysrhythmias, heart failure, and cardiogenic shock. On physical exam, there may be tachycardia that is disproportionate to fever or discomfort, an S3 or S4 gallop, and signs of fluid overload. As pericarditis often occurs concomitantly with myocarditis, a friction rub may be heard. The electrocardiogram is often abnormal, with changes including sinus tachycardia and widened QRS interval. There may be regional ST elevations that mimic acute myocardial infarction. Findings suggestive of pericarditis (e.g., diffuse ST elevation with PR segment depression) may also be present. Troponin, erythrocyte sedimentation rate and white blood cell count may also be elevated but are not diagnostic. Echocardiogram will show decreased ventricular ejection fraction with hypokinesis and wall motion abnormalities. Endomyocardial biopsy has long been the gold standard for diagnosis but is being replaced with cardiovascular magnetic imaging.

A 14-year-old boy presents complaining of intense pruritus in his groin, axillae, and between his fingers after returning home from summer camp 1 week ago. He reports several other campers had similar symptoms. On exam, you note excoriations in the inguinal region and axillae surrounding scattered, erythematous papules. Which of the following is the most appropriate treatment? Ketoconazole Lindane Permethrin Prednisone

Correct Answer ( C ) Explanation: This patient has scabies; a pruritic dermatitis caused by cutaneous infection with the mite Sarcoptes scabei, var hominis. Scabies is spread by skin-to-skin contact and should be considered in patients with generalized pruritus, especially when exposure to others with similar symptoms is reported. The rash of scabies involves papules, which are often excoriated. Burrows are pathognomonic but not uniformly present. Unless previously infected, pruritus generally takes 3-6 weeks to develop because symptoms are due to delayed (Type IV) sensitivity reaction. The pruritus is classically worse at night and affects the web spaces of the fingers, flexor aspect of the wrists, axillae, groin, nipples, and the periumbilical region. Except in cases involving an immunocompromised host, the scalp and face are generally spared. Diagnosis is clinical but can be confirmed by placing scrapings collected with a #15 blade scalpel in mineral oil for microscopic examination. The treatment of choice for primary scabies infection is the application of topical scabicidal agents, with repeat application in 7 days. The treatment of choice is permethrin 5% lotion. Individuals affected by scabies should avoid skin-to-skin contact with others. Patients with typical scabies may return to school or work 24 hours after the first treatment.

A 35-year-old woman presents to the ED with facial swelling. On physical exam, you note the above. Which of the following medications is most likely the cause of this process? AAmiodarone BHeparin CLisinopril DMetoprolol

Correct Answer ( C ) Explanation: This patient is exhibiting signs and symptoms of drug-induced angioedema. Angioedema is defined as edema of cutaneous and subcutaneous tissue secondary to capillary dilation. In addition to the drug-induced etiology, angioedema can also be idiopathic, allergic or hereditary. Signs and symptoms of angioedema include painless, nonpruritic, nonpitting edema of the skin. It may affect the upper airway, abdominal organs, or the genitalia. Abdominal involvement may mimic a surgical abdomen. Angioedema usually affects the tongue, lips, and face. Common triggers of drug-induced angioedema include ACE inhibitors like lisinopril which inhibit bradykinin degradation. This can occur at any time during treatment. Aspirin, NSAIDs, and any cyclooxygenase inhibitors can cause angioedema as can iodinated contrast material. Management of drug-induced angioedema is largely supportive as it is refractory to standard medical therapy with epinephrine, antihistamines, and steroids. Airway management may be required. Hereditary angioedema is an autosomal dominant process that results in C1 esterase inhibitor deficiency leading to permanently low levels of C4. Management of these patients is with fresh frozen plasma to replace C1 esterase. These patients may also require early airway management. Amiodarone (A) is most associated with lung toxicity leading to pulmonary fibrosis. Heparin (B) is most associated with thrombocytopenia leading to bleeding and microthrombi formation. Metoprolol (D) is associated with bradycardia, lightheadedness, and erectile dysfunction.

A 30-year-old man presents to the ED with three weeks of diarrhea. He reports colicky abdominal pain associated with frequent episodes of pale, loose, foul-smelling stools. He returned from a camping trip in New Hampshire one month prior. His vital signs are temperature 37.1°C, heart rate 85, blood pressure 125/80 mm Hg. Which of the following is the most appropriate treatment for this condition? Ciprofloxacin 500 mg daily for 7 days Clindamycin 300 mg QID for 7 days Metronidazole 250 mg TID for 7 days Rifaximin 200 mg TID for 7 days

Correct Answer ( C ) Explanation: This patient is presenting with signs and symptoms consistent with giardiasis, the most common cause of parasitic diarrheal infection in the United States. Fecal-oral transmission of Giardia lamblia occurs with the ingestion of cysts in contaminated water, either municipal water supplies or outdoor water sources (streams and rivers). It is rarely transmitted through food. Giardiasis is noninvasive, and infection remains confined to the lumen of the small intestine. Classic symptoms include explosive diarrhea, colicky abdominal pain, and pale, loose, foul-smelling stools. The incubation period is one to three weeks followed by an abrupt onset. Treatment is empiric. The appropriate regimen is metronidazole 250 mg TID for 7 days. Parasitic Diarrhea Giardia: lake water, camping trip, explosive Cryptosporidium: HIV, watery, lake water Entamoeba histolytica: hepatic abscess, RUQ pain, bloody

A 22-year-old man presents with upper extremity weakness. He notes that he was drinking heavily last night and when he woke up this morning he was unable to move his hand. On examination, he has normal strength in the biceps and triceps but weakness of the wrist extensors, finger extensors and brachioradialis. Injury to what nerve accounts for his symptoms? Median nerve Musculocutaneous nerve Radial nerve Ulnar nerve

Correct Answer ( C ) Explanation: This patient presents with findings consistent with a radial nerve palsy. The radial nerve is formed from nerve roots C6 through C8. It runs adjacent to the humerus in the spiral groove of the upper arm. Compression in this area can result in a radial mononeuropathy. This occurs more frequently in intoxicated patients resulting in the term "Saturday night palsy". On examination, the patient will have normal strength in the triceps as the radial nerve gives off the branch to the triceps muscle prior to wrapping around the humerus at the spiral groove. There will be weakness of the wrist extensors, resulting in a wrist drop, as well as weakness of the finger extensors and brachioradialis muscle. Sensory deficits will follow the radial nerve distribution which includes the radial aspect of the dorsum of the hand. Diagnosis is based on clinical findings. Treatment includes splinting the wrist, physical therapy and pain management as needed. The median nerve (A) is responsible for motor function of the flexor muscles of the anterior forearm compartment (except for the flexor carpi ulnaris and part of the flexor digitorum profundus), the thenar muscles and the lateral two lumbricals. Compression of the median nerve as it passes through the carpal tunnel of the wrist will resulting in pain and paresthesias in the distribution of the median nerve. The musculocutaneous nerve (B) innervates the coracobrachialis, the biceps brachii and the brachialis. Injury to this nerve will result in weakness of elbow flexion with associated sensory loss over the lateral forearm. Ulnar (D) neuropathy at the elbow is the second most common compression neuropathy and results in sensory loss and paresthesias over the fourth and fifth digits as well as weakness with finger and wrist flexion.

A 26-year-old man presents to the ED after being hit in the head by a foul baseball. The patient was initially alert and talking to you. He is now becoming progressively more somnolent. Which of the following would you expect to see on a non-contrast computed tomography scan of the head? A collection of blood layering in the basilar cisterns A crescent-shaped frontal hematoma crossing suture lines A lenticular-shaped hematoma in the temporal region An intraparenchymal hemorrhage within the frontal lobe

Correct Answer ( C ) Explanation: This patient's presentation and physical examination are consistent with a traumatic acute epidural hematoma, a collection of blood between the skull and the dura. The diagnosis is made with a non-contrast CT of the head. This characteristically shows a hyperdense lenticular-shaped hematoma in the temporal region that is sharply defined and does not cross suture lines. Epidural hematomas are usually associated with skull fractures in the temporal bone region resulting in laceration of the middle meningeal artery. Arterial bleeding is the etiology of ⅔ of epidural hematomas and is typically rapid. Signs and symptoms include severe headache, drowsiness, nausea, and vomiting. The classic finding is the lucid interval just prior to rapid deterioration; however, this is present in less than 30% of epidural bleeds. Neurosurgery should be consulted immediately for surgical evacuation of the hematoma. A collection of blood layering in the basilar cisterns (A) is the classic imaging presentation of subarachnoid hemorrhage. A crescent-shaped frontal hematoma crossing suture lines (B) is the classic appearance of a subdural hematoma. An intraparenchymal hemorrhage within the frontal lobe (C) is the appearance of a traumatic contusion or hypertensive hemorrhage.

Which of the following is more consistent with the diagnosis of encephalitis rather than meningitis? Fever Headache Neck stiffness New psychiatric symptom

Correct Answer ( D ) Explanation: The clinical distinction between encephalitis and meningitis is typically characterized by the presence of a distinct neurologic abnormality in encephalitis. Encephalitis should be considered in patients presenting with the following clinical features, alone or in combination: new psychiatric symptoms, cognitive deficits (aphasia, amnesia, acute confusional state), seizures, and movement disorders. Often, there are symptoms of meningeal involvement as well, such as a headache, photophobia, and fever. There is overlap between encephalitis and meningitis. When this occurs, the condition is referred to as meningoencephalitis.

A 19-year-old G1P0 at 6 weeks by dates presents to the ED complaining of a small amount of vaginal spotting for the past 2 days. Vital signs are within normal limits. Your pelvic exam reveals a closed os and no adnexal tenderness or masses. Laboratory analysis reveals a serum beta hCG of 1830 mIU/ml and type B, Rh-positive blood. A transvaginal ultrasound shows an intrauterine gestational sac without a yolk sac or fetal pole and a right ovarian cyst that measures 2 x 1 cm. Given this information, which of the following is the most likely diagnosis? Blighted ovum Ectopic pregnancy Implantation bleeding Threatened miscarriage

Correct Answer ( C ) Explanation: This patient's serum beta-hCG level and transvaginal ultrasound are suspicious for a 6-week gestation; however, due to the lack of yolk sac or fetal pole, it cannot be definitely called an intrauterine pregnancy (IUP) as the diagnosis of an IUP requires a gestational sac located within the endometrium and that contains a yolk sac and/or fetal pole. Given the lack of diagnostic criteria, this bleeding should be attributed to implantation bleeding, which is relatively common in early pregnancy and occurs in around 9% of women within the first eight weeks of pregnancy. A blighted ovum (anembryonic gestation) (A) is a pregnancy in which the very early pregnancy appears normal on an ultrasound scan. But as the pregnancy progresses, a visible embryo never develops. In a normal pregnancy, an embryo would be visible on an ultrasound by 6 weeks after the last menstrual period. Because there is vaginal bleeding, the presentation is classified as a threatened miscarriage rather than a blighted ovum. The possibility of an ectopic pregnancy (B) has not been eliminated in this patient, and she requires close follow-up. However, it is less likely, given her lack of ectopic risk factors and the close approximation of her dates, ßhCG level, and ultrasound. In the scenario that this could be determined as an actual intrauterine pregnancy (with necessary findings), the presence of 1st-trimester vaginal bleeding and a closed internal cervical os, she should be classified as a threatened miscarriage (D).

A 21-year-old woman presents with pain, tearing, photophobia, and left eye redness. She has been wearing her contact lenses continuously for the last two weeks. A slit lamp examination reveals a white, hazy opacity on the cornea at the 3 o'clock position of the cornea with associated limbal flush. Which of the following is the most appropriate pharmacotherapy? Topical acyclovir Topical amphotericin B Topical ciprofloxacin Topical cyclopentolate

Correct Answer ( C ) Explanation: Topical ciprofloxacin is first-line treatment for a corneal ulceration. This is because of the necessary coverage against Pseudomonas aeruginosa. A corneal ulcer is a bacterial infection that develops secondary to a break in the corneal epithelium. Risk factors for developing a corneal ulcer include incomplete lid closure (e.g. secondary to Bell's palsy) and soft contact lenses use. Symptoms include redness, swelling of the lids, foreign body sensation, and photophobia. Physical exam may reveal a round or an irregular ulceration with a white or hazy base. A secondary iritis can develop and cause ciliary flush or a limbal injection. Treatment includes topical antibiotics and emergent ophthalmologic consultation. Cultures can be obtained of the ocular drainage to guide antibiotic therapy. Cycloplegic drops can be used to help with the pain from the secondary iritis. Complications include corneal scarring, corneal perforation, and secondary glaucoma.

61-year-old man presents with intermittent shock-like spasms of pain in his right cheek. The pain only lasts a few seconds at a time but recurs frequently. He notes that shaving each morning causes intense spasms of pain, as does chewing. What is first-line treatment for his likely diagnosis? Baclofen Botulinum injections Carbamazepine Diazepam

Correct Answer ( C ) Explanation: Trigeminal neuralgia is characterized by unilateral, shock-like paroxysms of pain in distributions of one or more divisions of the trigeminal nerve. It is often the result of vascular compression of the trigeminal nerve root. It is more common in women than men and typically affects patients over 50 years of age. The pain can be triggered by such things as chewing, shaving, light touch or exposure to hot or cold temperatures. Diagnosis is made clinically based on history and exclusion of other possible sources of pain, such as an odontogenic infection, temporomandibular joint disease and herpes zoster. The first-line treatment of choice for trigeminal neuralgia is carbamazepine. The initial dosage is 100 mg twice daily. This can be increased up to 1200 mg/day as needed for symptom control. Patients should be monitored periodically with laboratory studies to rule out potential hematologic or hepatic side effects.

A 35-year old man presents to the emergency department with superficial facial lacerations and epistaxis. After treating the epistaxis, you examine the nasal mucosa. Which of the following is considered an emergency, requiring urgent treatment? A deviated septum Closed nasal fracture Septal hematoma Trauma to Kiesselbach plexus

Correct Answer ( C ) Explanation: A septal hematoma is considered an emergency. The problem is that the perichondrium, which supplies nutrition to the septum, is no longer in contact with the septum because of the intervening hematoma. Thus, the septal cartilage can necrose leading to a perforated septum. Septal hematomas should be drained acutely and the nose packed to keep the perichondrium in contact with the septal cartilage.

Which of the following drugs can cause mydriasis in overdose? Hydromorphone Lorazepam Meperidine Paraquat

Correct Answer ( C ) Explanation: At therapeutic doses, meperidine causes miosis (pinpoint pupils). However, unlike most other opioids, meperidine can cause mydriasis (dilated pupils) in cases of toxicity. Increased muscle tone, twitching, and tremors may also be seen with meperidine overdose. Meperidine is metabolized by the P450 system in the liver to the metabolite normeperidine, which can cause CNS toxicity at therapeutic meperidine doses. The half-life of normeperidine is up to 48 hours and subsequent hallucinations, seizures and psychosis may result. Therefore, meperidine is not routinely used for the management of acute pain in the Emergency Department. Meperidine can also potentially interact with monoamine oxidase inhibitors and cause serotonin syndrome.

A patient presents with symptomatic bradycardia following an overdose of metoprolol. After securing the airway, the most appropriate next step is administration of which of the following? Adenosine Amiodarone Atropine Glucagon

Correct Answer ( C ) Explanation: Atropine is used to treat symptomatic bradycardia. Metoprolol is a beta-adrenergic antagonist. Beta blockers are categorized according to their cardioselectivity. Cardioselective beta-blockers preferentially inhibit beta-1 receptors, resulting in depressed myocardial contractility, decreased automaticity of pacemaker cells, and decreased conduction velocity through the atrioventricular node. Patients with overdoses of beta-blockers usually become symptomatic within two to six hours after ingestion. Bradycardia and hypotension are the most common effects. Altered mental status, seizures, hypoglycemia, and bronchospasm are other potential effects. Hypoglycemia is more commonly seen in children. If beta blocker overdose is suspected, an ECG, fingerstick glucose, and electrolyte panel should be obtained. Airway management and advanced cardiac life support (ACLS) should be provided, followed by establishing intravenous (IV) access and continuous cardiac monitoring. Treat hypotension with IV boluses of isotonic fluid and hypoglycemia with boluses of 50 percent dextrose in water. IV glucagon, vasopressors, and calcium salts can be used depending upon the severity of the overdose. Glucagon is considered to be antidote for beta blocker overdose despite limited evidence. Glucagon may be effective initially for a brief period, but will usually become ineffective due to tachyphylaxis and has little effect on mean arterial pressure. High dose insulin therapy, however, is now the mainstay in treatment of beta blocker toxicity. Adenosine (A) is an antiarrhythmic drug used to convert paroxysmal supraventricular tachycardia to sinus rhythm. Adenosine slows conduction through the atrioventricular node and prevents re-entry pathways. Administration of adenosine would decrease this patient's heart rate. Amiodarone (B) is a class III antiarrhythmic drug that is used in ACLS to treat ventricular dysrhythmias, supraventricular dysrhythmias, and atrial fibrillation. Amiodarone has a black box warning for pulmonary toxicity and hepatotoxicity. Amiodarone is not used in the treatment of bradycardia. While glucagon (D) may be used in the treatment for beta blocker overdose, it is not first-line for treating symptomatic bradycardia. Glucagon can be used to help correct hypoglycemia but will have little effect on blood pressure.

Which of the following patients is most likely to sustain a Chance fracture in a motor vehicle collision? A 19-year-old unrestrained passenger A 2-year-old girl in a rear-facing seat with a five-point harness A 32-year-old pregnant passenger wearing a lap belt A 40-year-old driver wearing a three-point belt

Correct Answer ( C ) Explanation: Chance fractures, as seen above, are a variant of the flexion-distraction injury of the spine. These fractures typically occur in the thoracolumbar junction, which spans from T11-L2. Flexion-distraction injuries result from sudden deceleration and most commonly occur in restrained passengers of motor vehicle collisions and people who have fallen from a height. These injuries are most commonly seen in people who wear only lap belts that are improperly positioned above the pelvic bones. This is especially common in pregnant or obese patients. The sudden deceleration associated with a collision leads to forceful flexion at the location of the lap belt, resulting in compression of the anterior and middle columns and tearing of posterior ligamentous support. Flexion-distraction injuries are not typically seen in patients who were wearing three-point restraints at the time of the trauma. A seat belt sign may be seen on physical exam. Radiographic findings in these injuries include posterior vertebral wall fracture, increased height of the posterior vertebra, and fanning of the spinous processes. The Chance fracture also involves anterior vertebral compression and significant distraction of the middle and posterior ligaments. Chance fractures are often misdiagnosed as isolated compression fractures when viewed in the axial view during computed tomographic imaging, and sagittal reconstruction is often necessary for proper diagnosis. Chance fractures are commonly associated with intra-abdominal injuries and a high index of suspicion should be maintained for bowel perforation or solid organ damage in patients with Chance fractures.

Which of the following is the immediate first step in the treatment of chemical injuries to the eye? Application of topical antibiotics to prevent infection Complete ocular examination, including dilated fundoscopic examination Manual removal of particulate material followed immediately by irrigation with saline until the pH is 7.0 Topical steroids and artificial tears

Correct Answer ( C ) Explanation: Chemical injuries to the eye represent one of the true ophthalmic emergencies. While almost any chemical can cause ocular irritation, serious damage generally results from either strongly basic (alkaline) compounds or acidic compounds. Alkali injuries are more common and can be more deleterious. Bilateral chemical exposure is especially devastating, often resulting in complete visual disability. Immediate, prolonged irrigation, followed by aggressive early management and close long-term monitoring, is essential to promote ocular surface healing and to provide the best opportunity for visual rehabilitation. Immediate irrigation with normal saline is the initial step even before complete exam when a patient presents with a chemical eye injury. A lid speculum should be placed and topical anesthesia applied. Irrigation may be administered by a handheld bottle or through IV tubing with an irrigation lens. The pH should be checked with a pH strip and irrigation discontinued when the pH reaches 7.0. Any particulate matter should be removed prior to irrigation if it is a reactive substance such as ammonium hydroxide crystals since fluid may dissolve these causing more injury. The upper lid should be everted to check for any particulate matter.

A 69-year-old man with a history of atrial fibrillation had a syncopal episode while on vacation in India and struck his head on the stairs. He was transported to the local clinic to be evaluated. His wife accompanied him and she informs the provider that her husband is on a "blood thinner" but she is not sure which one. The patient is alert but is not oriented to time or place. You suspect a subdural hematoma but there is no access to a CT scanner at this time. His vital signs are within normal limits. While awaiting transportation to a facility with a CT scan, which of the following labs should be ordered? Alcohol level Basic metabolic profile INR, PT, aPTT, and platelet count Vitamin K level

Correct Answer ( C ) Explanation: Elderly patients on anticoagulation therapy are at high risk of subdural hematomas when they fall. In general, elderly patients are at risk of falls because they tend to have balance difficulties and decreased vision in older age. A non-contrast CT of the head is the gold diagnostic standard for diagnosing a subdural hematoma. A subdural hematoma usually appears as a hyper dense, crescent-shape-mass between the skull and the brain. A coagulation panel is necessary in the workup of a suspected subdural hematoma because individuals on anticoagulants or who are alcoholics may have an associated coagulopathy placing them at higher risk for a subdural hematoma. In addition, if INR levels are found to be elevated, reversal agents (e.g. vitamin K and fresh frozen plasma can be administered). Therefore, all patients with a head injury should have at least a basic coagulation panel (INR, PT, aPTT, and platelet count). Fresh frozen plasma or platelets should be given as needed.

A three-year-old boy is brought to the Emergency Department by his father because of possible poisoning. He was found in the garage with furniture polish on his cheeks, mouth, and clothes. The father saw him coughing but denies choking or vomiting. On examination, the boy is active with normal vital signs. Which of the following is the next best step? AConsult pulmonology BGive activated charcoal CObtain chest X-ray DPerform gastric lavage

Correct Answer ( C ) Explanation: Hydrocarbon toxicity can result from ingestion of furniture polish. A transient and mild CNS depression is commonly noted after hydrocarbon ingestion or inhalation. Aspiration is characterized by coughing, which usually is the first clinical finding. It is important to obtain a chest radiograph. It may initially be normal, but they often show abnormalities within six hours of exposure in patients who have aspirated. Respiratory symptoms can remain mild or progress rapidly to the acute respiratory distress syndrome (ARDS) and respiratory failure. If hydrocarbon-induced pneumonitis develops, respiratory treatment is supportive. Corticosteroids or prophylactic antibiotics have not shown any clear benefit. Standard mechanical ventilation, high-frequency ventilation, and ECMO have all been used to manage the respiratory failure and ARDS associated with severe hydrocarbon-induced pneumonitis.

A 17-year-old man presents after being thrown a far distance off of a horse. Which of the following is consistent with an anterior cord syndrome? Isolated motor function loss Loss of all motor and sensory function Loss of pain and temperature, loss of motor function Upper greater than lower motor weakness

Correct Answer ( C ) Explanation: In order to fully understand the different syndromes of injuries to the spinal cord, it is imperative to understand the location of the tracts of the cord. The posterior columns carry tracts responsible for ipsilateral position and vibratory sensation. The lateral spinothalamic tract carries fibers for contralateral pain and temperature. The lateral corticospinal tract is responsible for ipsilateral motor function. Syndromes may be incomplete depending on how much of the cord is affected by the injury. In the anterior spinal cord syndrome, just the posterior columns are preserved and so patients lose all pain and temperature sensation as well as motor function. Most cases of anterior cord syndrome follow aortic surgery, but it has also been reported in the setting of hypotension, infection, vasospasm, or anterior spinal artery ischemia or infarct. In trauma, typically hyperflexion of the cervical spine causes the injury to the spinal cord.

A 14-year-old girl presents to the ED with wrist pain one day after falling onto an outstretched hand while playing soccer. She has normal sensation and movement and a normal radial pulse. Her exam is unremarkable with the exception of tenderness at the anatomical snuffbox. X-rays of her wrist and forearm are normal. Which of the following is the next best step in management? No immoblization Sugar-tong splint Thumb spica splint Ulnar gutter splint

Correct Answer ( C ) Explanation: Scaphoid fractures are relatively uncommon in children but are at risk of nonunion, can cause significant long-term pain, and may have normal initial X-rays. The arterial supply of the scaphoid bone enters distally leading to decreased blood and nutrient supply proximally. A fracture through the proximal or waist (central ⅓) of the bone can decrease blood supply further, especially if the artery is disrupted. This causes poor healing conditions which can lead to nonunion or osteonecrosis. In addition, many scaphoid fractures are extremely difficult to visualize on X-ray and swelling is not always present on exam. Patients with a scaphoid fracture may only have pain at the anatomical snuffbox after a fall onto an outstretched hand. Current recommendations are to place such patients in a thumb spica splint and refer to outpatient orthopedics to determine the need for further immobilization and repeat X-rays in two weeks.

An 18-month-old boy presents with his father for a cough and difficulty breathing. The father states that the child has had nasal congestion and coryza for the last two days. On exam, he has a barking cough, inspiratory stridor, and a prolonged inspiratory phase. Which of the following would you expect to see on radiographic evaluation? Edema of the epiglottis Intraluminal membranes of the tracheal wall Subglottic narrowing Thickening of the retropharyngeal space

Correct Answer ( C ) Explanation: Subglottic narrowing and distention of the hypopharynx may be seen on chest radiograph or a soft tissue neck radiograph, respectively, in a child with croup. Subglottic narrowing is often called the "steeple sign" since on the posterior to anterior view of the chest, the trachea appears to narrow toward the top and come to a point like a steeple. Croup is typically caused by parainfluenza virus although a number of other viruses as well as mycobacterial pneumonia can also cause croup. Croup most often occurs in children and is less likely after six years of age. The virus causes swelling just below the vocal folds in the subglottic area. This results in the characteristic "barking" cough and inspiratory stridor. The onset of illness usually begins with several days of nasal congestion and coryza. A fever may also be present. Care should be taken to distinguish cases of croup from epiglottitis or other causes of acute upper airway obstruction. This can be accomplished through careful history taking, physical exam, and radiographic findings. Treatment should initially begin with dexamethasone and possibly nebulized epinephrine depending on severity. Children who are observed for a period of time and do not have recurrent or persistent symptoms, have good air exchange, and normal vital signs may be discharged home. For children with persistent, severe symptoms, hospitalization is indicated. Edema of the epiglottis (A) is seen in epiglottitis and can be noted on a lateral soft tissue radiograph of the neck as the "thumb print" sign. Epiglottitis can quickly cause airway compromise and if this finding is present, steps should be taken to protect the airway. Visualization of intraluminal membranes of the tracheal wall (B) is a finding with bacterial tracheitis. Patients with bacterial tracheitis may initially present with fever, cough, and stridor but rapidly progress to upper airway obstruction. They are often toxic-appearing. Thickening of the retropharyngeal space (D) can be seen with a retropharyngeal abscess. Symptoms include pain in the throat, fever, difficulty swallowing or breathing, and neck stiffness.

A 3-year-old girl with mild intermittent asthma is playing in the other room when her father hears acute onset of choking and coughing. The episode continues for five minutes, and he brings her to the Emergency Center. On exam she is awake and alert but coughing intermittently. Her respiratory rate is 25 breaths/minute with a saturation of 94% on room air. She has mild intercostal retractions, and wheezing is present over the left lateral and posterior lung fields. Chest radiographs reveal hyperinflation of the left lobe. Which of the following is indicated? Albuterol with ipratropium Ampicillin Bronchoscopy Intramuscular dexamethasone

Correct Answer ( C ) Explanation: The child's clinical presentation is concerning for a bronchial foreign body aspiration. The classic triad of foreign body aspiration includes sudden onset of coughing, wheezing, and decreased air entry, although these symptoms are not universally present. The right mainstem bronchus is the most common location of foreign body aspiration. It is most common during the third year of life. The most commonly aspirated foreign bodies include nuts, seeds, food particles, small toys or parts of toys, or hardware. As in the case above, onset may be acute with choking and coughing. Partial obstruction of the lower airway may be manifest as wheezing. On chest radiography, hyperinflation may be noted distal to the obstruction, as the object may cause partial obstruction and air trapping. Notably, only ten percent of aspirated foreign bodies are radio-opaque, and thus they will not be visualized on chest radiographs. Thus, if a foreign body is not visualized by radiography but is suspected based on history and examination, the patient should undergo further evaluation with bronchoscopy. Multiple studies have shown that delaying bronchoscopy may increase both morbidity and mortality.

A 3-year-old boy is eating with his brother in the other room when his mother hears coughing and choking. His brother reports that he had just eaten a handful of peanuts before the symptoms began. In the Emergency Room, the child has moderate respiratory distress with a respiratory rate of 30 breaths per minute and a saturation of 93% on room air. He is placed on oxygen by facemask with improvement in saturations to 96%. Faint expiratory wheezes are noted over the left lobe. The remainder of the examination is unremarkable. A chest radiograph is normal. Which of the following is indicated? Administration of intramuscular epinephrine Administration of nebulized albuterol Emergent bronchoscopy Endotracheal intubation

Correct Answer ( C ) Explanation: The onset of acute respiratory distress following consumption of peanuts should prompt consideration of both foreign body aspiration and anaphylaxis. This child's focal wheezing, especially located over the left lobe, make foreign body aspiration most likely. Notably, chest radiography visualizes only ten percent of foreign bodies, as the majority are radiolucent. Chest radiography may also reveal focal air trapping, mediastinal shift with expiration, or focal atelectasis. However, these findings are not sufficiently sensitive to rule out a foreign body aspiration. In fact, chest radiography may be normal in up to two thirds of cases. If foreign body aspiration is suspected based on history and physical examination, emergent bronchoscopy should be performed. In fact, delayed bronchoscopy has been shown to increase morbidity and mortality in children with foreign body aspiration.

A five-year-old boy is brought to the emergency room for altered mental status. The family had a party at their house the night before. In the morning, they found their son on the floor surrounded by empty cans of beer. He appears sleepy, and on the way to the hospital, he vomited twice. On examination, the boy is sedated with sluggish pupils and flushed skin. Which of the following abnormalities would you expect with the boy's toxic ingestion? AAbnormal head computed tomography (CT) BHyperkalemia CHypoglycemia DMetabolic alkalosis

Correct Answer ( C ) Explanation: The signs and symptoms of the boy in the vignette are suspicious for ethanol ingestion. Children younger than six years of age often ingest ethanol when their exploratory behaviors lead them to unattended alcoholic beverages or to unsecured household products with high ethanol concentrations. Ethanol acts as a CNS sedative in a dose-dependent manner in overdose. Ethanol intoxication commonly manifests as altered behavior, lethargy, coma, ataxia, slurred speech, hypothermia, bradycardia, hypotension, and respiratory depression. A characteristic sickly sweet breath odor is often prominent. A history of ingestion from the caregiver or patient and characteristic physical findings are sufficient to establish the diagnosis of ethanol intoxication. Rapid blood glucose measurements should be performed in all patients with altered mental status after ethanol ingestion. Decreased blood glucose below 40 mg/dL is well described in children who have ingested ethanol. This ethanol-induced hypoglycemia results from exhausted glycogen reserves.

A 30-year-old man presents to the emergency department after being involved in a head-on motor vehicle crash. He is complaining of severe pain in his right hip. On exam, his right leg appears to be shortened and is held slightly flexed, internally rotated, and adducted. Range of motion is severely limited due to pain. An anterior-posterior plain radiograph is negative for fractures, but the right femoral head appears smaller than the uninjured side. Which of the following is the most likely diagnosis? Anterior hip dislocation Avascular necrosis of the femoral head Posterior hip dislocation Slipped capital femoral epiphysis

Correct Answer ( C ) Explanation: This man most likely has a posterior hip dislocation. Posterior hip dislocations are most commonly associated with high-energy trauma such as motor vehicle collisions, falls from significant height, and high impact sports. Artificial hips can dislocate with less force. Posterior hip dislocations account for 90% of all hip dislocations. Posterior dislocations occur when a large axial load is transmitted through a flexed knee, which may occur when the knee comes into contact with the dashboard in a crash. Given the large amount of force needed to dislocate the femoral head, the majority of dislocations are associated with other injuries. Patients typically present with severe pain, an inability to bear weight, and deformity. Range of motion will be severely limited. A detailed neurovascular exam should be conducted to rule out sciatic nerve injury. In posterior dislocations, the affected leg tends to be held in slight flexion, adduction, and internal rotation. Plain radiographs are first-line imaging for diagnosing hip dislocations and associated fractures. A posteriorly dislocated femoral head will appear smaller than the contralateral side on anteroposterior film. After ruling out life-threatening injuries, the hip should be reduced within six hours. Concurrent femoral neck fracture is a contraindication for reduction. After reduction, computed tomography scan should be performed on all traumatic hip dislocation to evaluate for fractures. After reduction for simple dislocations, the patient should be on protected weight bearing for four to six weeks. Complications of hip dislocations include arthritis, femoral head osteonecrosis, sciatic nerve injury, and recurrent dislocations. An anterior hip dislocation (A) is much more rare than a posterior hip dislocation. Anterior dislocations usually result from a hyperextension force against an abducted leg, which levers the femoral head out of the acetabulum. Patients with anterior dislocations typically hold the affected leg minimally flexed, externally rotated, and markedly abducted. On anteroposterior radiograph, the dislocated femoral head will appear larger than the contralateral side. Avascular necrosis of the femoral head (B) results from an interruption of the blood supply to the bone. Avascular necrosis is most commonly caused by excessive corticosteroid and alcohol use. Other causes include trauma, embolic event, or coagulation disorder. Avascular necrosis of the femoral head is a complication of hip dislocations. Slipped capital femoral epiphysis (D) is instability of the proximal femoral head growth plate. Slipped capital femoral epiphysis (SCFE) is most commonly seen in patients between 10 to 16 years of age. Risk factors for SCFE include male gender and obesity. SCFE typically presents with hip, medial thigh, or knee pain. On anteroposterior radiographs, the femoral head is displaced posteriorly and inferiorly in relation to the femoral neck, but is within the confines of the acetabulum.

A 54-year-old woman presents with a swollen knee. On examination, a large joint effusion is present. With which of the following spaces does the knee joint communicate? Anserine bursa Infrapatellar bursa Prepatellar bursa Suprapatellar bursa

Correct Answer ( D ) Explanation: A knee effusion is defined as fluid within the knee joint. Symmetric peripatellar or suprapatellar swelling will be present. In large effusions, excess fluid elevates the patella above the femur and the patella is ballottable against the femur. Effusions can be due to excess synovial fluid from traumatic or atraumatic inflammation, or hemarthrosis following trauma to the knee. Traumatic effusions may result from ligamentous, bony, or meniscal injuries, or overuse syndromes. Atraumatic etiologies of knee effusions include osteoarthritis, crystal arthropathies (e.g. gout and pseudogout), and septic arthritis. The knee has several bursa, which are small fluid-filled sacs which serve to decrease friction between moving structures. Some of the bursa communicate with the joint cavity directly and some do not. The suprapatellar bursa, the largest of the bursa, is not a true bursa but rather an extension of the knee joint capsule. When a knee effusion or hemarthrosis is present, fluid can freely flow into and distend the suprapatellar bursa. One technique to increase detection of small effusions is to "milk" the suprapatellar bursa, forcing fluid back into the knee joint. The anserine bursa (A) is located beneath the the anserine tendon where the gracilis, sartorius, and semitendinosus muscles insert. It does not communicate with the knee joint. There are two infrapatellar bursa (B), a superficial infrapatellar bursa and a deep infrapatellar bursa, which, in most individuals, do not communicate with the joint space. The prepatellar bursa (C) is located between the patella and the skin, and does not communicate with the joint space.

A 30-year-old woman presents complaining of vaginal itching and irritation. She also complains of vaginal discharge. Which of the following supports a diagnosis of uncomplicated candidial vaginitis? Cervical motion tenderness on bimanual examination Positive amine odor with KOH preparation Trichomonads on microscopic examination Vaginal pH < 4.5

Correct Answer ( D ) Explanation: A vaginal pH < 4.5 is suggestive of candidal vaginitis which occurs when the normal vaginal flora is overtaken with growth of Candida species, most commonly C. albicans. Clinical signs and symptoms include vaginal itching, change in discharge (which varies from little to copious and thin to thick), external dysuria, dyspareunia, and leukorrhea. Vaginal examination often shows vulvar erythema and edema, vaginal erythema, and an adherent vaginal discharge.

A 50-year-old man presents to the Emergency Department with sudden onset shortness of breath. He has a risk of heart failure with an ejection fraction of 25%. Vital signs include temperature 100º F, blood pressure 220/110 mm Hg, heart rate 125 beats/minute, and respiratory rate 30 breaths/minute. On examination, he appears anxious and tachypneic. Chest X-ray demonstrates diffuse bilateral interstitial opacification. Which of the following is the most appropriate initial treatment? Albuterol Dobutamine Furosemide Nitroglycerin

Correct Answer ( D ) Explanation: Acute hypertensive heart failure can result in pulmonary edema that must be managed with aggressive preload and afterload reduction. Nitroglycerin can be administered sublingually and intravenously to achieve rapid and titratable improvement in hypertension via vasodilation with reduction in preload and afterload. Initial management involves sublingual administration until IV access is established and a titratable infusion can be initiated. In addition to addressing hypertension, acute heart failure exacerbations are managed with supplemental oxygen, non-invasive positive pressure ventilation, loop diuretics (e.g. furosemide) in cases of severe volume overload, and assessment of an underlying cause (e.g. acute myocardial infarction, pericardial tamponade). Patients may require intubation when hypoxic or hypercarbic respiratory failure ensues. Patients who present with hypotension in the setting of acute heart failure (i.e. cardiogenic shock) should not receive nitroglycerin and may require vasopressors to maintain hemodynamic stability. Albuterol (A) is a beta-agonist used in the treatment of bronchospasm and is not used in the setting of acute heart failure. Dobutamine (B) is an inotropic vasopressor used in the management of cardiogenic shock. It alone does not improve hypotension in cases of overt shock. If the systolic blood pressure is < 90 mm Hg, norepinephrine can be added to dobutamine to achieve improvement in hypotension. Furosemide (C) may be given after vasodilators in patients with acute heart failure and severe volume overload. Prior to diuretic use, it is imperative to manage hypertension and cardiac contractility to optimize pulmonary function.

A 55-year-old man presents with fever and right upper quadrant pain. On examination you note that the patient is jaundice and slightly altered. Clinically you make the diagnosis of cholangitis. Which of the following is the final component of Reynold's pentad? Acute kidney injury Dilated common bile duct Elevated lipase Hypotension

Correct Answer ( D ) Explanation: Cholangitis is caused by obstruction of the common bile duct and subsequent ascending infection. Most commonly, a stone is the source of obstruction. The diagnosis was first described by Charcot in 1877 with the clinical triad fever, right upper quadrant pain and jaundice. It is important to recognize that both cholecystitis and acute hepatitis can cause similar symptoms. The most common organisms responsible for cholangitis are the typical pathogens of the biliary tract: E. coli, Klebsiella, Enterococcus and Bacteroides. Without relief of the obstruction and treatment with antibiotics, patients do poorly and go on to develop sepsis. The presence of Charcot's triad and sepsis - defined as hypotension and altered mental status - is called Reynold's pentad.

A 5-year-old boy with a history of sickle cell anemia presents with dyspnea and fatigue. Mom notes that one week prior he had a fever, headache and bright red rash on his cheeks which has since resolved. Laboratory studies show a decrease in his baseline hemoglobin by 30%. Infection with which virus likely precipitated his current symptoms? Coxsackie virus Human herpesvirus 6 Measles virus Parvovirus B19

Correct Answer ( D ) Explanation: Erythema infectiosum is a generally benign, self-limiting disease that affects young children ages 4 to 10 years. It is caused by parvovirus B19. After a brief, mild prodrome of malaise and low-grade (if any) fever, patients develop a characteristic facial rash with a "slapped-cheek" appearance. One to four days later, the rash spreads to the trunk and extremities and takes on a lacy appearance. While most children recover in a week, certain children with a history of hematologic abnormalities, such as sickle cell anemia, may develop transient aplastic crisis due a marked reduction of erythroid cell precursors. These patients may present with onset of pallor, fatigue, lethargy, and shortness of breath due to anemia. Laboratory studies will show an undetectable reticulocyte count and a drop in hemoglobin concentration of > 30%. Anemia is often severe enough to require transfusion. Fortunately, as the name suggests, transient aplastic crisis is self-limited as red cell production returns to normal once the infection has cleared (typically in one to two weeks).

A 56-year-old woman with a history of lung cancer presents to the emergency department with confusion, nausea, and vomiting. She is unable to provide much history due to her confusion. She is dehydrated and is oriented only to self. She is afebrile. Her renal function is normal. Her serum calcium level is 14 mg/dl. What is the most likely mechanism for her hypercalcemia? Abnormal calcium clearance in the kidneys Bony erosion due to metastases Production of vitamin D analogues by the tumor Secretion of parathyroid hormone related protein from the tumor

Correct Answer ( D ) Explanation: Hypercalcemia is a common complication of malignancy. Breast cancer, lung cancer, and multiple myeloma are the most common malignancies associated with hypercalcemia. Solid tumors are most likely to result in hypercalcemia due to secretion of parathyroid hormone related proteins (PTHrp) from the tumor. PTHrp stimulate osteoclast activity leading to increased bone resorption. The resultant hypercalcemia leads to an osmotic diuresis and dehydration. Initial treatment is to replenish hypovolemia with crystalloid fluids.

An 81-year-old man with a history of atrial fibrillation presents with abdominal pain. He fell two weeks ago and was advised to stop his anticoagulation medication after the fall. He reports diffuse, severe abdominal pain but is minimally tender on examination. He has symmetric pulses in his extremities. What is the most likely anatomic location of his pathology? Aortic bifurcation Celiac trunk Inferior mesenteric artery Superior mesenteric artery

Correct Answer ( D ) Explanation: Impairment of blood flow to the mesentery is a life-threatening emergency that leads to mesenteric ischemia. The gut receives approximately 20% of the cardiac output and within only 15 minutes of ischemia microscopic changes in the intestinal villi and mucosal sloughing within 3 hours. Arterial emboli cause most cases of acute mesenteric ischemia. Patients classically describe diffuse abdominal pain that is out of proportion to their tenderness on physical examination. Over time, with worsening ischemic changes to the intestine, the physical examination will progress to peritonitis. The most common source of embolization is cardiac either from an atrial or ventricular thrombus. Patients with atrial fibrillation (particularly those not adequately anticoagulated) are at high risk. The most common location for embolization is the superior mesenteric artery. This vessel has the largest caliber and narrow takeoff angle from the aorta making it the most likely path for clots to travel. Ischemic changes are most common in the jejunum since this portion of the small bowel has the least amount of collateral circulation from other sources.

A 27-year-old woman presents with chest pain. The patient reports that she was about to give a presentation at her job and suddenly had a feeling of impending of doom. In addition to chest pain, she had palpitations and sweating. Which of the following is the most likely diagnosis? AAcute stress disorder BAgoraphobia CHypomania DPanic attack

Correct Answer ( D ) Explanation: Panic attacks fall under the spectrum of anxiety disorders along with agoraphobia, obsessive-compulsive disorder, posttraumatic stress disorder and acute stress disorder. A panic attack is the sudden onset of symptoms including fear or terror often associated with a sense of impeding doom. Anxiety disorders in general are caused by a combination of biopsychosocial factors and often interactions with situations of stress or trauma. Panic attacks can occur spontaneously without an expected trigger. The attacks are described as periods of significant fear followed by a constellation of symptoms that peak about 10 minutes after symptom onset. There are many manifestations of the attack including palpitations, sweating, shortness of breath, chest pain, nausea, dizziness, feeling of impeding doom, numbness and tingling, and trembling.

A 31-year-old man presents to the Emergency Department in police custody after he was found unclothed, confused, and running down a snowy street. His urine drug test is positive for phencyclidine (PCP). Which of the following ocular findings is most likely present on physical examination? Pinpoint pupils Proptosis Ptosis Rotary nystagmus

Correct Answer ( D ) Explanation: Phencyclidine (PCP) is a dissociative anesthetic that presents with agitation, violence, and confusion. It oftens causes a diminished pain response and perceptions of superhuman strength. Tachycardia and hypertension are commonly seen vital sign abnormalities. Rotary nystagmus is classically associated with phencyclidine, but horizontal and vertical nystagmus may be present. Phencyclidine is associated with several complications including rhabdomyolysis, seizures, and hypoglycemia. Benzodiazepines are the first-line treatments for the severe agitation. Antipsychotics may be used as an adjunct therapy. Miosis or pinpoint pupils (A) are associated with opioids and cholinergic agents. Proptosis (B) is the anterior displacement of the globe in relation to the orbit that is frequently seen in Graves' disease, orbital cellulitis, and retrobulbar hemorrhages. Ptosis (C) is drooping of the upper eyelid, often associated with Horner syndrome and myasthenia gravis.

Which of the following types of traumatic events has the highest probability of resulting in posttraumatic stress disorder? Combat exposure Death of a loved one Kidnapping Rape

Correct Answer ( D ) Explanation: Posttraumatic stress disorder is characterized by reexperiencing a traumatic life event. While many types of events can cause PTSD, sexual relationship violence, such as rape or childhood sexual assault, has the highest probability of causing PTSD. Other potential inciting events include the unexpected death of a loved one or life-threatening illness of a child, physical abuse or assault as a child or adult, combat exposure, exposure to organized violence (eg. refugees or civilians in a war zone) or involvement in a motor vehicle collision. Women are more likely than men to develop PTSD. Patients who are diagnosed with PTSD develop intrusive recollection of the inciting event in the form of recurrent distressing dreams or flashbacks. The disorder is also characterized by avoidance of stimuli associated with the trauma and hyperarousal. Symptoms can occur any time after the event, although most occur within months, and need to be present for at least one month and cause significant impairment in functioning.

An 18-year-old man presents complaining of fever, right-sided abdominal pain, anorexia and vomiting. When you palpate his left lower quadrant, he complains of pain in his right lower quadrant. Based on this information, which of the following signs is considered positive? McBurney's sign Obturator sign Psoas sign Rovsing's sign

Correct Answer ( D ) Explanation: Rovsing's sign is positive when palpation of the left lower quadrant causes pain in the right lower quadrant, by pushing bowel contents towards the ileocecal valve and thus increasing pressure around the appendix. A positive Rovsing sign supports a diagnosis of acute appendicitis, although other pathology can cause the test to be positive.

A 33-year-old man presents with pain and swelling in his left knee since yesterday. He has also had a fever with a maximum temperature of 102.3°F. Physical exam reveals swelling of the left knee with erythema and warmth noted. There is pain with passive range of motion. What is the most likely diagnosis? Bursitis Gout Osteoarthritis Septic arthritis

Correct Answer ( D ) Explanation: Septic arthritis is an infection of the joint that is typically characterized by joint pain, joint swelling, and a fever. The involved joint can present with an effusion and be painful with both active and passive range of motion. It is more common in individuals with a history of intravenous drug use, rheumatoid arthritis, immunocompromise, diabetes mellitus, recent joint surgery, or a prosthetic joint. Rigors may be present as well. Joint aspiration and synovial fluid analysis is the mainstay of diagnosis. Serum erythrocyte sedimentation rate (ESR) will be elevated. Management includes hospital admission for parenteral antibiotics directed at Staphylococcal and Streptococcal species including resistant strains of these bacteria. This typically consists of intravenous vancomycin and a third-generation cephalosporin. An emergent orthopedic consultation should be obtained for consideration of surgical irrigation of the joint.

A 35-year-old woman with a history of migraines and polycystic kidney disease presents to the Emergency Department with a severe, diffuse headache. The onset was abrupt approximately one hour prior to arrival. Her vital signs are within normal limits. She has photophobia and phonophobia, as well as pain with extraocular movements. Which of the following is the most likely diagnosis? Classic migraine Idiopathic intracranial hypertension Meningitis Subarachnoid hemorrhage

Correct Answer ( D ) Explanation: Subarachnoid hemorrhage (SAH) accounts for approximately one-third of all hemorrhagic strokes. Atraumatic SAH is due to ruptured aneurysms. A history of polycystic kidney disease is a risk factor for development of berry aneurysms which may spontaneously rupture and cause a subarachnoid hemorrhage. Other medical conditions associated with SAH include Marfan syndrome, coarctation of the aorta and fibromuscular dysplasia. Classic symptoms include an abrupt "thunderclap" headache that is maximal in severity at onset. Patients often have signs of meningeal irritation secondary to blood in the subarachnoid space. These include nuchal rigidity, painful extraocular movements, photophobia, and a positive Brudzinski or Kernig's sign. Subarachnoid blood appears white on noncontrast head CT and most often appears in the cerebral cisterns within which lies the vessels that compose the circle of Willis. Management includes supportive care, including airway management as needed, nimodipine, and neurosurgical consultation.

Which of the following is correct when estimating an adult burn patient's percent of total body surface area (TBSA) affected? Each arm is approximately 18% The area covered by the patient's palm is approximately 5% The entire head is approximately 18% The perineum is approximately 1%

Correct Answer ( D ) Explanation: The "Rule of Nines" is used to estimate the percentage of area burned. Only second-degree burns or greater are used in this estimation. However, a few areas do not fit into the "Rule of Nines", such as the perineum which accounts for approximately 1% of the total body surface area.

Question: Chronic diarrhea in patients with AIDS is most commonly caused by which pathogen?

Cryptosporidium.

A 30-year-old man presents to the emergency department with an ankle injury after he twisted his ankle when stepping off of the curb. The patient mainly complains of pain near the right lateral malleolus. Which of the following examination findings would be more indicative of an ankle sprain that would not require further diagnostic imaging? Bony tenderness in the malleolar zone Bony tenderness in the midfoot zone Inability to bear weight on the ankle Swelling over the bilateral malleoli

Correct Answer ( D ) Explanation: The most common type of ankle injury is an ankle sprain to the lateral ankle. Typically, these are minor and are due to an inversion injury. Sprains are categorized into three grades. Grade I involves no tearing of the ligaments with minimal functional loss, pain, swelling and ecchymosis. Weight bearing is tolerable. Grade II sprains occur with a partial tear and some loss of functional ability. They tend to be more painful, with swelling, ecchymosis, and difficulty bearing weight. Grade III sprains result from a complete tear, with significant functional loss, pain, swelling, and bruising, and an inability to bear weight. The Ottawa Ankle Rules are frequently used by physicians and nurses to determine the likelihood of a fracture versus a sprain and the need for imaging. Swelling over the bilateral malleoli is a common finding in ankle sprains. This finding, particularly without other bony tenderness in a patient who is able to bear weight, is more indicative of an ankle sprain and would not make a physician more suspicious for an ankle fracture. In patients with a lateral sprain and the ability to bear weight, treatment consists of analgesics, an elastic bandage or ankle brace and no sport involvement with follow up in a week if no improvement.

A 51-year-old man with a history of alcohol abuse presents with three days of anorexia, nausea, vomiting, and right upper quadrant abdominal pain. Vital signs are HR 115, BP 114/83, RR 20, and oxygen saturation 96% on room air. On physical examination he appears jaundiced and tender hepatomegaly is noted. Liver function tests are notable for AST 433 IU/L, ALT 206 IU/L, and total bilirubin 8.0 mg/dL. Prothrombin time is prolonged at 1.6. Which of the following is the appropriate treatment for this condition? Cholecystectomy Liver transplantation N-acetyl-cysteine Supportive care

Correct Answer ( D ) Explanation: The patient has alcoholic hepatitis. Alcoholic hepatitis is an acute inflammatory condition of the liver triggered by alcohol use or abuse. The pathophysiology of alcoholic hepatitis is complex, and includes activation of the inflammatory cytokines, oxidative stress, and metabolic derangements. Key findings on physical examination include jaundice and tender hepatomegaly. In severe cases with hepatic decompensation, ascites and encephalopathy may be present. Laboratory findings in alcoholic hepatitis include moderate AST and ALT elevations (up to 7 times normal) with a classic AST to ALT ratio of 2:1 or greater. Total bilirubin is typically 5 mg/dL or above and prothrombin time is also elevated, indicating hepatocellular necrosis. Leukocytosis may also be present. Treatment for alcoholic hepatitis is supportive, including intravenous hydration, thiamine, glucose supplementation, correction of electrolyte disturbances, nutritional support, and control of alcohol withdrawal symptoms with benzodiapezines. Abstinence from alcohol is critical in the long-term management of alcoholic hepatitis.

A 24-year-old woman presents with right thumb pain. She recently fell while skiing. On examination, she has swelling and tenderness over the base of the thumb on the ulnar side. She has weakness when pinching an object between her thumb and index finger. Which of the following structures is most likely to be injured? Extensor pollicis brevis Extensor pollicis longus Radial collateral ligament Ulnar collateral ligament

Correct Answer ( D ) Explanation: The patient has an injury to the ulnar collateral ligament, also known as "gamekeeper's thumb" named after Scottish gamekeepers who developed this injury pattern from repeatedly twisting the necks of hares. It is also known as "skier's thumb" since skiing is now the most common cause of this injury. The injury occurs when the thumb gets stuck in the pole straps during a fall. The mechanism of injury is a forceful radial abduction of the thumb with a subsequent partial or complete tear in the ulnar collateral ligament at the insertion into the proximal phalanx of the thumb. Patients present with pain, swelling, and tenderness on the ulnar side of the metacarpophalangeal joint of the thumb and a weak pincer grasp. Greater than 35 degrees of laxity of thumb abduction or additional 15 degrees of laxity beyond the uninjured side suggests a complete rupture. Radiographs may show a small associated avulsion fracture. Patients should be immobilized in a thumb spica splint and referred to orthopedics. A partial ulnar collateral ligament rupture will often heal well with immobilization alone, but a complete rupture often requires surgical repair. Extensor pollicis brevis (A) and extensor pollicis longus (B) are incorrect. Injury to these structures would cause weakness of thumb extension, not pincer grasp. Radial collateral ligament (C) injuries are less common than ulnar collateral ligament injuries.

An 18-month-old boy presents to the emergency department with worsening shortness of breath. The parents report he has had a cough, runny nose, and fussiness for the past five days. On exam, the patient demonstrates subcostal retractions, tachypnea, and diffuse wheezing. The patient is given an albuterol nebulizer treatment without any improvement of his wheezing. Chest X-ray does not show any abnormality. Which of the following organisms is the most likely cause of his symptoms? Bordetella pertussis Haemophilus influenzae Parainfluenza virus Respiratory syncytial virus

Correct Answer ( D ) Explanation: The patient has bronchiolitis, which is the most common lower respiratory tract infection in patients less than two years of age. It remains the leading cause for hospitalization in infants under one year of age. Bronchiolitis is most commonly caused by respiratory syncytial virus (RSV), but may be caused by other viral agents. Bronchiolitis is inflammation of the lower respiratory tract, which involves edema, epithelial cell necrosis, bronchospasm, and increased mucus production. The resultant lower airway obstruction causes increased work of breathing and wheezing. Bronchiolitis is a clinical diagnosis based on age under two years old, rhinorrhea, tachypnea, and wheezing. Unlike asthma or reactive airway disease, there is often no significant improvement with albuterol. There is often a history of several days of upper respiratory symptoms, such as rhinorrhea, mild cough, and mild fever. Rapid antigen tests, blood work, and radiographs are not usually needed. Radiographs may demonstrate hyperinflation and atelectasis, but do not show any focal infiltrates like with pneumonia. Bronchiolitis is usually self-limited, with respiratory status typically improving over 2-5 days. Management involves supportive care.

A 58-year-old man presents with chest pain. His electrocardiogram is shown above. A right-sided electrocardiogram is also performed and shows elevated ST segments in V4R and V5R. Which of the following medications is contraindicated? Aspirin Bivalirudin Heparin Nitroglycerin

Correct Answer ( D ) Explanation: The patient has evidence of a right ventricular infarction. In the majority of people, the right ventricle receives its blood supply from the right coronary artery. Inferior myocardial infarctions typically result from blockage in blood flow to the right coronary artery, and approximately 30 percent of inferior myocardial infarctions involve the right ventricle. It is critical to recognize the presence of right ventricular involvement because it is associated with increased mortality and cardiovascular complications, and treatment differs from other types of myocardial infarction. Right ventricular involvement can be diagnosed by obtaining a right-sided ECG. ST elevation in precordial lead V4R in the presence of an inferior myocardial infarction is diagnostic of right ventricular infarction. Patients with right ventricular involvement are dependent upon preload to maintain cardiac output and are at risk for hypotension if preload is reduced. Treatment of right ventricular infarction, in addition to early reperfusion, includes using fluids to maintain adequate preload, reducing right ventricular afterload, and using inotropes like dobutamine, if needed, to support the failing right ventricle. Nitroglycerin, which is commonly used in acute myocardial for its preload and afterload reducing effects, should not be used in right ventricular infarction because it can precipitate critical hypotension and cardiovascular collapse.

A 75-year-old man with a long-standing history of constipation presents complaining of abdominal pain for the last 2 days. He is now experiencing abdominal distention, constipation and inability to pass gas for the last 12 hours. On examination, his abdomen is distended and tympanitic to percussion. You obtain an upright abdominal X-ray as seen above. What is the most likely diagnosis? Cecal volvulus Intussusception Irritable bowel syndrome Sigmoid volvulus

Correct Answer ( D ) Explanation: The patient is suffering from a sigmoid volvulus, an intestinal obstruction that resulted when a non-fixed, redundant loop of the sigmoid colon twisted and obstructed the lumen. Sigmoid volvulus is more common in elderly, debilitated patients with a history of chronic constipation. The hallmark symptoms include abdominal pain, distention, and constipation. If the twist involves the vascular supply, the patient can develop gangrenous bowel. If this occurs, the patient may have associated fever, chills, peritonitis and cardiovascular instability. Plain films of the abdomen are often sufficient to make the diagnosis. Findings include a grossly distended loop of colon lacking haustral markings and classically have a "bent inner tube" appearance in which the loop points to the RUQ. Contrast enema will show the characteristic "bird's beak" appearance. If there is no concern for underlying gangrene, endoscopic detorsion should be attempted. Surgical consultation should be placed and the patient should be admitted for definitive repair.

57-year-old man presents to the ED with severe dyspnea and frothy sputum. His wife notes he has a "blood problem" that his doctor is monitoring. On physical exam, you note bilateral pulmonary rales to the mid-thorax. Vital signs are BP 124/78 mm Hg, HR 140 beats per minute, RR 35 breaths per minute, and T 99.5°F. Which of the following is the most common etiology of this disease process? Acute myeloid leukemia Multiple myeloma Polycythemia vera Waldenstrom macroglobulinemia

Correct Answer ( D ) Explanation: This patient is exhibiting signs and symptoms associated with hyperviscosity syndrome. Hyperviscosity syndrome is defined as elevated serum viscosity that causes sludging, decreased microvascular perfusion, and vascular stasis. The most common cause of hyperviscosity syndrome is elevated serum protein, most often secondary to Waldenstrom macroglobulinemia or less often due to multiple myeloma. It can also be caused by marked leukocytosis with > 100,0000 cells/mm3 or erythrocytosis from diseases such as leukemias, polycythemia vera, or hemoglobinopathies like sickle cell disease, respectively. Signs and symptoms of hyperviscosity syndrome are primarily secondary to end-organ ischemia creating mimics of disease processes such as myocardial ischemia, stroke, pulmonary infarction, congestive heart failure, renal failure, mucosal hemorrhages, or visual disturbances secondary to retinal ischemia. Laboratory studies will yield significant leukocytosis or hyperproteinemia depending upon the etiology. Management of hyperviscosity syndrome also depends upon the etiology. Supportive care for any etiology includes intravenous fluid hydration and diuresis in tandem. Temporizing measures for leukemias include leukapheresis, patients with elevated proteins require plasmapheresis, and patients with polycythemia vera require phlebotomy. While acute myeloid leukemia (A), multiple myeloma (B), and polycythemia vera (C) are all potential etiologies of hyperviscosity syndrome, Waldenstrom macroglobulinemia is the most common etiology of this disease process.

A 54-year-old woman with a history of hyperthyroidism presents to the ED with fever, vomiting, palpitations, and tremors. Which of the following would you expect to find on physical exam? Dry, scaling skin Loss of eyebrow hair Maculopapular rash Ocular proptosis

Correct Answer ( D ) Explanation: This patient is exhibiting signs and symptoms consistent with thyroid storm. Thyroid storm is a rare, life-threatening hypermetabolic state caused by severe thyrotoxicosis. The most common precipitating factor is infection. Signs and symptoms of thyroid storm include bilateral ocular proptosis, anxiety, tremulousness, psychosis, obtundation, seizure, coma, fever, tachycardia (out of proportion to fever), high-output heart failure, circulatory collapse, diarrhea, and vomiting. Thyroid storm is a clinical diagnosis. Management of thyroid storm occurs in a stepwise fashion. Adrenergic blockade is the most important initial component of therapy. Propranolol, a nonspecific b-blocker, is used to decrease sympathetic hyperactivity and partially block peripheral conversion of T4 to T3. Antithyroid drugs should then be initiated. Propylthiouracil and methimazole are both options as both block the synthesis of thyroid hormone; however, propylthiouracil has the added benefit of decreasing conversion of T4 to T3 and works faster than methimazole. One hour after antithyroid drug administration, iodine should be given to inhibit the release of stored thyroid hormone. Steroids also serve as an adjunct to decrease peripheral conversion of T4 to T3.

A 93-year-old man on aspirin presents to the ED with epistaxis. On physical exam, you note bleeding from bilateral nares and down the posterior pharynx. You are unable to visualize the source of bleeding. Which of the following vessels is most likely the source of bleeding in this patient? Facial artery Kiesselbach plexus Labial artery Sphenopalatine artery

Correct Answer ( D ) Explanation: This patient is exhibiting signs and symptoms of posterior epistaxis. Posterior epistaxis is less common than anterior epistaxis and is most commonly due to bleeding from the sphenopalatine artery, located at the posterior aspect of the middle nasal turbinate. Patients with posterior epistaxis typically complain of bleeding from both nostrils. Inspection of the posterior pharynx may reveal profuse bleeding. In treating epistaxis, start by having the patient gently blow his nose or suction out the blood. If the bleeding is profuse, apply cotton balls soaked in a topical anesthetic and vasoconstrictor for at least five minutes. A good option is 1% tetracaine plus 0.05% oxymetazoline solution. In posterior epistaxis, this may not achieve hemostasis or allow visualization of the location of bleeding. Management of posterior epistaxis should be with either a Foley catheter or dual balloon pack. A 10 to 14 French Foley catheter with a 30 cc inflatable balloon may be inserted past the site of the bleeding and inflated with 5 to 7 cc of air or saline. It should then be pulled back onto the site of the posterior bleed and inflated until it is snug. An anterior nasal pack should then be placed in both nares. A dual balloon pack is placed by anesthetizing the nare and advancing the pack past the site of the bleeding. The posterior balloon is inflated with 5 to 7 cc of saline or air and pulled back onto the site of bleeding. It is the further inflated until it is snug. The anterior balloon is then inflated. The opposite nare should be packed as well. Complications of posterior epistaxis packing include aspiration, hypoxia, hypercarbia, and symptomatic bradycardia. Antibiotics should be administered after all packing; however, there is significant controversy regarding whether prescribing antibiotics actually prevents toxic shock syndrome as there is no evidence to support this. All patients with posterior packing should be admitted to a telemetry bed for further monitoring while the packing is in place.

A 60-year-old man with a history of chronic obstructive pulmonary disease presents by EMS in respiratory distress. He is diaphoretic and is sitting up in bed leaning forward. His vital signs are T 100.1ºF, HR 95 beats/minutes, RR 30 breaths/minute, BP 150/90 mm Hg, and oxygen saturation 79% on room air. There is poor air movement with prolonged expiratory wheezes. A nonrebreather face mask is placed, but his work of breathing and oxygen saturation are not improving. In addition to intravenous glucocorticoid and nebulized beta-agonist medications, which of the following is most appropriate next step in management? Ceftriaxone and azithromycin Endotracheal intubation Intravenous magnesium sulfate Noninvasive positive pressure ventilation

Correct Answer ( D ) Explanation: This patient is having an acute exacerbation of his chronic obstructive pulmonary disease (COPD). Acute exacerbations are characterized by worsening pulmonary symptoms that are brought on by pulmonary irritants or infection. The most common bacterial causes of infection are Streptococcus pneumoniae, Haemophilus influenzae and Moraxella catarrhalis. Other triggers include cold weather, narcotics, beta-blockers and hypoxia. Inflammatory mediators are released, leading to bronchoconstriction, pulmonary vasoconstriction and mucus secretion. This creates increased airway resistance and lung hyperinflation, both of which lead to increased work of breathing. Hypoxemia is the most life-threatening feature of COPD exacerbations. Patients are often tachypneic, tachycardic, and may be altered due to carbon dioxide retention. An arterial blood gas should be obtained to assess for acid-base status and management is guided accordingly. Emergency department management is aimed at improving oxygenation and ventilation, alleviating bronchospasm and treating the underlying cause. Therapeutic options include administration of supplemental oxygen, aerosolized beta-adrenergic agonists and anticholinergics (e.g., albuterol, ipratropium bromide), glucocorticoids (e.g., methylprednisolone 125 mg IV), and antibiotics as needed. Noninvasive positive pressure ventilation (NIPPV) is beneficial for patients with severe acute exacerbations who are acidemic (pH < 7.36), hypercapnic (PaCO2 > 50 mm Hg), hypoxemic (PaO2 < 60 mm Hg or SaO2 < 90 mm Hg), or who have severe dyspnea, respiratory fatigue or increased work of breathing. Ceftriaxone and azithromycin (A) are reasonable antibiotic therapies for treating COPD exacerbations, as most of them are caused by infection. However, in a hypoxic patient in respiratory distress, improving oxygenation and ventilation with NIPPV takes precedence and may preclude the need for invasive ventilation. Endotracheal intubation (B) may be needed for patients whose respiratory status does not improve with noninvasive ventilation or who have contraindications to NIPPV (e.g., respiratory arrest, hemodynamically unstable, mental status changes, severe agitation, active vomiting). Intravenous magnesium sulfate (C) may be beneficial in severe asthma exacerbations but is not typically used in the management of COPD exacerbations.

A 54-year-old man presents with abdominal pain, vomiting and fever. Physical examination reveals an ill-appearing man with a mass in the right groin as seen above. Which of the following is the most immediate next step required? Abdominal X-ray CT scan of the abdomen and pelvis Scrotal ultrasound Surgical Consultation

Correct Answer ( D ) Explanation: This patient presents with signs and symptoms concerning for an strangulated hernia and should have an emergent surgical consultation for possible operative management. A hernia occurs when part of the viscous protrudes from its surrounding tissue walls. There are three presentations seen with hernias. The first is a reducible hernia where the hernia is soft and non-tender (or minimally tender) and can easily be reduced through the neck of the defect. An incarcerated hernia is firm and painful and cannot be reduced through its defect. Lastly, a strangulated hernia occurs after incarceration and involves impairment of blood flow (arterial, venous or both) to the viscous. Patients with strangulated hernias typically present with bowel obstruction and are toxic appearing. The hernia site itself is firm and extremely painful to palpation. There are often changes to the overlying skin. A strangulated hernia is a true surgical emergency. Emergent surgical consultation should be obtained prior to imaging as patients will require operative management. A delay in definitive treatment can lead to bowel ischemia or infarction. Strangulated Hernia Vascular compromise of hernia contents Usually incarcerated prior to strangulation ABX Surgical emergency

A 65-year-old woman presents to the Emergency Department with acute onset vertigo. She denies associated decreased hearing or tinnitus. Vital signs are normal. Horizontal head impulse test shows a corrective saccade when the head is turned to the left. Dix-Hallpike maneuver elicits a leftward horizontal nystagmus that is fatigable. Which of the following mechanisms will provide the most definitive treatment of this condition? Dopamine receptor antagonism Inhibition of the reabsorption of sodium from the distal convoluted tubules Positive allosteric modulators of GABA type A receptors Relocation of free floating otoconia

Correct Answer ( D ) Explanation: When evaluating a patient with vertigo, it is important to differentiate between central and peripheral causes. The history and physical exam findings that suggest a peripheral cause include rapid onset, increase in intensity with head movement, quality defined as intense spinning, change in sensation of hearing, horizontal or rotary nystagmus that does not change direction, normal neurologic exam, positive and fatigable Dix-Hallpike test, positive horizontal head impulse test, and negative test of skew. A central cause is more likely if the onset is insidious, quality is ill-defined, head positional changes do not affect the severity of the vertigo, symptoms are not proportional to stimulus, nystagmus is vertical or horizontal and changes direction, positive neurologic findings are present, negative horizontal head impulse test, and positive test of skew. The horizontal head impulse test is useful in differentiating between a peripheral and central cause by assessing the vestibulo-ocular reflex. T perform the head impulse test, have the patient fixate on a visual target. Then rapidly rotate the patient's head from the center position to the approximately 40 degrees to the right and then back to the center. An intact vestibulo-ocular reflex compensates by rapidly and smoothly moving the eyes in the opposite direction of the head rotation. If the vestibulo-ocular reflex is impaired, the patient will not be able to maintain their gaze on the visual target and instead will exhibit a rapid simultaneous movement of both eyes (corrective saccade) to reacquire fixation upon the visual target. The test should then be repeated on the left side. Patients with acute vestibular syndrome and an abnormal horizontal head impulse test are likely to be suffering from a peripheral vestibular lesion. If there is no corrective saccade, this is highly suspicious for a central lesion (e.g. stroke). One of the most common causes of peripheral vertigo is benign paroxysmal positional vertigo (BPPV) which is a mechanical disorder of the inner ear causing transient vertigo and associated nystagmus precipitated by certain head movements. BPPV is caused by inappropriate activation of a semicircular canal by free-floating otoconia (stones in the semicircular canals) that have become displaced from macula by aging, head trauma, or labyrinthine disease. It is most common after the age of 50 years. The Dix-Hallpike test can aide in the diagnosis of BPPV, but it should not be attempted in patients with a carotid bruit. To perform the test, start with the patient seated upright, rotate the head 30-45 degrees to one side. Keeping the head in this position, rapidly bring the patient supine until the head is 20 degrees below the level of the examining table. Rotatory nystagmus following a latency of < 30 seconds is considered a positive test; the nystagmus exhibits rapid eye torsions toward the affected ear and fatigues after 10-40 seconds. The side exhibiting the positive test is the side of the lesion. The most definitive treatment for BPPV is the Epley maneuver which uses gravity to relocate free floating otoconia along the semicircular canals and into the utricle, where they are unlikely to cause vertigo.

Rapid Review Cushing's Triad:

Cushing's Triad: Pre-terminal findings Suggestive of increasing intracranial pressure with herniation or impending herniation Hypertension, bradycardia, irregular respirations Indicative of ischemia and arterial compression

Which of the following is most likely to be associated with a bilateral interfacetal dislocation? Anterior cord syndrome Brown-Sequard syndrome Central cord syndrome Complete cord transection

Correct Answer ( D ) Explanation: Bilateral facet dislocation is an unstable injury that occurs from forceful hyperflexion of the neck. It occurs when the articular masses of one vertebra dislocate anteriorly and superiorly from the articular surface of the vertebra below it causing anterior displacement of the spine. This injury involves forceful disruption of multiple structures at the level of the injury, including all ligamentous structures, the articular facet joints, and the intervertebral disc. This is commonly associated with a complete spinal cord injury due to transection of the cord at the level of the injury. Diagnosis is made by radiographic evidence of displacement of the superior vertebral body anteriorly more than one half of its width

A 37-year-old man is brought into the emergency department after being stabbed in the flank during a bar fight. On examination, the patient has 2/5 strength and decreased vibration sense in his left lower extremity as well as decreased pinprick sensation in his right lower extremity. Which of the following is the most likely pattern of this patient's injury? The corticobulbar tract was transected on the left The corticospinal tract was transected on the right The dorsal column was transected on the right The spinothalamic tract was transected on the left

Correct Answer ( D ) Explanation: Brown-Séquard syndrome is a hemicord syndrome which manifests as ipsilateral loss of motor function, vibration, and proprioception and contralateral loss of pain and temperature sensation. Brown-Séquard syndrome most commonly occurs due to penetrating trauma to the spine but can less commonly result from lateral cord compression due to spine fractures, hematomas, herniated discs, or tumors. Loss of motor function occurs due to transection or compression of the ipsilateral corticospinal tract, which carries upper motor neurons from the motor cortex of the brain. The same is true of the ipsilateral dorsal column, which carries sensory neurons for proprioception and vibration to the sensory cortex. In contrast, the spinothalamic tract, which carries pain and temperature sensation, travels two levels along the spinal tract before decussating (crossing over), resulting in loss of pain and temperature sensation contralateral to the lesion. Thus, in Brown-Séquard syndrome, the spinothalamic tract is always damaged contralateral to the side of pain and temperature loss, in this case, on the left. he corticobulbar tract (A) is a motor tract that carries the upper motor neurons for the cranial nerves and therefore terminates in the brainstem. It is typically not damaged in Brown-Séquard syndrome. The corticospinal tract (B) carries motor neurons and is damaged on the ipsilateral side of the loss of motor function in Brown-Séquard syndrome. Since the patient described above has loss of motor function on the left side, it can be determined that the corticospinal tract was transected on the left side. The same is true of the dorsal column (C) which is damaged on the same side as the proprioception and vibration deficits, in this case on the left side.

Which of the following findings on electrocardiogram are indicative of left main coronary artery occlusion? Biphasic T waves in leads V2-3 Coved ST segment elevation > 2 mm followed by a negative T wave in leads V1-3 Horizontal ST depression with tall, broad R waves and upright T waves in leads V1-3 ST elevation in aVR > VI with horizontal ST depression in I, II and V4-6

Correct Answer ( D ) Explanation: By correctly interpreting an electrocardiogram, it is possible to determine the distribution of an acute myocardial infarction and the most likely coronary artery involved. ST segment elevation in lead aVR greater than 1 mm or greater than the elevation seen in the ST segment of V1 should prompt concern for occlusion of the left main coronary artery. The finding of ST elevation in aVR that is greater than that seen in VI can distinguish occlusion of the left main coronary artery from the left anterior descending artery with a sensitivity and specificity of approximately 80%. It was also found that the extent of ST segment elevation in aVR correlated with increased mortality. Other ECG findings consistent with left main occlusion include horizontal ST depression in leads I, II, and V4-6. Deep symmetric T wave inversions or biphasic T waves in leads V2-3 (A) are seen in Wellens' syndrome and are highly specific for critical stenosis of the left anterior descending artery. The ECG pattern is seen when the patient is pain-free but they often present after an episode of angina. Urgent catheterization is indicated as these patients are at high risk for extensive anterior wall MI. Coved ST segment elevation > 2 mm followed by a negative T wave in leads V1-3 (B) is referred to as Brugada sign. Taken in conjunction with a family history of early sudden cardiac death, documented ventricular fibrillation or ventricular tachycardia, or a history of syncope, this ECG finding identifies patients with a high incidence of sudden cardiac death. Definitive treatment is with placement of an AICD. A posterior MI is characterized by horizontal ST depression with tall, broad R waves and upright T waves in leads V1-3 (C). Posterior MIs are often seen in conjunction with inferior or inferolateral infarctions.

A ten-year-old boy collapses on the football field and is quickly evaluated by the sports medicine doctor. He had been practicing on the field for approximately two hours, without any rest. His temperature is 105°F and his skin is sweaty. On testing his mental status, he can say his name and age but does not know where he is. Which of the following is the most appropriate initial treatment for this boy? Cool the body with fans Offer oral rehydration solution Provide cold water to drink Whole body cold-water immersion

Correct Answer ( D ) Explanation: Heat stroke is a severe illness with mortality rates of up to 50%. It is characterized by CNS abnormalities and may lead to multi-organ damage. It may occur in the setting of intense physical activity during sports play. Rectal temperature is usually > 104°F, and patients will have profuse sweating. In contrast, "classic" heat stroke is seen in elderly patients and is usually a slower onset, with the physical exam notable for dry, hot skin. Treatment of children with heat stroke consists of immediate whole body cooling, usually performed by cold water immersion. Rapid cooling should be continued until the temperature decrease to 101 or 102°F. Careful monitoring of the airway, breathing, circulation, temperature, and CNS status is crucial. IV fluids should also be initiated as soon as possible, at a rate of 800 ml/m2 in the first hour.

A 2-year-old unvaccinated boy is rushed to the Emergency Department with concerns for respiratory distress. His mother reports that he awoke in his usual state of health and was playing alone in the playroom when mom heard him have a coughing fit. Since that time, he has been choking and coughing, has had inspiratory stridor, and developed significant increased work of breathing. What is the most likely etiology of his stridor? AAcute asthma exacerbation BCroup secondary to parainfluenza virus CEpiglottitis secondary to H.influenzae DForeign body aspiration

Correct Answer ( D ) Explanation: The child's sudden onset of respiratory distress in an unobserved setting is suggestive of a foreign body aspiration. The child's inspiratory stridor is indicative of an upper airway obstruction, and he most likely has a laryngotracheal foreign body aspiration. Aspiration into the bronchi, and especially the right mainstem bronchus, is significantly more common than laryngotracheal aspiration. However, objects with sharp or irregular edges are more likely to become lodged in the upper airway. Children with laryngotracheal foreign body aspiration require immediate medical attention for airway stabilization and removal of the foreign body. Notably, if a child is able to cough, the airway obstruction is partial. A blind finger sweep of the mouth is not recommended, as this may lead to dislodgment of the foreign body and subsequent complete airway obstruction. Radiographic evaluation of the neck is recommended, but only approximately ten percent of foreign bodies are radio-opaque. Instead the radiograph may show subglottic swelling. However, radiographs are insufficient to rule out a foreign body. If the diagnosis is suspected, it should be further evaluated by bronchoscopy. Epiglottitis secondary to H.influenzae (C) is always a consideration in children with acute, severe upper airway obstruction. It is especially important to maintain a high index of suspicion in unvaccinated children. Many of the symptoms of epiglottitis are shared with laryngotracheal foreign body aspiration. However, children with epiglottitis are more likely to have fever and appear toxic.

A five-year-old girl is rushed to the ED because of possible ingestion. She was unattended for a few minutes and later found playing with her grandmother's bag. The grandmother's bag contains over-the-counter medications and herbal products. The girl complained to her grandmother that her ears were buzzing. Upon arrival at the ED, the girl had one episode of vomiting. Her examination was normal except for diaphoretic skin. Which of the following is the medication that most likely caused her symptoms? Acetaminophen Chlorpheniramine Ibuprofen Oil of wintergreen

Correct Answer ( D ) Explanation: The girl manifests signs and symptoms of salicylism. Oil of wintergreen contains 5 g of salicylate in one teaspoon and ingestion of very small volumes of this product has the potential to cause severe toxicity. Early signs of acute salicylism include nausea, vomiting, diaphoresis, and tinnitus. Moderate salicylate toxicity can manifest as tachypnea, tachycardia, and altered mental status. Signs of severe salicylate toxicity include hyperthermia, coma, and seizures. For the patient who presents soon after an acute ingestion, initial treatment should include gastric decontamination with activated charcoal. Initial therapy focuses on aggressive volume resuscitation and prompt initiation of sodium bicarbonate therapy in the symptomatic patient, even before obtaining serum salicylate levels. The primary mode of therapy is serum and urinary alkalinization wherein it enhances the elimination of salicylates by converting salicylate to its ionized form, "trapping" it in the renal tubules, and thus enhancing elimination.

A three-year-old girl was brought by her parents to the ED because of a possible ingestion. The girl was caught with an open bottle of amitriptyline although the mother is not sure how many tablets were missing. The girl did not experience any mental status changes, vomiting, abdominal pain, or fever. She arrived at the ED within 60 minutes of her possible ingestion. On examination, she is alert, active, with tachycardia, dry mucous membranes, and pupils 5-6 mm. Which of the following is the antidote for this type of ingestion? AFlumazenil BN-acetylcysteine CNaloxone DSodium bicarbonate

Correct Answer ( D ) Explanation: The girl possibly took amitriptyline, which is a tricyclic antidepressant (TCA). Cardiovascular and CNS symptoms dominate the clinical presentation of TCA toxicity. Patients often develop features of the anticholinergic toxidrome such as delirium, mydriasis, dry mucous membranes, tachycardia, hyperthermia, mild hypertension, urinary retention, and slow GI motility. CNS toxicity can include lethargy, coma, myoclonic jerks, and seizures. Sinus tachycardia is the most common cardiovascular manifestation of toxicity. Patients can also develop widening of the QRS complex, premature ventricular contractions, and ventricular arrhythmias. Initial management should be directed to supporting vital functions. ECG should be obtained as soon as possible and followed serially to monitor for progression of toxicity. Sodium bicarbonate is the antidote of choice and works via overcoming the sodium channel blockade by providing a sodium load and via inducing an alkalosis to decrease drug binding to sodium channels. Indications for sodium bicarbonate include a QRS duration > 100 ms, ventricular dysrhythmias, and hypotension.

A two-year-old girl is brought to your clinic by her father because of abdominal pain. This is accompanied by decreased energy, vomiting, and constipation. CBC with peripheral smear shows microcytic anemia with basophilic stippling. Which of the following is the most likely diagnosis? Acute lymphocytic leukemia Folate deficiency Iron deficiency anemia Lead poisoning

Correct Answer ( D ) Explanation: The girl's symptoms and laboratory value are most likely due to lead poisoning. Since lead blocks iron from being incorporated into heme, lead poisoning can look like iron deficiency in which there is microcytic anemia. To differentiate this from iron deficiency, look for basophilic stippling of RBCs accompanied by gastrointestinal (GI) and central nervous system (CNS) symptoms. GI symptoms of lead poisoning include anorexia, abdominal pain, vomiting, and constipation, often occurring and recurring over a period of weeks. CNS symptoms are related to worsening cerebral edema and increased intracranial pressure. These symptoms include headaches, change in mentation, lethargy, papilledema, seizures, and coma. Lead lines are found on imaging of long bones. The most common pathway by which lead enters the body is through nonnutritive hand-to-mouth activity of young children. In most cases, lead is ingested, either as a component of dust licked off surfaces or in swallowed paint chips, through water contaminated by its flow through lead pipes or brass fixtures, or from food or liquids contaminated by contact with lead-glazed ceramic ware.

Which of the following distinguishes traumatic injuries of the thoracic spine from injuries to the remainder of the spinal column? The thoracic spine requires less force to injure than the cervical spine Thoracic spine injuries are less likely to be associated with complete cord injuries than are injuries at the thoracolumbar junction Thoracic spine injuries are more common due to relative mobility of thoracic vertebrae Thoracic spine injuries are more likely to be associated with cord injuries

Correct Answer ( D ) Explanation: The thoracic spine is more rigid than other segments of the spine and its relative stiffness is furthered by articulation with the ribs. Due to its rigidity, the thoracic spine is less commonly injured than other spinal segment and requires a more severe traumatic force to cause injury. As such, thoracic spine injuries are commonly associated with intrathoracic injuries due to the severity of the traumatic force required to injure the thoracic vertebrae. Furthermore, the spinal canal in the thoracic spine is more narrow than in the cervical and lumbar spine, increasing the rate of spinal cord injury in trauma of the thoracic spine. Spinal cord injuries associated with thoracic spine injuries are more likely to be complete cord lesions than spinal cord injuries associated with cervical, lumbar, or transitional zone trauma.

A 68-year-old man presents to the ED after a fall down twelve stairs. On physical exam, you note a large parietal scalp hematoma. His non-contrast computed tomography scan of the head is shown above. Which of the following is the most likely diagnosis? Basilar skull fracture Epidural hematoma Subarachnoid hemorrhage Subdural hematoma

Correct Answer ( D ) Explanation: This patient's imaging and physical examination are consistent with a traumatic acute subdural hematoma. A subdural hematoma is a collection of blood between the dura and the arachnoid mater. Subdural hematomas occur most commonly in individuals with brain atrophy, such as alcoholics and elderly patients. Subdural hematomas are significantly more common than epidural hematomas. Subdural hematomas occur when there is rupture of the bridging veins from movement of the brain relative to the skull, such as that seen with acceleration-deceleration injuries. This results in blood filling the potential space between the dura and arachnoid. Subdural hematomas may be acute, subacute, or chronic. Acute subdural hematomas are characteristically seen after a traumatic head injury. Patients typically present with headache, mental status changes, seizures, or focal deficits. Subacute or chronic subdural hematomas may occur without reported history of trauma in up to 50% of patients. These patients may present with symptoms similar to those seen in acute subdural hematomas; however, they may also present with more indolent symptoms such as personality changes, depression, or memory deficits. Subdural hematoma is diagnosed on non-contrast CT of the head and appears as a crescent-shaped hematoma that may cross suture lines. Management of subdural hematomas includes neurosurgical consultation for possible surgical evacuation; however, small subdural hematomas may be closely observed with serial non-contrast CTs of the head. Indications for emergent surgery in a patient with a subdural hematoma include neurologic deterioration or > 5 mm midline shift on CT.

Question: What toxicity is associated with use of nitroprusside?

Cyanide poisoning.

Question: What is the classic finding of hypertrophic cardiomyopathy on echocardiogram?

Disproportionate septal hypertrophy.

Question: What is the treatment of Stevens-Johnson syndrome and toxic epidermal necrolysis?

Discontinuation of the offending medication, fluid resuscitation, and meticulous wound care. Intravenous immunoglobulin may be used in severe cases.

Ransons Criteria

During the initial 24 hours, severe acute pancreatitis can be predicted using clinical, laboratory, and radiologic risk factors, many of which have been incorporated into scoring systems such as the systemic inflammatory response syndrome (SIRS) score, the Acute Physiology and Chronic Health Examination (APACHE) II score, the bedside index of severity in acute pancreatitis score, and the computed tomography (CT) severity index.

G6PD Deficiency Overview

G6PD Deficiency X-linked recessive Asymptomatic until exposed to oxidative stress Antimalarials, sulfonamides, nitrofurantoin, methylene blue, fava beans, vitamin K Heinz bodies Hemolytic anemia

Rapid Review Ethylene Glycol Toxicity

Ethylene Glycol Toxicity Patient will be complaining of nausea and vomiting PE will show CNS depression and acute mental status change Labs will show calcium oxalate crystals in the renal tubules, renal tubular necrosis, hypocalcemia Treatment is fomepizole Comments: Commonly found in antifreeze

Question: Which anatomical regions of the brain does infectious encephalitis have a predilection for?

Gray matter of the temporal lobes and inferior frontal lobe. Encephalitis Arboviruses, HSV Fever, HA, AMS Psychiatric sx MRI

Alcoholic Hepatitis Question: What chronic histologic changes occur after continued insults to the liver?

Hepatic fibrosis.

Question: What is another serious complication of right ventricular infarction?

High-degree heart block.

Implantation vs. Abortion Bleeding

Implantation Bleeding: Occurs in ~9% of women. Does not require IUP diagnosis Typically light bleeding lasting around 1 day. Abortion Inevitable: vaginal bleeding with open os Threatened abortion: vaginal bleeding with closed internal os Incomplete: partial passage of products of conception (POC) Complete: complete passage of POC Missed: fetal death <20 weeks without POC passage Missed abortion complications: infection, coagulopathy Rh-negative women: RhoGAM

Appendicitis Question: What is the most common physical exam finding in acute appendicitis?

Pain localized to the right lower quadrant. Appendicitis Patient will be complaining of fever, pain that began periumbilical then moved to RLQ, nausea and anorexia PE will show Psoas sign (RLQ pain on extension of right hip), Obturator sign (RLQ pain on internal rotation of flexed right hip), Rovsing sign (right lower quadrant pain when the left lower quadrant is palpated) Diagnosis is made by ultrasound, CT Most commonly caused by fecalith Treatment is surgery

Endometriosis

Patient will be a woman Complaining of pre or midcycle dysmenorrhea, dyspareunia, dyschezia (painful bowel movement) PE will show uterosacral nodularity or a fixed or retroverted uterus Diagnosis is made by laparoscopy Most common site is ovaries Mittelschmerz (B) is unilateral midcycle pelvic pain related to ovulation. Pelvic inflammatory disease (C) is characterized by fever, pelvic pain, bilateral adnexal pain, cervical motion tenderness, and vaginal discharge. Patients with a ruptured ovarian cyst (D) often present with acute onset of unilateral pelvic pain which may be accompanied by nausea and vomiting.

Septic Arthritis

Patient will be complaining of fever, monoarticular pain with decreased ROM Labs will show WBC > 50,000 with > 75% PMNs Diagnosis is made by arthrocentesis Most commonly caused by: Age < 35: N. gonorrhea, S. aureus overall Treatment is IV ABX, surgical washout

Pediculosis Capitis

Permethrin is first-line therapy for treatment of pediculosis capitis (head lice) in all populations, including pregnant and lactating women. It has a 95% cure rate when used correctly. Permethrin causes a respiratory paralysis of the adult louse, however it is not effective on recently laid eggs. It should be applied to dry hair and requires repeat treatment in 2 weeks to remove any eggs that were present during the first application. It is available as an over-the-counter medication and in a prescription strength formula. Pediculosis capitis is caused by infestation with Pediculus humans var. capitis. Lice are obligate human parasites that cannot survive off of a host for more than 10 days. However fertile eggs can survive for 3 weeks. The lice feed by piercing host skin, injecting irritating saliva, and sucking blood. Itching develops as people often develop hypersensitivity to the saliva and fecal material of the louse. The female louse lays six eggs, known as nits each day. The nits are attached to the host's hair and are difficult to remove, which differentiates a nit from dandruff. Lice are transmitted by close personal contact and contact with infected objects. Other findings include posterior cervical and occipital lymphadenopathy, excoriation, and secondary bacterial infection from scratching. Infection often occurs in outbreaks of schools, daycare facilities, nursing homes, and families.

Lice Treatment

Permethrin is first-line therapy for treatment of pediculosis capitis in all populations, including lactating women (B). Lindane use should be avoided in anyone weighing under 50 kilograms and children under the age of 2 (A), as it has been associated with seizure activity. Selenium sulfide shampoo (D) is used to treat fungal infections of the scalp and skin, however is ineffective in the treatment of head lice

Question: What pulmonary infection is associated with pneumothorax in AIDS patients?

Pneumocystis jirovecii pneumonia.

Question: What underlying disease processes should you suspect in a trauma patient with severe subcutaneous emphysema?

Pneumothorax or tracheobronchial tree disruption.

Rapid Review Salicylate Toxicity:

Salicylate Toxicity: Aspirin, wintergreen, Pepto-Bismol Respiratory alkalosis + anion gap metabolic acidosis Hypoglycemia Tinnitus Rx: AC (if < 1 hour from ingestion), urine alkalinization, K+ Hemodialysis indications: Level > 100 Coma Rising levels despite alkalinization Renal failure Pulmonary edema Altered mental status Clinical deterioration

Shizophrenia Overview

Schizoaffective disorder (A) is a condition in which symptoms of schizophrenia exist concurrently with a mood disorder (mania or major depression). Schizophreniform disorder (C) is characterized symptoms of schizophrenia lasting longer than one month but less than six months. Schizotypal personality disorder (D) is a cluster A personality disorder and part of the schizophrenia spectrum of disorders. It is characterized by odd beliefs and eccentric behaviors, thoughts, dress and style. Patients often do not engage in interpersonal relationships and display a pattern of social deficits.

Question: Where should the needle be placed for an emergent pericardiocentesis?

Subxiphoid

Question: What emergent procedure needs to be performed in a patient with paraphimosis when all reduction techniques have failed and ongoing ischemia is clearly evident?

Superficial dorsal slit incision of the constricting band

Question: Does surgery play a role in the management of trigeminal neuralgia?

Surgical options, including microvascular decompression and ablative procedures, may be considered in patients who are refractory to medical therapies.

Stroke Distribution

The lenticulostriate arteries (A) are small penetrating arteries that supply the deep structures of the brain. Ischemia of the left lenticulostriate arteries would cause right-sided symptoms, which can be motor, sensory, or both depending on the deep brain structure affected by the stroke. The posterior cerebral artery (B) supplies the occipital lobe. Symptoms of a posterior cerebral artery stroke are subtle and may include ataxia, vertigo, contralateral motor weakness, and visual field cuts. The middle cerebral artery (D) is the artery most commonly involved in ischemic stroke. Symptoms depend on what area of the artery is involved as well as which lobe is dominant in the patient. Typical symptoms include contralateral hemiparesis (involving both the arm and leg, in contrast to an ACA stroke), facial droop, and sensory loss. Involvement of the dominant lobe will result in aphasia, which may be expressive, receptive, or both.

Question: What is the discriminatory zone in relation to serum β-hCG levels?

The minimum level that the gestational sac can be visualized consistently on ultrasound.

Correct Answer ( A ) Explanation: The image represents impetigo, a pustular eruption most commonly seen in preschool children. Acute poststreptococcal glomerulonephritis (APSGN) results from an antecedent infection of the skin or throat caused by nephritogenic strains of group A beta-hemolytic streptococci. APSGN usually occurs 10 days after pharyngitis and 14 days after skin infection. The incidence is decreasing in the United States but is still common in some rural areas. APSGN is not well understood, but it probably results from the deposition of circulating immune complexes in the kidney. This results in decreased glomerular filtration, allowing proteins to flow freely into the urine. Urinalysis shows significant blood and protein with RBC casts in 60% of cases. Pyuria with granular or hyaline casts also may be found.

The rash seen above is associated with which of the following conditions? AAcute glomerulonephritis BArthritis CClostridium difficile colitis DSecondary syphilis

Childhood Respiratory Distress

The steeple sign (A) is most often seen in croup and stems from subglottic narrowing of the trachea. Croup most often presents with a barking cough, low-grade fever, stridor, and an overall well-appearing child. Subcutaneous emphysema (B) may be seen in patients with severe asthma exacerbations, pneumothoraces, or esophageal rupture. Patients with severe asthma may have wheezing or simply minimal breath sounds due to poor air movement. A pneumothorax would present with asymmetric breath sounds on auscultation. Esophageal rupture occurs most often after severe retching and vomiting or procedural dilation of the esophagus. Widening of the prevertebral soft tissues (D) is the classic presentation associated with a retropharyngeal abscess. Retropharyngeal abscesses usually occur in children under six years of age, and more often in children less than two years of age. While they also often present with drooling, fever, and stridor, the hallmark of a retropharyngeal abscess is limited neck extension and would typically be included in the question stem.

Question: True or False: Asymptomatic bacteriuria in pregnancy should be treated with antibiotics.

True.

True or False: Most cases of adult intussusception are due to underlying malignancy?

True.

True or false: Maternal use of nonsteroidal anti-inflammatory medications can cause fetal closure of the ductus arteriosus?

True.

Question: Is there a familial inheritance of bipolar disorder?

Yes, first-degree relatives have an increased risk of development of bipolar disorder.


Related study sets

Water Soluble Vitamins for Test 3

View Set

XCEL Chapter Exam - Life Provisions

View Set

study guide for exam 2 human anatomy & physiology

View Set

MGT Exam 3 - Intrapersonal Conflicts

View Set

Chapter quiz: insurance terms and related concepts

View Set